There is a Greenhouse Effect on Venus

I‘m peppered with emails asking me if articles like this one (which claims there is no Greenhouse Effect at all on Venus) could be right.

Michael Hammer has some 20 patents in spectroscopy, and he explains why the Greenhouse Effect — where CO2 and other gases absorb and emit infra red — is very real, and backed by empirical evidence. The calculations using the Stefan-Boltzmann Law on the atmosphere of Earth and Venus, argue that the Greenhouse Effect is not-detectable. But not-detectable (by that method) does not “prove” the effect is zero. Other methods — like satellite observations of Earth’s atmosphere, and countless lab experiments, tell us that the Greenhouse Effect is real. (The Stefan-Boltzmann Law is used to create the first graph below). Huffman’s calculations suggest other factors are more important than greenhouse gases (with which we heartily agree) and that Hansen et al were barking up the wrong tree by pretending that Venus “shows” us anything much about the Greenhouse Effect. (Indeed, the IPCC mention “Venus” in their first Assessment Report back in 1990 as one of the three key reasons.)

So here in middle-of-the-road centrist land, the people who claim Earth could become more like Venus are wildly exaggerating, but the people who claim  that Venus “proves” that the Greenhouse Effect doesn’t exist are just as wrong.

For your average reader, sorry it is esoteric, but there will be avid interest by some science-aficionados in this topic.

— Jo

Guest Post: Michael Hammer

Joanne sent me an email to ask my opinion on the “Huffman blog”: There is no Greenhouse Effect on Venus.

Let’s start with a plot of the long wave infrared emission from Earth as seen by the Nimbus satellite.  This is not model output, it is real experimental data.  A plot is shown below:

Nimbus emissions earth to space infra red, greenhouse gas absorption

Caption from Petty: Fig. 6.6: Example of an actual infrared emission spectrum observed by the Nimbus 4 satellite over a point in the tropical Pacific Ocean. Dashed curves represent blackbody radiances at the indicated temperatures in Kelvin. (IRIS data courtesy of the Goddard EOS Distributed Active Archive Center (DAAC) and instrument team leader Dr. Rudolf A. Hanel.)

The horizontal axis shows wavenumbers, which are the reciprocals of wavelengths.  The vertical axis shows the energy density of the radiation from the Earth observed by Nimbus.  The dotted overlaid traces represent the emission spectrum from black bodies at various temperatures (calculated from Plank’s law which is known with a very high degree of surety).  Without any atmosphere, Earth’s emission pattern as seen from space would look like one of these dotted lines.

Note the big bite out of the spectrum at around 660 wavenumbers and the smaller bite at around 1000 wavenumbers.  The former is at the CO2 absorption line and the latter at the Ozone (O3) absorption line.  Those two bites represent energy that is not being radiated to space that would be if there was no atmosphere.  In short it is the signature of a green house gas reducing Earth’s radiation to space at the green house wavelengths.  I invite those who disagree to give an alternate explanation for what is causing these notches.

So how to explain the Venus data cited in Mr Huffman’s blog?  Shown below is a plot of the temperature versus altitude profile for Venus with the corresponding profile for Earth overlayed (from site http://www.datasync.com/~rsf1/vel/1918vpt.htm )

The similarly is not as close as claimed.  In fact the two profiles are only similar between 50km and 60 km and quite different at other altitudes.

Temperature and Pressure Venus

...

Source: Shade Tree Physics. Earth data based on  The Digital Dutch 1976 Standard Atmosphere Calculator. Venus:  J.M. Jenkins, P. G. Steffes, D.P. Hinson, J.D. Twicken, and G.L. Tyler in their article, Radio Occultation Studies of the Venus Atmosphere with the Magellan Spacecraft, Icarus, Vol. 110, 79-94, 1994.

OK but what about that bit between 50km and 60km – surely that’s till needs some explanation.  Well distance from the sun is not the only thing affecting received sunlight.  Venus is covered with clouds which according to the same source have their tops at an altitude of around 60 km.  These clouds give Venus an albedo of 0.6 which means 60%of the suns energy is reflected back out to space, 40%is absorbed.  Earth by comparison has an albedo of 0.3 which means 70% is absorbed.  The net difference in energy absorbed between Earth and Venus is thus the inverse ratio of distance to the sun squared time the difference in fraction of energy absorbed.

=   (93/67) 2 *  0.4/0.7  =  1.1

Venus only absorbs 10% more solar energy than does Earth yet its temperature at equivalent atmospheric pressure is  66C vs 14C.  The difference in black body emission is 749 watts/sqM versus 390 watts/sqM.  The close equivalence cited by Mr Huffman would appear to only exist if one ignores the difference in albedo.

The green house effect is real however that does not mean its incremental impact is anywhere near as great as claimed by AGW advocates.  The real debate is about how large the effect is.  Specifically, whether the direct effect of about 0.8C per doubling of CO2 is amplified by claimed massive positive feedbacks or diminished by negative feedbacks such as occur in all stable systems.  But that is a different issue.

Often combined with the argument that there is “no greenhouse effect on Venus” is the argument that its not possible due the second law of thermodynamics, Michael Hammer helped to write: Why greenhouse gas warming doesn’t break the second law of thermodynamics and So what is the Second Darn Law?

————————————-

Michael Hammer is an electrical engineer who has spent over 30 years conducting research for a major international spectroscopy company.  In the course of this work he generated around 20 patents which have been registered in multiple countries.  Patents are rarer and more rigorous than peer reviewed papers, only available for economically valuable work, and costing thousands of dollars to process and maintain. Spectroscopy deals with the interaction between electromagnetic energy (light) and matter and it is this interaction which forms the basis of the so called “green house effect” in the atmosphere. Other posts by Michael Hammer.

WUWT has a post by Ira Glickstein on the Greenhouse effect as seen in the emission spectra.


 

8.6 out of 10 based on 8 ratings

321 comments to There is a Greenhouse Effect on Venus

  • #
    Asmilwho

    Wearing my pedantic hat, i’d like to point out that it’s the “Stefan-Boltzmann” law and not the “Stephan Boltzmann” law or even the “Stefan Boltzmann” law.

    It’s named after Josef Stefan and Ludwig Boltzmann, not after “Stephan Boltzmann”, or even “Stefan Boltzmann” .

    There, I feel better now. I knew my physics degree would be for something, someday 😉

    REPLY: Thanks 🙂 I do appreciate the proof reading. Fixed! JN

    30

  • #

    As some of you know, I’ve been Posting on this, er, debate, for more than three and a half years now.
    I started with my very first Post in March of 2008.

    In that Post (and it’s a pretty amateur thing really) I mentioned that I had heard that there was in fact warming on Mars that had been documented.

    Thinking that this might somehow be indicative of our Solar System, I wanted to see if there was any correlation between this warming on Mars and what was happening here on Earth, I discussed the matter with a retired Nuclear Physicist, Professor Ron Neilsen. He hosted a slot on talk back radio on a Community radio station on the Gold Coast. I joined the queue and the show finished before my call got to air. Professor Neilsen talked with me off air, thankfully, and because of that we spoke for half an hour.

    He didn’t seem to think the two were related, but he was interested, as he hadn’t heard of it.

    Rather than go over it all again, I’ll just link to that first Post of mine, again, seemingly shameless of me to do so here at Joanne’s site. Beware, as this first Post shows that I’m just beginning, so it’s a pretty amateur effort.

    Kyoto – A Perspective (Part1)

    Professor Neilsen also has a site of his own at the following link.

    Critical Trends Shaping the Future of Our Planet

    For those of you who are interested, there’s a World Population counter at the bottom of the home page there that updates in real time. Kewl!

    Tony.

    20

  • #
    elsie

    Actually I thought one reason why Venus is so hot was the extreme density of its atmosphere. Offhand, I think it is about 90 times more than earth. The pressure at the surface is greater than the deepest ocean rifts here. The Russian probe which made it to the surface was crushed in seconds after taking a few brief pictures of rocks.

    10

  • #
    Louis Hissink

    Elsie #3

    That’s an interesting observation Elsie, but the problem is that at the deepest ocean depth the water is extremely cold, insofar that the energy from the sun cannot penetrate down there. So too with the 90 atmosphere Venusian atmosphere which is basically more dense that the earth’s ocean. So how to explain the extreme heat of the Venusian surface? Adiabatic compensation, or as some think, Venus is a young planet recently formed according to our ancestors. Oddly I recall reading that the night side of Venus radiates more heat than the dayside; this seems to support the youthful planet hypothesis, but a runaway greenhouse mechanism it isn’t, despite Carl Sagan introducing it to counter arguments he disagreed with at the time. James Hansen did his PhD on this topic, by the way, so we can see why Hansen is so concerned with CO2.

    10

    • #
      Truthseeker

      Louis, you are correct that the difference between the deep oceans of Earth and the deep atmosphere of Venus is the level of energy being received by each. After all Earth’s deep oceans receive very little if any energy in the form of light from the Sun and so the intense pressure is meaningless because there is no energetic reactions of photons hitting deep ocean molecules. With Venus, yes there is cloud cover, but there are still photons from the Sun penetrating to the low altitude atmosphere of Venus and with the high density (which is what air pressure represents) there are a lot of photon to air molecule interactions occurring, thereby generating the heat that we can detect in the Venus atmosphere. High air density means a high number of photon to air molecule collisions and a high number of air molecule to air molecule interactions to convey this energy as heat. It also means that there is more molecules to “retain” this energy which is why the night side of Venus does not cool as much as you would expect.

      I am happy to be corrected by someone with a higher level of understanding of physics that I have.

      10

  • #
    Peter Lang

    James Hansen reckons there is a strong greenhosue effect on Venus – so strong it boiled off Venus’s oceans. He reckons that could happen on Earth too, and we are at risk of boiling off our oceans.

    10

    • #
      Louis Hissink

      Yes, Hansen’s belief in this runaway greenhouse effect on Venus is driving his urge to reduce earthly carbon pollution.

      10

  • #
    michael hammer

    Consider what we mean by the term “green house gas”. It is simply a gas that absorbs somewhere in the thermal infra red range ie: between about 4 and 50 microns. Also remember the ability to emit equal the ability to absorb – absorptivity = emissivity. Thus a gas that does not absorb also cannot emit and a gas that does absorb at some wavelengths can only emit at those wavelengths.

    Now, what would happen if we had an atmosphere truly devoid of any green house gas (or more accurately devoid of any green house material since particulates can more or less act like green house gases). There would be no absorption of thermal infrared energy and no emission of thermal infra red either. In short the atmosphere could neither absorb nor emit thermal infra red. In such an (unlikely) scenario the surface would emit to space as a black body irrespective of how thick the atmosphere was and there would be no emission from the atmosphere at all. The surface would absorb energy from the sun and in turn emit energy to space, the temperature set by equilibrium between the two processes.

    But surely the atmosphere would heat up from contact with the surface and that would start convection wouldn’t it? Where would the energy from that convection go? Yes the bottom of the atmosphere would heat up from surface contact although the amount of heating would be far smaller than we are used to on Earth ( much of the heating comes from absorption, by the first few meters of the atmosphere, of thermal infrared radiated by the surface) , non the less that would start some convection. As that warmer air rose it would cool through expansion and that would establish a temperature gradient or lapse rate in the atmosphere. Such action could distribute energy around the globe through global atmospheric circulation (warm equator to cool poles) but it would not contribute to energy loss to space. Alternatively if energy distribution did not occur the convection would simply stop once a stable lapse rate had been established because the temperature gradient would be in equilibrium with the pressure gradient and there would be no net convective force.

    So, what changes when a green house gas is present? In that case, energy radiated from the surface at the green house gas wavelengths is absorbed by the atmosphere and replaced by radiation to space from the top of the atmospheric column. Put another way, the atmosphere becomes opaque at the GHG wavelengths. In principle a GHG dilute enough would not be totally opaque allowing some surface radiation to escape but in such a case the absorption line would be so narrow that the total energy involved would not be significant (absorption lines broaden as the concentration increases). Now energy loss to space will occur from both the surface at non GHG wavelengths and from high up in the atmosphere at GHG wavelengths.

    Well, so what? Total emission to space must equal total absorption from the sun for equilibrium. This emission is thermal emission which means it is determined by the temperature of the emitter and the temperature will adjust until emission equals absorption. Without any GHG’s the only temperature relevant to this equilibrium is the surface temperature but as the portion of the spectrum intercepted by green house absorption increases so the temperature at the top of the atmosphere becomes increasingly important. Once green house absorption intercepts the entire thermal infra red spectrum the temperature of the surface becomes irrelevant and it is temperature of the top of the atmosphere which establishes equilibrium. In this latter case, how does the surface temperature relate to the temperature at the top of the atmosphere? Ahh well, that is set by the thickness of the atmosphere and the lapse rate. In that case, the thicker the atmosphere, the greater the temperature difference between surface and top of atmosphere.

    So those who say the surface temperature on Venus is very high because of the very thick atmosphere are right but only and precisely because there is a strong green house effect. So strong in fact that almost all the energy loss is from the top of atmosphere with very little from the surface. If there was no green house effect, the thickness of the atmosphere would be irrelevant.

    So why is the green house effect so strong on Venus? There is an effect called pressure broadening. As the pressure of a GHG increases the absorption lines get broader. On Venus the partial pressure of CO2 is about 97 bar while on earth it is about 0.000390 bar. That’s a difference of more than 200,000 times which accounts for very significant broadening and explains why the green house effect is so much stronger on Venus than on Earth. Couple that with a far thicker atmosphere on Venus and the very high surface temperature is explained.

    In the case of Earth, the Nimbus data shows very clearly that well over half the total energy loss to space comes directly from the surface (despite what Keihl and Trenberth claim) whereas on Venus there is essentially no energy loss to space from the surface. However, lets consider a very crude speculation. Let’s assume that some catastrophic event changed the atmosphere of Earth so that the green house effect intercepted all thermal infra red wavelengths (before I go any further, this is not a reasonable possibility and certainly cannot even remotely come about through burning fossil fuels, its a thought experiment only). In that case the top of the atmosphere would set thermal equilibrium. Clearly the surface would become hotter and that would cause more evaporation and cloudiness would increase. More cloudiness would have to increase the planet’s albedo meaning that the energy absorbed from the sun would decrease. Earth’s albedo is currently 0.3 and that of Venus 0.7. I am going to make a wild guess and assume Earth’s albedo would rise to 0.5 (a reasonable value in my view). The planet would then absorb an average 170 watts/sqM from the sun. For the top of the atmosphere to emit 170 watts/sqM (remember since it absorbs at all wavelengths it can emit at all wavelengths) it would have to be at a temperature of 234 Kelvin (Stefan Boltzman law). Now the current temperature difference between the surface and the “top of the atmosphere” (actually I am using the tropopause temperature here which is the coldest point in the atmosphere) is 288K to 220K – a difference of 68K. Applying that to our 234K computed number yields a surface temperature of 234+68 = 302K or 29 centigrade.

    So, even such an extreme scenario matching the situation on Venus does not give the sort of massive temperature rises seen on Venus firstly because we are further from the sun and secondly the atmosphere on Venus is 100 times thicker than ours. Yes the green house effect has a massive impact on Venus but the situation on Earth is quite different.

    Please note, the above analysis is indicative only. There are secondary effects I am ignoring, non the less it gives a very easily understood idea of the way Earth would respond to a Venus type scenario and the result given is a reasonable ball park estimate.

    20

    • #

      In the case of Earth, the Nimbus data shows very clearly that well over half the total energy loss to space comes directly from the surface (despite what Keihl and Trenberth claim)

      I most certainly agree with you on this point. A quick glance at your first chart shows the large chunk going through the “atmospheric window” between wavelength 780 and 1250. I referred to this in #19.1 below. K&T appear to justify their figure by using cloud cover percentages in a not entirely convincing way.

      WRT Venus, your “100 times thicker” atmosphere would result in a similar surface pressure, and consequent surface temperature without much of a “greenhouse effect” (Ideal Gas Law). I’m still not entirely convinced , though at least I’m open to reasoned argument!

      10

  • #
    michael hammer

    A further point. It has been commented many times that the green house effect on earth makes it 32C warmer than it would otherwise be. This is calculated on the basis that Earth absorbs 240 watts/sq M and to radiate that amount of energy as a black body the temperature would be 255K. However it is actually at 287K hence 32K warming. Well the maths is correct but the analysis is wrong.

    Unless we change the laws of physics, which is a completely pointless exercise in this case, the only way we can have no green hosue effect is to have no water vapour in the atmosphere. However if we have no water vapour we have no clouds and clouds cause the bulk of Earth’s albedo. Without clouds the Earth would be absorbing about 320 watts/sqM not 240 watts/sq M and if it was radiating 320 watts/sqM as a black body the temperature would be 274K. Instead it is at 287K so the net effect of greenhouse gases is not 32C but 13C.

    So why do I raise this point? Two reasons, firstly it shows up the danger of considering one effect without also considering other effects of the same component. Secondly and more importantly it shows up just how significant clouds are. If we assume each doubling of CO2 increases temperature by about 1C (a commonly quoted figure), at 280 ppm CO2 there are about 10 doublings above line centre saturation so CO2 is contributing about 10C of warming. That means that everything else INCLUDING water vapour is contributing a net 3C. In short the green house warming from water vapour is almost completely balanced by the cooling impact of clouds. But we know that green house warming is logarithmic whereas cloud cooling is much closer to linear. That means an increase in evaporation leads to both a higher water vapour concentration and more cloud mass (since rain comes from clouds and more evaporation must be balanced by more precipitation). The incremental linearly increased cooling from more cloud mass will be significantly greater than the incremental logarithmically increased warming from more water vapour: ie; more evaporation would lead to incremental net cooling not warming. In short the feedback from increasing water vapour (constant humidity at a higher temperature) is negative not positive and that means the actual temperature rise from doubling CO2 will be less than 1C not 3-4C as claimed by the warmists.

    20

    • #

      Well said – that largely mirrors my own opinion. Some comparisons are between an Earth with “greenhouse effect”, and an Earth with no atmosphere, but persist in using the same overall albedo. It should be obvious that an Earth with no atmosphere would have no clouds, and would have no oceans either – they would never have formed in the first place. The overall albedo would be very different.

      WRT a “greenhouse effect” on Venus, there’s a little fly in the ointment, an “inconvenient truth”. The clouds on Venus are tens of km deep, and very little light penetrates to the surface. It’s been described as “perpetual twilight”, so where’s the driver for the “greenhouse effect”? It takes “two to tango” – both greenhouse gases and sufficient incoming radiation.

      10

    • #
      bananabender

      The idea that the Earth is heated by 33K by the atmosphere is total garbage. It is calculated by assuming the Earth is a flat disc that is evenly illuminated.

      The fact is the Earth receives 1340 W/m2 at the Equator and zero W/m2 at the poles. It also radiates heat from the non-illuminated side.

      20

  • #
    Kevin Moore

    I looked up my Websters dictionary to find the definition of “aficionado” – no such word in it – probably too old,so I had to find its meaning on the internet.

    For those of us who are not science-aficianados,will those who are please take George Orwells good advice so that we can better follow what you are talking about.

    Literary legend, George Orwell wrote an essay in 1946 called Politics and the English Language as something of a cure for the state of writing in publications of the day. PickTheBrain.com brings to light 5 rules from said essay that will bring out your writing from the pack.

    1. Never use a metaphor, simile, or other figure of speech which you are used to seeing in print.
    2. Never use a long word where a short one will do.
    3. If it is possible to cut a word out, always cut it out.
    4. Never use the passive where you can use the active.
    5. Never use a foreign phrase, a scientific word, or a jargon word if you can think of an everyday English equivalent.
    6. Break any of these rules sooner than say anything outright barbarous.

    10

    • #

      I’m sure I break many of those rules, but I do try to speak and write plainly and clearly (and think accordingly). I heartily agree though – “plainspeak” beats “newspeak” every time.

      10

    • #

      Kevin: aficionado in the Free Dictionary. I guess I could have used a cliche… ;-). Orwell was brilliant.

      10

    • #

      These rules look very much as if they could have been lifted from William Strunk Jr’s Elements of Style, 1919, a textbook used in Prof. Strunk’s classes at Cornell University. The book has since been revised and expanded to its present form by one of Strunk’s students and noted author, E.B. White.

      Perhaps Orwell was aware of the rules in Strunk’s “little book”, as it was called, or had even read it. Whether Orwell had read Strunk or not, the compilation of Orwell’s list illustrates some of the rules that good writings have in common.

      10

  • #
    Siliggy

    How much of the reason for Venus being hotter is the very basic shorter distance to the sun? As well as the greater radiant heat because of the shorter distance there would also be greater electrical interaction between it and the sun. Also it would get more frequent hits from solar storms. There would also be a greater modulation of the atmospheric pressure by the pressure of the solar wind.
    Also notice that Venus has no moon. How much of our surface energy is tidal? How much of the electrical interaction between the sun and the earth is altered each lunar cycle by the moon cutting the path of the solar and cosmic radiation?
    It is interesting to note that the chart above does not show radiation of longer wavelengths than 25uM yet it does show that the level at this left end is high. Our planet has life and life will alter what happens at these wavelengths. So will the highly conductive salt water of the oceans.
    Surely there is no comparison that will work well between the two planets.

    10

    • #
      Joe Lalonde

      Good! Your using your head besides being a hat rack! 🙂

      Most people would just spout what was on a computer screen with no thought of their own.

      10

  • #
    Gee Aye

    Sigh… centrist science. It is a joke but it is completely at odds with science.

    20

  • #
    Rereke Whakaaro

    Michael,

    I thank you for your post, but I must confess that I am confused, not over what you have written, but over the first graph.

    I notice that there are two scales on the horizontal axis of this graph: Wave number, which you say are the reciprocals of wavelengths, and Wavelength, measured in micrometers. I also notice that wave number is a linear scale, whereas wavelength is logarithmic.

    Since one is the reciprocal of the other (a simple arithmetic relationship), how can one be logarithmic and the other linear?

    Also, I remember from studying applied physics (a long time ago, now) that frequency was the reciprocal of wavelength. How is “wave number” different to frequency? I ask, because “wave number” is a term I have not come across before?

    10

    • #
      Siliggy

      Rereke The so called speed of light divided by wavelength gets you frequency.
      Play with this: http://www.photonics.byu.edu/fwnomograph.phtml

      The above chart uses wavenumber which is: One divided by wavelength equals wavenumber but that just leaves us radio techs puzzled because the head full of handy formulas and rules of thumb fail with wavenumber but work with frequency.
      I do wonder how they go about calculation of the power of the CO2 absorbtion. No doubt they have just derived a different set of formulas. We (Radio Electronics types) would find the 3dB points and note the frequency difference. Then the formula for power in Watts is Just KTB. K=Boltzmann’s constant(1.38 x 10e-23) T=Temperature B=Bandwith (as frequency).
      http://www.rfcafe.com/references/electrical/noise-power.htm

      10

  • #
    stephen richards

    The former is at the CO2 absorption line and the

    That is not simply the CO² absorption line. You are fundementally wrong there. That is the H²O absorption line particularly at the edge. CO² absorps in 2 very fine lines where H²O absorps in 2 very broad spectra in the same wavelengths as CO².

    10

  • #
    Nullius in Verba

    I’ve got a different thought-experiment for you. Suppose instead of making the atmosphere transparent to IR, you made it more opaque. Suppose you took something with the IR spectrum of water vapour and increased its density/opacity 20,000-fold. Wouldn’t that result in a far more intense greenhouse – intense enough for its effects to be more obvious?
    Or to put it another way, suppose sunlight shone down through a pool of water, was absorbed by the bottom, and re-radiated upwards. Only the bottom layer of water on top immediately absorbs/re-radiates half of it back down, and the same for every minimally opaque layer of water above it. What would happen? What would happen if you added more water? Why is that different from the greenhouse effect?

    Back to Venus – suppose it had an IR-transparent Nitrogen atmosphere at 90 bar at the surface, but still had those thick clouds at 80 km altitude. Suppose enough light got through to the surface with enough variation to drive convection. (Venus is also quite windy.) The clouds settle out at radiative equilibrium, whatever that might be. The adiabatic lapse rate is still around 10 C/km. Wouldn’t the surface still be hot?

    PS. I quite agree that Earth and Venus aren’t the same temperature at the same pressure, and that greenhouse gases play a role in determining the temperature of atmospheres. I’m more interested in clarity on precisely what that role is and how it works.

    10

    • #
      bananabender

      Infrared radiation is totally absorbed by the first millimetre of water.

      Venus has a very geologically active surface. The atmosphere is heated both by sunlight and volcanic activity.

      The density and gravity differences between Earth and Venus effect the adiabatic lapse rate.

      10

  • #
    bananabender

    No legitimate physics or chemistry textbook even acknowledges the existence of the Greenhouse Effect – it is unscientific and refuted by Bohr Theory. Chemistry spectroscopy textbooks openly ridicule the entire concept of “Greenhouse gases”.

    There is no Greenhouse Effect on Venus, Earth or anywhere else. Planetary atmospheres are heated by physical processes including convection, conduction and the evaporation and condensation of water.

    20

    • #
      Craig Thomas

      Honestly – do we still have to be subjected to this kind of nonsense?

      10

    • #
      CameronH

      I thought water vapor was a GHG?

      10

    • #
      Richard C (NZ)

      Planetary atmospheres are heated by physical processes including convection, conduction and the evaporation and condensation of water

      Yup, via ocean, land and sun. Which means (in the earth case) that the ocean/air and land/air interfaces are critical.

      But what effect do GHGs have at those interfaces? None, I posit.

      The ocean is the planet’s greatest heat sink; if GHGs don’t heat the ocean in bulk (they don’t), they don’t contribute to global warming.

      And if GHGs don’t promote evaporation (they don’t), they don’t contribute to an amplification of the (misnamed) greemhouse effect.

      CAGW game over.

      20

  • #
    Joe Lalonde

    Jo,

    Two things with Venus that makes it a very interesting planet to study.
    It is the only planet that rotates backwards.
    The other is that it takes 243 days to rotate on it’s axis.

    In order for an object to rotate backwards, it must be peeled off another much larger object. Like peeling an orange,you can rotate the orange and the peel will be mirrored but opposite. The only mass big enough in the area would be the sun. If the object grazed the sun, it would rotate backwards.
    Rotating so slowly, the sunny size MUST be vastly more hotter. But we have NEVER shot a satellite to this size of the planet. Only used instrumentation on the “backside” of the planet.

    The core of the sun and all the planets but the first two rotate within one day of each other. No matter the size or density composition. This tells us that the sun influences the other planets while holding them at a flat plane of the solar system using magnetic field interaction.

    So, Venus really is a bad example to use as a sister planet.

    10

  • #
    Joe V.

    Venus is on a slow spit roast, rotating on its axis only once in 243 Earth days.
    It goes round the Sun faster than that, taking just 225 Earth days. So you get all 4 seasons in just one day & then some – every day.

    Its atmosphere would seem to be more shaken, than stirred.

    Drawing parallels about greenhouse gas effects here, with planet Earth, seems a might fantastic.

    10

  • #

    Michael, where’s the water vapour absoption in the Nimbus satellite data in your first graph?e

    10

  • #
    Truthseeker

    The other thing about Venus is that it does not have a magnetic field. This may have all sorts of interesting effects that make it a tricky comparison. However, lets be clear that the original Venus/Earth comparison by Dr Huffman is specifically for comparable air pressures which in the case of Venus are fairly high up in the atmospheric profile. We have to be sure that we are comparing like to like.

    10

    • #
      Crakar24

      TS,

      I thought Earth and Venus were the only two “rocky” planets that had a magnetic field (molten core). Mars did have one but it is now dead and so its atmosphere is all but blown away.

      10

      • #
        Truthseeker

        Crakar24, I got the lack of magnetic field thing from Wikipedia. Happy to be corrected if you have a better source. I always had assumed it was gravity that caused an atmosphere to stay or not. Mars has a low gravity therefore it has a thin atmosphere. Venus has a high gravity and therefore a thick atmosphere. Again this is simplistic and maybe I need to update my knowledge in this area.

        10

        • #
          Crakar24

          TS,

          I stand corrected about the mag field thingy. Let me rephrase, Venus and Earth are the only rocky planets to a liquid core, Mars had one but not anymore and read somewhere that without a mag field (Mars) you lose your atmosphere through solar winds etc.

          Anyway what i find most contradictory in all this is that Venus is hot because of co2 re GHG theory and yet Venus shines so bright due to albedo effects that the Mayans knew of its existence only to be told albedo cools the planet.

          Too many theories, not enough answers (read facts).

          10

  • #
    Bryan

    There are various versions of the so called “greenhouse effect”.
    The one that claims a 33K effect for Earth is pure nonsense.

    Some claim that because CO2 and H2O are significant absorbers and transmitters in the infra red this proves a 33K effect.
    If you examine the logic behind this claim you will find it childishly silly.

    The top graph of the Earth as seen by the Nimbus satellite shows that around 15um CO2 is a strong absorber.
    The energy absorbed is soon distributed to mainly N2 by collision.
    The energy is then radiated preferentially by H2O vapour having longer available wavelengths and hence more probable transmissions as a result of collision induced radiation.
    What is the connection with the claimed 33K greenhouse effect – none !!!!

    The greenhouse theory has been debunked many times most recently by Joseph Postma.

    You should read the article below.
    It will answer all misconceptions about the “greenhouse theory” which is pure fiction.

    1. It is true that the radiative equilibrium temperature of the Earth is -18C.
    2. This figure is confirmed by Satellite measurement from space.
    3. It is NOT true that all the radiation emitted from the Earth surface.
    4. The emission is mostly from the cloud level.
    5. If we pick say around 5km as the average emission height to space we will be near enough.
    6. Now use thermodynamics expressed in the form of the lapse rate.
    7. An average moist lapse rate is -6.6K/km.
    8. The lapse rate has nothing to do with back radiation as even scienceofdoom admits.
    9. Work back down from -18C at 5km to the Earth surface and we get 5×6.6 = 33K.

    So now you have an Earth surface temperature of 15C without any “greenhouse effect”.

    http://www.ilovemycarbondioxide.com/pdf/Understanding_the_Atmosphere_Effect.pdf

    10

    • #

      Calculating an emission temperature and consequent “emission height” from IR emission to space is invalid. To do that there must be a single source with a single temperature. That first chart is all that’s needed to debunk the idea. Firstly, it’s not a smooth Planck curve (the dotted curves are), which is needed if the Stefan-Boltzmann law is to be applied to the total IR emission. That emission is defined by the area under the curve, which clearly has big chunks missing.

      Secondly, the emission comes not from one, but two sources, the GHGs and the surface, at different temperatures. The section between 780 and 1250 is the main “atmospheric window”, transparent to long-wave IR (except for the O3 band in the middle, so it’s really two windows). The line in this area tracks a little below the 300°K Planck curve, say 298°K or 25°C, consistent with the “tropical Pacific Ocean” source, and an emissivity close to unity.

      Since the calculated “emission temperature” is invalid, anything derived from it is also invalid.

      10

  • #

    Notice in that first chart the “big bite” for CO2 and the “smaller bite” for ozone (O3) mentioned by Michael. At wave number 1300, there’s a tiny downward spike – that’s methane (CH4), claimed to have 20 (or 25 or 30, choose any one) times the “global warming potential” of CO2. Make up your own minds on this one.

    10

  • #
    michael hammer

    Banabender at 8:12 Chemistry spectroscopy textbooks openly ridicule the entire concept of “Greenhouse gases”.
    Really! I would need some evidence for that because as someone who has conducted research for a spectrscopy company for the last 30 years I don’t think you are right. The only requirement for a green house gas is that it absorbs in at least part of the spectrum egtween 4 and 50 microns. Are you really saying CO2 does not absorb in this region of the spectrum? I suggest you go away and check your facts

    Mike Borgelt at 8.34. Water vapour absorbs prinicpally in the region above 1250 wavenumbers and below 600 wavenumbers. You can see how the energy density to space is lower in these regions. The region below 600 wave numbers (ie: above 16 microns) is particularly interesting. It looks like the emission temperature is neither the surface temnperature nor the tropause temperature which would contradict my comment about emission from the surface being replaced by emission from the top of the green hosue gas column (which for water vapour is certainly the tropopause since the stratospchere is deficient in water vapour). However nots how jagged the plot is in this region. In fact water vapour has many closely spaced lines in this region – the analogy to a picket fence is often used. The spacing I suspect is to close for the interferometer abord the Nimbus satellite to resolve so it smears it into a very noisy average value.

    A number of you have raised other differences with Venus. Retrograde rotation, no magnetic field, closer to the sun etc. Yes of course there are many differences but these issues are too braod for this essay, I was focussing only on the green house effect.

    Nullis in Verbena. You raise the analogy of the green house effect of liquid water. Think about what I said, radiation from the solid surface to space is blocked and instead radiation to space occurs from the top of the green house gas column. In your analogy the “top of the green house liquid column” is the surface of the liquid water and indeed infra red radiation from a body of water all emanates from the top millimeter or fraction thereof. Any radiation from lower down is absorbed by the water above it. Its in effect the same thing.

    10

  • #
    michael hammer

    Mostly harmless at 8:29. You say it takes 2 to tango, both a green hosue effect AND energy release at the surface. I agree little light penetrates down to the surface but little is not none. Also there will be some energy released by the core of the planet. The only way the surface can lose energy is to space (the core of the planet is hotter than the surface) and since it cannot radiate this energy, because of green house effect, loss can only be by conduction/convection which will immediately establish a lapse rate and a temperature differential between surface and top of atmosphere.

    10

    • #
      Crakar24

      So Michael,

      Based on your previous comments and with very little light penetrating the surface of Venus one could safely assume Venus has a very high albedo, a very high albedo would cause a cooling….yes. Over time Venus should begin to revert to a climate more like on Earth (ignoring distance to sun, pressure etc) if so then why do some claim Earth is heading towards a Venusian climate (would this give teh game away?).

      Cheers

      10

  • #
    michael hammer

    Rereke Whakaaro at 7:29 I don’t understand why you say wavelength is logarithmic. A graph may plot wavelength on a logarithmic axis or on a linear axis but that does not mean wavelength is logarithmic. I frequenctly use wavelength plotted on a linear scale. Wave number is indeed linearly proportional to frequency. Infra red chemissts use wave number instead of frequency becuaes the units are more convenient. Frequency is is very large number. Why wave number instead of wavelength? Because wave number is directly proportional to photon energy and that is useful for chemists.

    10

    • #
      MaryFJohnston

      Hi Michael Hammer 🙂

      Didn’t read the original comment that you were referring to when you wrote “”but that does not mean wavelength is logarithmic”. Must go and read RWs original next.

      There is only one way I can see to connect “wavelength” with “logarithmic” in the context of “CO2” and “Greenhouse Effect”.

      CO2 can only absorb Infra Red at two or three spectral locations and it is possible to know or quantify the Amount of IR energy available for absorption at each frequency or wavelength.

      If we theoretically add more CO2 to the air mix then we absorb more IR and move closer to having absorbed ALL of the IR available at that wavelength.

      To absorb or soak up one additional unit of IR we must add ”Logarithmically” increasing amounts of CO2 to achieve this.

      People refer to the “law of diminishing returns” because less and less IR is absorbed for the equal increments of extra CO2 added.

      Another way of putting it is that the extra CO2 has less and less effect.

      Another way of saying it is that extra CO2 will not cause runaway Global Warming.

      Have read a lot of your other posts above but must go back and finish them off . very dense material, had to take a break.

      10

  • #
    michael hammer

    Nullis in Verbena, I missed the second part of your comment. What if the atmosphere was nitrogen with thick clouds. wouldn’t the effect be the same as for green house effect? Well no it would not be. In the hypothetical situation you suggest the surface would radiate straight to the clouds. These would absorb some energy and reflect some back. In fact water droplets which is what clouds are, are good IR absorbers and thus also good IR radiators so we can consider the clouds to be a black body. If we ignor transmission through the clouds for the moment (imagine they are thin – and yes I do realise they are not thin at all) what will happen is as follows

    The clouds are in radiative equilbrium and thus will be at a temperature at which their emission to space balances the energy absorbed by the planet. Lets pick a number just for example, say 260K. The surface now radiates not to space at 4K but to the clouds which are at 260K. The difference in energy radiated from the surface and the energy radiated back to the surface from the clouds must equal the energy absorbed by the surface. Thus energy absorbed by surface = 5.67e-8*(Tsurface^4-260^4). From that if we know the energy absorbed by the surface one can calculate T surface. Lapse rate and atmospheric thickness wuld not come into it (unless the convection also carried some energy so that the radiate component was slightly less but that I ssupect would be a second order effect).

    10

  • #
    michael hammer

    Mostly harmless at 9:30. When people talk about methane having a much greater potential warming effect what they really mean is that the same concentration of methane at its characteristic absorption wavelength will be far more opaque than the same concentration of CO2 at 14.8 microns. In effect its like saying ink has far more colouring power than does say beetroot juice. The total impact of methane still depends on its concentration which is far far lower than the CO2 concentration.

    10

    • #

      It would help us all if you’d reply in thread. In fact that’s not “what they mean”. What they mean is that CO2 is already saturated in its main absorption band centred around 15 microns, and methane is not, at its main band (more of a sharp “spike” than CO2) around 7.8. Any further absorption from CO2 occurs by broadening the CO2 band “shoulders”, and not deepening the”trough”. A 1% increase in methane concentration will therefore have a proportionally greater effect on its absorption than would a 1% increase in CO2 on its total absorption.

      Putting it another way, it would take a higher %age increase in CO2 to create a similar effect to an increase in methane.

      10

  • #

    This is a very lazy treatment, typical of the consensus. The “greenhouse effect” promulgated by the “consensus” is not merely the absorption (and emission) of infrared radiation by atmospheric gases like carbon dioxide (CO2). It is the absorption of infrared from the surface of the planet, with strong downward re-emission (backradiation). With that understood, to say that carbon dioxide absorbs infrared and is therefore a greenhouse gas (as so many defenders of the consensus do) is incompetent. As I pointed out to Judith Curry on her blog way back in December 2010, the issue is whether the infrared absorption is of incident solar infrared, and directly heats the atmosphere, or of radiation from a previously heated surface; but consensus defenders cannot seem to get that message. Make no mistake, infrared absorption by atmospheric gases heats the atmosphere; that is why defenders of the consensus are forced to compare the radiation observed through the atmosphere with what it would be without an atmosphere, rather than with what it would be without CO2. The point is, what is the effect of increasing CO2 in the atmosphere, and my simple comparison of Venus (with 96.5% CO2) and Earth (with 0.04%) shows the effect is certainly NOT to increase the temperature, at all. Mr. Hammer and others, like Jo Nova and every other believer in the greenhouse effect, need to clear their minds and read my analysis without prejudgment, for they are ignoring the clear lessons to be learned from my Venus/Earth comparison.

    As for the evidence of observed radiation “bites”, I can make one or two observations. Even before I made the Venus/Earth comparison, I deduced from simple principles that the real effect of increased atmospheric CO2 is to increase the efficiency, hence the speed, of heat transfer within the atmosphere. This, I say, is why the dark side of Venus (with 96.5% CO2) is as hot as the sunlit side. Simply put, heat transfer includes the processes of convection, conduction, and radiation, and adding more IR-active gas (CO2, H2O, CH4, etc) just shifts the balance of these three processes more toward the radiation portion–the fastest portion, thus the increase in speed of heat transfer. Such an increase does not increase the temperature at any given pressure level, within the range of Earth tropospheric pressures at least, it merely quickens the attainment of the equilibrium vertical temperature distribution governed by the hydrostatic condition, as long known in the Standard Atmosphere. But of course, it also quickly funnels the infrared radiation into the channels provided by CO2 and other more IR-active gases. So I believe what is being seen in the radiation graphs is just that funneling effect into the more active wavelength regions, and it has nothing to do with the AMOUNT of atmospheric heating. In other words, the graphs do NOT show “radiative forcing” of an atmospheric temperature change, as the consensus scientists believe. I do not have all the answers, such as a detailed re-analysis of the radiation graphs, but Mr. Hammer has not shown, within the context of his own understanding of the consensus theory, why the only difference in atmospheric temperatures between Venus and Earth, over the range of Earth tropospheric pressures, is PRECISELY explained by the difference in their distances from the Sun, nothing else. There is no room–NO ROOM–for either the consensus “greenhouse effect”, nor for an albedo effect (because as I have shown, the atmosphere is warmed by the portion of incident solar radiation that is directly absorbed by the atmosphere–the SAME, and infrared, portion, for both Venus and Earth).

    Mr. Hammer, like so many others, clearly has been convinced of the “greenhouse effect” for a long time, and feels comfortable defending it without bothering to consider the alternative so strongly–I say definitively–indicated by my Venus/Earth comparison. I made it as simple as possible, but in the end he and those many others must re-educate themselves (just as I have educated myself), by actually confronting the ideas I have brought forward, rather than ignoring my plainly-stated insights and dismissing them without thought. I am not an electrical engineer with 30 years’ spectroscopy experience (and thus have no dogmatic loyalty to current interpretations of the radiation graphs); I am a physicist, with 40 years of experience, and I am only impressed by the truth, when I find it.

    50

    • #
      michael hammer

      Hmm; I think I have already answered your comments regarding Venus in my earlier post (muber 9 or 10 for some reason I can’t see the numbers on my computer).

      You comment that the green house effect is not just about absorption of infra red energy but also claims emission of energy at these wavelengths in all directions including downwards. Yes indeed that happens and I went into that at some considerable length in my previous article that Joanne was kind enough to post. There is no point in repeating myself. For those interested please refer to my previous article.

      With regard to your explanation of how the “bites” in the spectrum come about, you seem to be suggesting “is just that funneling effect into the more active wavelength regions”. What are the more active regions and what exactly brings them about? The science established for nearly 100 years (long before global warming was remotely an issue) found that thermal emission is determined by the temperature of the emitter and the emissity of the surface. Are you saying this theory is wrong and if so, on what basis? I don’t see any evidence to back up such an assertion. The level of the emission at each wavelength gives a measure of the temperature of the emitter (assuming we know the emissivity) and as the graph shows the emission at 700 wave number is coming from an object at around 215K while at 1100 wave numbers the emitting object is at about 300K. Corresponding to the tropopause and the surface respectively. How do you explain all this?

      I am most certainly not a believer in CAGW, in fact I am very strongly sceptical of that theory but the basis of my scepticism is soundly based on well established science.

      13

  • #
    cohenite

    For purposes of AGW how Venus became the way it did is crucial if Earth is to avoid the Venus ‘fate’. DeWitt Payne explained the creation of the current Venus thus:

    That’s the runaway part. You start with a planet with liquid water that’s a little too close to the sun. Or maybe it’s ok if the faint sun hypothesis was correct but then TSI increased. Anyway, for whatever reason, the temperature starts going up, more water vapor, higher temperature until eventually the oceans boil away and there is no more liquid water. Now the atmosphere is mostly water. Photolytic decomposition of water to hydrogen and oxygen proceeds and because of the higher temperature and lower gravity, the hydrogen bleeds away to space because the upper tail of the Boltzmann distribution exceeds the escape velocity. Once the oxygen concentration gets high enough, any organic carbon is converted to CO2. With no water condensing, there’s no weathering mechanism to return CO2 to the crust so any volcanic emission stays in the atmosphere. Eventually the surface gets hot enough to cook CO2 out of carbonate rocks near the surface and the entire planetary inventory of carbon exists as CO2 in the atmosphere.

    Current TSI at the Earth’s orbit isn’t high enough to cause a runaway even at several thousand ppmv CO2 or it would have by now, like during the PETM for example. How much higher it would need to be is controversial. IIRC, some think as little as 5%. But that’s many million years away if the sun stays on the main sequence.

    So nothing to worry about even if Hansen is right.

    10

    • #
      Dagfinn

      Also, Hansen says the “Venus syndrome” is not going to happen until all the ice on earth is melted, and that will take 1000 years or so. That time scale is so far beyond our ability to predict any thing at all that there is no point even considering it. Technology and society (if any) at that point is simply totally and absolutely unknown.

      10

  • #
    Kevin Moore

    One reason that we are able to survive on this Earth is because of the angle of the planet in relation to the Sun and not how close we are to the Sun.

    If the Earth were not tilted at all the equatorial regions would become unbearably hot and the Poles would be a lot colder.

    The tilt angle of the Earth ensures that the Suns energy is spread over most parts of the Earths surface throughout the year – enabling climate change or the four seasons.

    So how does the Earth compare with Venus?

    10

  • #
    Tel

    Without any atmosphere, Earth’s emission pattern as seen from space would look like one of these dotted lines.

    No.

    No it would not. The Earth without any atmosphere would not be a black body, not would it be a uniform surface temperature. Without any atmosphere the variation in surface temperature over the Earth would be far greater than the current situation, we would have a very hot equator during the day, and very cold polar regions.

    One of the primary effects of the atmosphere is to carry heat away from the equator toward the poles.

    The Earth at sea level is 15 degrees C? Really? I live pretty close to sea level and I’ve experienced betwen single-digit below zero temperatures up to a bot over 40 degrees C. What’s more I note that the North pole generally has ice at sea level, while the beaches at Bali never have any ice. Thus the thin green line of the graph doesn’t come close to describing the Earth, it’s not even a plausible radiation model.

    10

    • #

      The Earth without any atmosphere would not be a black body, not would it be a uniform surface temperature

      The first part of that sentence is a rather bold assertion. What evidence do you have to back it up? The Earth with or without an atmosphere behaves as a grey-body, that is a black-body with emissivity less than one, though in the case of Earth it’s very close to unity.

      Michael’s statement that “Without any atmosphere, Earth’s emission pattern as seen from space would look like one of these dotted lines” is correct, if emission was integrated over the surface, which is what’s implied.

      10

  • #
    Tel

    Calculating an emission temperature and consequent “emission height” from IR emission to space is invalid. To do that there must be a single source with a single temperature.

    Exactly, and more than that, attempting to draw conclusions about changes to one variable is even less valid when you ignore that the distribution of temperatures almost certainly will change. For example, it is well known that during past warm/cold cycles on Earth, the poles change more than the equator, so the temperature distribution changes and thus the entire shape of the spectral pattern changes.

    On Venus we only have a small number of samples, but the convection currents in the atmosphere are completely different anyhow.

    10

  • #
    Bryan

    It would be wrong to jump to conclusions about Venus and then apply them to the Earth.
    Much is unknown about Venus.

    For instance it emits much more energy than it absorbs from Sun.
    This indicated that there must be significantly more geothermal energy on Venus than Earth.
    There is also significantly more atmospheric electrical activity(lightning).

    Its reasonable to suggest that the surface light on Venus has an atmospheric rather than a solar origin

    http://www.bibliotecapleyades.net/tierra_hueca/esp_tierra_hueca_8e.htm

    10

  • #
    Nullius in Verba

    “You raise the analogy of the green house effect of liquid water. Think about what I said, radiation from the solid surface to space is blocked and instead radiation to space occurs from the top of the green house gas column. In your analogy the “top of the green house liquid column” is the surface of the liquid water and indeed infra red radiation from a body of water all emanates from the top millimeter or fraction thereof. Any radiation from lower down is absorbed by the water above it. It’s in effect the same thing.”

    So does it result in super-intense greenhouse warming or not?

    “Any radiation from lower down is absorbed by the water above it.” And immediately re-radiated in all directions, yes?


    “The difference in energy radiated from the surface and the energy radiated back to the surface from the clouds must equal the energy absorbed by the surface.” Only if the only means of transporting energy is radiation.

    “Lapse rate and atmospheric thickness would not come into it (unless the convection also carried some energy so that the radiated component was slightly less but that I suspect would be a second order effect).” Why would the lapse rate not come into it? If gas convects vertically, it is compressed or expanded and its temperature changes. If it is compressed 90 bar by being convected from the clouds down to the surface, it’s temperature will rise by 800 C. If it happens, that’s not second order. So is the answer that it doesn’t happen? Even though there are large differences between day and night sides, convection doesn’t occur?

    10

  • #
    R.M.B.

    There may be a greenhouse gas effect on Venus but there is none here because the ocean won’t allow it. The ocean has surface tension which is effectively a protective film and heat cannot pass through it. The molecules of co2 in the atmosphere have nowhere near the temperature needed to reduce the surface tension so that heat can pass. This is the explanation for Trenberth’s “missing heat”. If you find this hard to believe get a bucket of water and try heating the water with a heat gun. Remember to fill the bucket to overflowing.Agw is a nonsense and wind farms are chocolate teapots.

    10

  • #
    chris y

    The dry adiabatic lapse rate is dominated by the ratio of the gravitational constant to the specific heat of the atmosphere. You need an atmosphere, a gravitational field and a source of heat to have a lapse rate. No greenhouse gas property is required to create this lapse rate.

    The moist adiabatic lapse rate is dominated by the ratio of the gravitational constant to the specific heat of the atmosphere, modified by including the latent heat of condensation of water vapor, i.e. a phase transition. No greenhouse gas property is required to create this lapse rate.

    The greenhouse effect should manifest as an increase in the lapse rate. At the same pressure, increasing CO2 concentrations from 390 ppm (Earth) to 965,000 ppm (Venus), should result in a massive increase in the measured lapse rate.

    Your graph clearly shows that this canonical greenhouse fingerprint is not at all obvious.
    Even at 965,000 ppm CO2, the greenhouse warming is well disguised among the myriad other forcings and feedbacks in the atmosphere.

    10

  • #

    Any post that makes me think and ask questions is a positive one and very welcome. So I thank Michael Hammer for this post.

    Regarding the hypothetical “what would happen if the earths atmosphere consisted of only non-GH gasses?” i.e. only nitrogen, oxygen and argon.

    Take a point on the planet at 0Deg longitude and 0Deg lattitude; the equator.
    As this point rotates into the view of the sun (6am) the ground starts to warm up and radiate directly to space. As you state, some of that warmth would be conducted by the air in contact with the ground. As the minutes tick away and the temperature rises, the warming rising air would be replaced by cool air, not so much descending from higher up, but from the cooler west i.e. behind the rotation direction of the planet, essentially giving us gentle westerlies.

    As we reach noon, the hottest part of the day, more and more of the atmosphere conducts this warmth and rises up, causing the higher altitudes to also warm up. Yes the rising air would cool from expansion, but the total energy content would not change, only the temperature would change.

    As we reach late afternoon and a setting sun at 6pm, the above process would slow down (and the wind direction would change to easterlies).
    After sunset, the ground would cool very quickly as it always does. The air in immediate contact with the ground would also cool BUT COOL AIR DOES NOT RISE.
    Now we’re in a dillemma. We have a cool lower atmosphere which doesn’t rise, and a warmer parcel of air above it which doesn’t fall (a temperature inversion). Nor does the warmer parcel of air “radiate” it’s energy away. With the exception of minute amounts of energy transferred via conduction by adjacent molecules, this warmer parcel of air retains almost all of it’s warmth thoroughout the night.

    By 6am the next morning, we start the whole process again BUT THIS TIME WE ALREADY HAVE A WARMER ATMOSPHERE THAN THE PREVIOUS DAY.
    Now continue this process day after day, year after year and what we have is an accumulation of warmth (energy) with no means of shedding this warmth directly to space (due to the air molecules NOT being GHGs)

    My question is, what physical process(es) would inhibit this non-GHG atmosphere (at all lattitudes) from attaining a temperature equivalent to that of the surface at noon at the equator?

    10

    • #

      Any post that makes me think and ask questions is a positive one and very welcome. So I thank Michael Hammer for this post.

      I second that.

      I’ve thought along the lines of your analysis for some time, and agree – it seems the only logical conclusion is that GHGs actually cool the planet, and not heat it. I’ve seen similar analyses before, in one case by an atmospheric scientist, but can’t find a link to the article.

      Your description of a GHG-free atmosphere consisting only of nitrogen, oxygen and argon is however not quite accurate. Oxygen absorbs weakly, and is usually lumped in with ozone in absorption statistics. Nitrogen absorbs very weakly, despite almost universal statements to the contrary, but bear in mind there’s a lot of it. However, such weak absorption/emission wouldn’t affect your scenario much.

      10

    • #
      Julian Braggins

      “My question is, what physical process(es) would inhibit this non-GHG atmosphere (at all lattitudes) from attaining a temperature equivalent to that of the surface at noon at the equator?”

      Observations in the plumbing world may answer that question. In the old gravity water radiator systems it was not always possible to have a strict rise for every run throughout the system for the hot side, but the plumbers found from experience that as long as the cooling return column had sufficient capacity it still worked just as well. In other words the cooling effect is greater than the warming effect.
      In the hypothetical atmosphere the cooling night-side air at ground level would spread to less dense warm areas and draw down the warm air to be cooled in turn. (and the lapse rate would still operate 😉 )

      10

      • #

        @Julian

        Thanks for the reply. Interesting about plumbers and gravity water radiator systems. I’ve often said, if you wish to understand how our climate works, ask an evaporative air conditioner expert.

        I don’t know much about gravity water radiator systems, but accepting that what you say is true, the cooling cycle during night time would still be slower than the warming cycle during daytime leaving us with a surplus warmth at the next dawn.
        This will still lead to an accumulation of warmth (energy) in the atmosphere.

        So the process may take longer (though I didn’t specify a time frame) but the result will still be the same I would think.

        10

        • #
          Julian Braggins

          Baa Humbug,
          Thinking further on this subject while trying to get to sleep last night I realised that the polar circuit would be even stronger. One page on a NASA website suggested that the circulation on Neptune (2000Kph winds) might be driven by temperature differences between the equator and poles. It does have a phase change gas for clouds, methane, at 3% so it wouldn’t quite fit the bill of GHG free atmosphere, anyway, food for thought.

          10

          • #

            Yes so much food for thought my head is bloated.

            Must remember how far neptune is from the Sun.

            Conduction is a slow process. High winds may be par for the course initially. But eventually, unless the system can lose energy via radiation as fast as it can receive it from the Sun, energy must accumulate.

            Enough accumulation will end up (given enough time) with an atmosphere with uniform temperature throughout. No winds.

            10

  • #
    J.H.

    We need more satellites sent to Venus…. Science is observation, not speculation…;-)

    10

  • #
    Jerry Hudson

    Many thanks for a thought-provoking blog.

    My comments below are mainly questions, since I’ve not studied atmospheric physics, other than browsing books.

    I’m used to seeing a plot of z (height, Km) vs. T (Kelvin), such as may be found for Venus at:

    http://www.imcce.fr/vt2004/en/fiches/fiche_n13_eng.html

    The slope of their graph between 10 and 50 Km is about 7.3 K/Km, which is somewhere between Earth’s wet and dry adiabatic lapse rates. Not sure where the 1000 mbar height is on their plot, but I thought it was supposed to be around 55 Km. At that height, temperatures would be maybe room temp., and perhaps you could attribute that to a greenhouse effect?

    Also, I thought the dry air lapse rate only depended upon g (gravitational acceleration) and Cp (heat capacity at const. press.), so that indicates there should be a lapse rate in dry nitrogen. Wonder whether there is any experimental data on that?

    I would argue that if Earth had a dry nitrogen atmosphere, convection still might occur – I think there would be more molecules bouncing off the surface carrying heat energy than the surface could radiate photons. Any comment?

    Again, thanks for a nice presentation.

    – Jerry

    10

    • #
      Bryan

      Jerry Hudson says

      …..”Also, I thought the dry air lapse rate only depended upon g (gravitational acceleration) and Cp (heat capacity at const. press.), so that indicates there should be a lapse rate in dry nitrogen. Wonder whether there is any experimental data on that?

      I would argue that if Earth had a dry nitrogen atmosphere, convection still might occur – I think there would be more molecules bouncing off the surface carrying heat energy than the surface could radiate photons. Any comment?”…..

      Good comment!

      The free near surface atmosphere in a gravitational field is invariably adiabatic.
      But also it is heated from below and cooled at the top.
      That is all that is required to set up an adiabatic lapse rate.
      Even Convection is not required, diffusion(conductive collision transfer) on its own can fulfill the temperature gradient requirement.

      N2 and O2 have a much smaller radiant contribution than say CO2 but it is not zero.
      So a DALR would be set up with a higher average emission altitude.

      An even more interesting question is what is the natural atmosphere in a gravitational field without an imposed temperature gradient?

      The orthodox physics position is that it should be isothermal.
      This view is supported by the heavyweights Clausius and Boltzmann.

      However Loschmidt, Laplace and Lagrange argued that it would be a temperature gradient with higher temperatures near the Earth surface and decreasing with altitude.

      Unfortunately no experiment has been carried out to resolve the dispute.

      10

  • #

    I am in fact writing on a Venus-article myself, and the dealines are:

    Solar radiation is supposed to reach surface of Venus with around 750 W/m2.
    This is around 90% of the intensity that reaches the surface of Earth.
    One problem though: No one has ever measured any temperature rise durin a Venus day…

    So Basically: Untill it is actualy measured that the Sun can significantly make the Venus surface warmer at day than night, we have to conclude that Solar energy hardly reaches the Venus surface at all. Only a dim twilight with hardly any energy reaches Venus Surface.

    In fact, day and night temperatures are the same up to around 60-70 km altitude.
    This suggests, that mostly all Solar energy is reflected back from altitudes very high in the Venus atmosphere.

    Such a total blocking of Soar irratiation has to be connected with aerosols due to sulphuric and water. Deeply frozen CO2 might even help throwing Sunlight back.

    Still, the temperatures going down in Venus atmosphere is significantly hotter the deeper you go. Hottest at the very surface where no winds can explain transfer of heat, cause ther are no winds. You have to move up 6 – 10 km just to find a mild breeze…

    Still Venus has 460 degrees Celsius on the surface, just 20 degrees more and the surface would glow dark redísh !

    Why is it hotter on the surace when Sunligh over 58 Earth-days is not strong enough to change the temperature?

    Venus is extreeeeeeemlt different from the Earth, not only when it come to the atmosphere.

    Also the interior of Venus is “something else”.

    Fact: Venus surface on almost the whole planet is young. Very young, some say 100 mio years, some say up to 500 mio years. but its young, and hardly even stable today.

    On many places of Venus we have huge round kilometer-high formations with diamter of 1-300 km “pancakes” that appear to be originally just holes where lava just came up in volumes we dont know from the Earth. Its a kind of volcano without mountains. but “normal” volcanoes Venus has too and they count hundreds of thousands, or millions.

    So, to rule out that heat is indeed coming from benieth, and trapped inside the totaly bocke Venus atmosphere up to high altitude would be not well supported. Volcanic heat trapped over millions of years can make Venus atmosphere very hot. At least, this is a likely scenario.

    There are in fact a new theorym that venus surface is about to melt down again, and that Venus actualy has a cycle of around 500 mio years for each time the surface is renewed.

    And back to subject: Of course one should not compare Venus temperature at 90 atm with Earth temperature a 1 atm. I tink most here can agree on this.

    A CO2 – role under a completely aerosol closed atmosphere is not existing. Period .

    Any CO2 effect has to be measure in the top layer where sunlight can penetrate.

    K.R. Frank Lansner

    10

    • #
      cohenite

      In respect of the age of Venus and its volcanic and tectonic activity you may be interested in this:

      http://www.cosmosmagazine.com/node/1658

      O’Neill’s thesis is that surface conditions caused the Venus atmosphere which was then maintained by greenhouse; I hope he is wrong about the first, otherwise it makes SETI a bit of a waste.

      10

  • #
  • #

    Michael,

    My point was that you label the 15 micron absorption as CO2. This is wrong Water vapour overlaps this and there is a lot more of it.
    This is less than helpful to your argument.

    10

  • #
    CameronH

    Michael, You say “Note the big bite out of the spectrum at around 660 wavenumbers and the smaller bite at around 1000 wavenumbers. The former is at the CO2 absorption line and the latter at the Ozone (O3) absorption line. Those two bites represent energy that is not being radiated to space that would be if there was no atmosphere”. Would this be only partially correct. As the IR is absorbed by the CO2 molecule it would temporarily raise the temperature, and therefore the energy, of the molecule before it was again re emitted. I would imagine that a significant proportion of the energy would not be re-emitted but past to other atmospheric molecules such as O2 and N2 through molecular collisions. These molecules would then emit this radiation at a different frequency. This would mean that the radiation is still escaping but not a the frequency that CO2 absorbs and emits. Am I correct in this?

    10

  • #
    1DandyTroll

    less the greenhouse. So, essentially, the question becomes, how can you have an effect of a physical t’ing without the actual t’ing without that physical t’ing affecting everyt’ing? :p

    10

  • #
    bananabender

    The explanation for the difference between Mars and Venus is the Ideal Gas Law Pv=nRT.

    Sceptics shouldn’t waste another minute arguing about a non-existent Greenhouse Effect.

    30

  • #
    kuhnkat

    Mr. Hammer,

    GIGO.

    10

  • #

    Truthseeker 4.1 see if correct grammar is more important, than real science: 1] Venus is hotter, but not for reasons presented. 2] earth and Venus travel trough the same / very cold space… VERY DIFFERENT SPEED. The earth, much bigger orbit – 365 days. Venus, smaller orbit – even slower travel.

    Experiment: on very cold night; you outside in a Ute, travels 3kmh. On the other Ute is your friend Memoryvault at the back – 100kmh – monitor how deep the coldness will penetrate in each of you… BIG DIFFERENCE. Venus is very slow moving, spinning in opposite direction – no wander she represents woman…? In a month, will be 7billion people on the planet – but only from me you will get what counts. Can you say it on English: Stefan say, the most important is, the speed.

    Talking of coldness penetration; without taking in consideration of the velocity the earth’s / Venus orbits = is custom made for confusion. Do you know: why the boss for climate is not an English Grammar teacher? 2: how do you say it on English: the day on Venus is 4 months long. If the sunlight on earth was 4 months long – the sunny side would have being much hotter here too. Where is night on Venus, before morning is too cold for anything to survive – they will not tell you that. CO2 intercepts more heat than oxygen + nitrogen; but intercepts much more coldness also. THOSE TWO FACTORS CANCEL EACH OTHER. If you read on my website, how the laws of physics function – no need to go to Venus, Arctic, Antarctic; you will know from home: what is correct / wrong… no need to go to 5BC, or 2100AD you will know now. I doubt if you have stomach for the truth. 12y old children can understand what is on my website – because I present real science on simple way. Reading 3-4 sentences of complex issues; than blaming somebody for you not understanding… Give yourself second chance: http://globalwarmingdenier.wordpress.com If you read, you will lose the burden of your ”truth phobia” trust me

    10

    • #
      Truthseeker

      Stefan – the communication is getting better but could be improved. Your analogy with the ute is clear but misleading in this case. Both Earth and Venus are in a vacuum and so the speed that they travel is irrelevant. Answer me this question, if I was in space going 3kph and then accelerated to 100kph would my external temperature be any different? The ute at night analogy is referring to a “wind chill” factor and I believe that a vacuum means that there is no wind to have the same effect.

      Also, I am not sure of what you mean by “intercepts much more coldness also”. Cold is a relative term and is perceived as a negative difference from the heat of the object you are comparing it to. That is, it is a lack of heat relative to another object. How does something intercept a lack of something?

      I like to think I do not have a truth phobia, however allow me to make myself exceptionally clear as it is clearly required. I am not saying that you are wrong in a scientific sense. I am only saying that you are not communicating your ideas effectively. I can understand that if you have got a lot of negative comments you can feel that everyone is against you, but I am not. I have been to your website, a number of times and I think I get some of the ideas but they are communicated so badly I cannot be sure.

      10

      • #
        memoryvault

        Careful Truthseeker

        As a professional engineering technical writer I offered to help Stefan with his communications (gratis) on the last thread.

        As a result apparently now I am a cow-hating vegetarian hardcore green/pretend sceptic saboteur who doesn’t understand the significance of O4,

        and Stefan is the Pope of Rome (comment #71).

        .
        PS: – I thought traveling in the back of utes was illegal these days.

        10

        • #
          Truthseeker

          Memoryvault, I am not taking Stefan’s comments personally. I can see that he has so much passion about his theories and how correct they are that not seeing them as self-evident like he does is clearly the reader’s fault.

          As for travelling in the back of utes being illegal – only on public roads. On your own rural property, you can do what you like.

          10

  • #

    mostly harmless 6.1 would you explain on proper English to the Truthseeker: photons don’t carry any heat. Photons is light, so you can see when bounces from something, so you don’t bump into it. Photons are what the headlights on your car produce; or your torch. Tell him that: radiation produces the heat, UV, Infra-red. Better say, some colours radiation. Please tell him that: if he read what is on my website, he would have already known all that, and much more. Criticising without reading is not very bright. His ”truth-phobia”should be studied.

    10

    • #
      Truthseeker

      Stefan – like I said in my earlier post, I am happy to be corrected by someone with a better understanding of physics that I have. If it is not photon interactions causing the heat in the atmosphere then that means the radiation from the Sun that is doing so. OK, change “photons” to “radiation” and the logic of what I said still stands as far as I am aware. The mechanic is that radiation interacts with the fluid (gas or liquid) causing heat and the vacuum of space is cold because there is nothing for the radiation to interact with.

      I will repeat myself (yet again) since you seem to like repetition. I would have a better understanding of what you are trying to convey on your website if you communicated it better.

      10

    • #

      “Heat is motion, motion is heat”. Heat is kinetic energy of molecules (or atoms in a solid). Photons are electromagnetic radiation, all electromagnetic radiation is photons. Photons absorbed by molecules may result in flexing or rotation of molecular bonds between atoms, increased internal motion. That excess energy may be transferred in whole or in part to another molecule in a collision, resulting in that molecule moving faster. The original photon energy has been transformed into heat. In short, photon energy can only become heat if the photon is absorbed by matter. That’s as I understand it, at least.

      I’ve had a look at your site Stefan, and I agree with truthseeker – your means of communication needs improvement. A little less bold and lot less bold and underlining combined would help. Try underlining alone – it still draws the eye. Emphasis is best used sparingly – too much gives the impression of shouting.

      10

      • #
        Truthseeker

        MostlyHarmless – thank you for this clear explanation. Understanding comes when clarity of communication is achieved.

        Also the bolding versus underlining and the over use of emphasis is only the minor communication error that Stefan makes. There are much more important corrections that are required in the conveying of ideas that he needs to adopt.

        10

  • #
  • #

    Hi Cohenite!

    O´Neil as you mention may have a point 🙂

    My basic point is:
    1) There is NO detected warming of Venus surface during a looong day compared to night tme => There is hardly ANY solar energy reaching the Venus surface.

    2) Due to the fact that temperatures day vs. night are also constant much longer up in the atmosphere, it seems that sunlight is totally blocked furher up (60 km?).

    3) CO2 even in 120% concentration can only block around 20% of the outgoing Venus frequences. Therefore the total block points to a shield of blocking aerosols.

    4) Aerosol block ALL frequencies.

    => With a total blovking aerosol cloudlayer that blocks ALL frequencies, ANY blockng-effect from CO2 is not possible to measure / has no additional effect.

    5) And when the heat from below surface of Venus is stronger due to significant volcanic activity, it is very likely indeed, that the blocking Venus atmosphere could hold back heat from the interieur and thus cause a strong accumulation of heat.
    But this would then be due to aerosols, a CO2-role is limited or rather non-existing when all freqeuncies are blocked.

    SO!

    I agree, the strongly isolating Venus atmosphere could very well be maintaining an unstable Venus surface. However, this is more likely due to aerosol effect than traditional greehouse effect (CO2 etc).

    K.R. Frank Lansner

    10

  • #

    Michael hammer #21. you did waste 30y on half the job… Now you can learn in minutes, the other half: CO2 does absorb more heat than oxygen + nitrogen. You are spot on! But the other half: CO2 absorbs much more coldness than oxygen + nitrogen; that’s why is used to make ”dry ice” They kept it as taboo for you, for 30years… Unless you still believe in ”flat earth” or flat Venus – sunlight 24h on every spot = you are completely wrong – apologize to the Banana-bender.

    In my book says: CO2, same as iron, absorbs much more heat than O+N. But at night, CO2 (same as iron) absorbs much more coldness) THOSE TWO FACTORS CANCEL EACH OTHER = ZERO EXTRA AFFECT 30 years… tax $$$ ?? If you start collecting carbon tax to cool Venus – impose it only on women. Venus spins in opposite direction than any other planet = women’s fault! 30 years, now in 3 minutes, you learned more, and correctly. Few hundred hours before sunrise on Venus is colder than on Mars Looks like, you are incognito Warmist. If you want to learn – ask. Trying to feed /confuse the people with Warmist crap = naughty

    10

  • #
    MaryFJohnston

    Well there is a “Greenhouse” effect on Earth associated with CO2.

    That is if by “Greenhouse” you mean that CO2 absorbs some of the IR radiation released from the Earth and holds it for a short while, and delays heat loss to space.

    Yes then we have a GH effect.

    Many problems exist for the Warmer theory though.

    Other gases, such as Water Vapour, overshadow CO2 in the GH stakes and human CO2 is a very, very small part of the overall GH effect.

    There is NO RE – RADIATION OF TRAPPED HEAT FROM SOME IMAGINARY UPPER ATMOSPHERIC BOUNDARY LAYER.

    That theory is total CRAP.

    There is a constant temperature gradient from about 10m above ground right up to space.

    10

  • #

    Stefanthedenier:

    You write about night time “CO2 absorbs much more coldness than oxygen + nitrogen”

    Could you explain this? Are you thinking of the gas constant etc?

    Exactly how does a CO2-molecule “absorb coldness” ?

    K.R. Frank

    10

    • #
      cohenite

      Yeah, I was wondering that too Frank; maybe he means negative pressure or some such thing.

      10

    • #
      MaryFJohnston

      It gives out heat.

      Stefan is either making the concept simpler for people with no thermodynamics or he is not very good at science.

      10

  • #
    Manfred

    All this wandering about the solar system prompts me to mention, as many of you undoubtedly already know, that the climate modelling of Martian atmosphere, much to the shock-horror-surprise of the modelling gurus failed to predict the observed nature of the atmosphere. The real data recently demonstrated a state of supersaturation with water frequently occurs.

    L. Maltagliati, F. Montmessin, A. Fedorova, O. Korablev, F. Forget, J.- L. Bertaux. Evidence of Water Vapor in Excess of Saturation in the Atmosphere of Mars. Science, 2011; 333 (6051): 1868 DOI: 10.1126/science.1207957

    A certain ring of familiarity?

    10

  • #
    michael hammer

    For those who think I should be replying in thread, I have to apologise but I don’t know how to do that. I will have to get a driving lesson from Joanne some time. In the mean time to try and reply to some of the comments.

    Cameron H at 8:42. You suggest the energy absorbed by the CO2 will be passed to N2 and O2 molecules and radiated away by them at other frequencies. Remember that the ability to emit equals tha ability to absorb. If N2 and O2 were able to emit at wavelengths between 4 and 50 microns (the approximate range for thermal emission from objects at atmospheric temperatures) they would also be able to absorb at those wavelengths which would make them green house gases. The reality is that N2 and O2 cannot emit significant amounts of thermal infra red energy. So no N2 and O2 will not radiate the energy away at other wavelengths.

    Jerry Hudson 3:59 and Nullius in Verba 11:26 you both raise similar points with regard to the situation of an atmosphere entirely without green house gases. Notably, does convection occur and if so why is the lapse rate not the determining factor. The situation without green house gases is that the surface can radiate directly to the clouds. If we assume (first step) all surface energy loss is by radiation then we can calculate a surface temperature as I indicated. If this surface temperature is below that which one would expect from the predicted lase rate based on large scale convection then the predicted lapse rate will not be achieved. There may in fact be little convection and most of the surface energy loss will be by radiation. If it is higher then considerable convection will occur and this will carry away some of the surface heat meaning less is lost by radiation.

    Basically it means the heat will always find the path of least resistance but without green hosue gases in the atmosphere I suspect that will be mainly by radiation.

    10

    • #
      Louis Hissink

      Michael Hammer,

      Answering the thread is simple – you click on the small word “reply” at the end of the date and time line under your name at the start of the post.

      10

    • #
      CameronH

      Michael, Thanks for the answer. I still, however, am a bit confused. If O2 and N2 cannot radiate away energy at the top of the atmosphere how do they cool down? All other processes such as convection must stop at the TOA and so these molecules must give up energy somehow or they would just continue to get warmer and warmer. I am not trying to be clever and I really appreciate these posts. I am just trying to understand. If they do radiate away energy, at what prequency do they do this? Energy must exit the system via other frequencies, visible light for example, or the Earth would not be visible from space. As the only way energy can enter and exit the system is via radiation, would it be better to monitor all electromagnetic frequencies in and compare this with all electromagnetic frequencies out?

      10

      • #
        michael hammer

        Hi CameronH (I am trying the tip of clicking on the reply tab). The simple answer is that without green house gases the O2 and N2 would not be able to cool down. They would not get warmer and warmer indefinitely. That would imply there was a warmer source to provide the energy. They would simply stay at the temperature they were at. That is why I suspect that without any green house gases in the atmosphere there would be little convection and a low lapse rate. You must remember the scenario being proposed of an atmosphere entirely free of green house gases is distinctly bizzare and very different inded to anything we are familiar with.

        As to emitting at other frequencies such as visible light. The amount that even a black body can radiate at a given wavelength is given by Planks law and for objects in the range of atmospheric temperatures the emission at wavelengths shorter than about 4 microns is almost zero. Worse, N2 and O2 are also transparent in the visible. O2 has very high emissivity below 190 nanometers )0.19 microns but it would have to be SERIOUSLY hot to emit at that wavelength (6000K plus). Also from memory it emits in the microwave region but the emission is very small. It is hot enough for that so maybe there would be some energy loss by emission in that band but it would be small compared to what a green house gas would emit.

        By the way, I do not think you question is in any way unreasonable. On the contrary it is an extremely interesting and very challenging question and I thank you for it even if I don’t have a really definitive answer.

        10

        • #
          CameronH

          Thanks Michael, I guess what you are saying is that, without any greenhouse gases at all, all IR radiation from the surface would exit straight to space. Would the air, therefore, only get heat energy from actual physical contact with the surface?

          10

        • #
          CameronH

          If all this is true then, without GHGs there would be no cooling from the TOA then the atmosphere would heat up until it all reach the same same average temperature as the surface which would reach some maximum based on the heating effect of the total solar radiation? Does this mean that the GHGs actually cool the atmosphere and hence the earths surface?

          10

  • #
    bret

    I always find the greenhouse effect strange,

    the moon has no atmospheric gases, receives the same amount of Sun as the earth and gets way hotter,

    at night the moon temp plummets, the earth slowly cools,

    seems to me that during the day the earth gases cool the earth and at night slow don cooling,

    take the desert v tropics we can see the effect of humidity, desert hotter in the day rapid cooling at night.

    10

  • #
    michael hammer

    Baa Humbug at 1:45 I started to write a reply to you several times before I realised I need to think about your question some more. The reality is that an atmosphere without green house gases is so foreign to what we are used to that what seems ridiculous in our experience might in fact be the case for such a situation. I suspect that without any green house gases we would end up with a low or very low lapse rate and very little convection. Its just a suspicion so please don’t take it as a declaration of fact. However a blog is about discussion and sharing of ideas so I invite others to think about this hypothetical situation as well.

    10

  • #

    @Michael #21 said..(my emphasis)

    Nullis in Verbena. You raise the analogy of the green house effect of liquid water. Think about what I said, radiation from the solid surface to space is blocked and instead radiation to space occurs from the top of the green house gas column. In your analogy the “top of the green house liquid column” is the surface of the liquid water and indeed infra red radiation from a body of water all emanates from the top millimeter or fraction thereof. Any radiation from lower down is absorbed by the water above it. Its in effect the same thing.

    That statement provoked my thoughts.

    I’m led to believe that radiation is in all directions randomly (“a photon doesn’t choose not to go to a warmer surface” etc)
    In that case, if radiation was possible within a body of water, it would radiate EQUALLY up and DOWN. So a warm parcel of water would warm the layers below. BUT IN REALITY THAT DOESN’T HAPPEN.

    I guess because water being so dense (compared to a gas) that conduction and convection overwhelm any radiation that tries to take place.

    That brings me to my second provoked thought.

    The atmosphere of venus is so dense (90 times that of Earth) that every molecule of Venusian CO2 has 90 times more chance of a collision with another molecule than that of a Terrestrial molecule of CO2 has on Earth.
    Put another way, an Earth CO2 is surrounded by 999,999 other molecules whereas a Venusian CO2 is surrounded by 89,999,910 other molecules.
    The chances of a molecular collision on Venus is 90 times higher than on Earth.

    If the Greenhouse theory of radiation + backradiation holds true on Venus, then it must be all the more obvious here on Earth. Any warming via conduction, convection or any other means would be minute.

    Logic says, the more dense an object, the less chance of radiative transfer. The Venusian atmosphere is 90 times more dense than Earths atmosphere. Does this mean 90 times less chance of radiative transfer?

    Discussion please.

    10

    • #
      michael hammer

      Hi Baa Humbug;
      Your point about radiating in all direction is correct. Your point about a warm parcel of water not warming the layers below however is not quite correct, its a matter of degree. In fact the warm water will radiate downwards but the molecules in water are so close together that the distance travelled before it is reabsorbed is extremely tiny. As a result the diffusion of heat is slow.

      To go on to your second point, it is not true to say the every molecule in the Venusian atmosphere has 90 times more chance of collision with another molecule. Molecules collide with each other all the time and every molecule in earth’s atmosphere collides with others countless times per second. What is true is that the average distance travelled by a molecule before it collides with another is 90 time shorter on Venus than on Earth. This is called the mean free path. In general, as gas pressures rise the impact of conduction and convection become greater. Assuming radiation can take place (non zero emissivity and absorptivity) it probably dominates at very low pressures, at higher pressures such as a dense gas or a liquid, convection may be dominant and in a solid conduction is probably the dominant mechanism. Of course if emissivity/absorptivity is zero or near zero then its a different matter. A sheet of glass is a dense solid yet visible light passes through it freely. Your comments overall are pretty accurate.

      10

      • #

        Michael thankyou for the reply and the clarifications, and I’m glad to learn that they don’t change the gist of my post.

        Lets take this a little further.

        Let us now assume we increase the density of Earths atmosphere to that of Venus, but we keep the percentage of the atmospheric constituents the same. viz dominant Nitrogen, ~400ppm CO2.

        Now we have an atmosphere with a thermal radiation process……well….I guess 1/90th that of the current Earth atmosphere. (because convection and conduction now play a greater role)
        But we know temperatures will be much much higher due to the density of the atmosphere, but the thermodynamic role played by radiative transfer is diminished to about 1.1% of what it is now.

        With (arguably) a negligable radiative transfer process, the role of a greenhouse effect is greatly diminished.

        Unless my thought process and conclusion is in error (and they may very well be) I’d suggest we’d have to take the duster to the Venusian greenhouse blackboard and think about it back from scratch.
        What do you think?

        10

        • #
          michael hammer

          Hi Baa Humbug; No I don’t agree with your analysis. The hypothetical situations you suggest are deceptively complex and require very careful thought to answer correctly. I cover some of the material in my post number 9 in this thread (please see above).

          At 90 times the pressure with the same percentage of CO2 the total amount of cO2 will be 90 times as great. The absorption line will be broader so a greater percentage of the surface emission will be intercepted. The thermal radiation effects will not be 90 times smaller and will certainly not be neglegible. Nor will convective processes necessarily be 90 times larger. Some radiation will still escape to space through the “atmospheric window” unless blocked by dust and particulates in the 90 times denser atmosphere. In reality this issue is very far from negelgible but outside the scope of your thought experiment. So some of the energy is lost to space from the surface and some from the top of the atmosphere. Each will adjust in temperature until they are in thermal equilibrium.

          So does convection occur and its its role enhanced or reduced? This is basically the same question you asked earlier in relation to a GHG free atmosphere which I answered truthfully that I am not sure. My opinion again is that for the senario you give, there would still be enough atmospheric window left so that the radiative equilibrium temperature was lower than would result from a convective lapse rate so you would end up with little convection, a cool surface but of course significantly warmer than at present and a fairly low lapse rate. However, I think your scenario is not realistic, the 90 times higher density would trap so much more dust and particulates that it would block most emission from the surface preventing the surface from losing much energy to space by radiation and you would probably end up with a hot surface and more or less conventional lapse rate.

          If you go on to ask, at what concentration of GHG would one switch from a cool surface to a hot surface, or will the trapped dust ensure that anyway, I simply don’t know. As I said, this is a very complex question which does not allow a top of the head answer. It would take a reasearch project to answer with any degree of accuracy and I clearly do not have the time for that.

          None of this changes the theory of what is happening, just the relative magnitudes of each factor.

          10

          • #

            Hi Michael

            Just popped in to say I haven’t abandoned our discussion. I work some weird hours.

            You’ve been terrific in giving your time to respond to all our comments. A little break (and reflection time) might be good all around.

            I have more questions and comments which I’ll post when time allows. thnx again

            10

  • #

    The equation Mr. Hammer uses

    (93/67)^2 x .4/.7 = 1.1

    to pretend that an albedo correction can explain my results is scientifically incompetent. I found the RATIO OF TEMPERATURES, Venus/Earth, to be 1.176 (outside the cloud layer), and as PRECISELY predicted solely by the difference in their distances from the Sun, using the Stefan-Boltzmann equation. His equation purports to show the expected ratio in received ENERGY (actually power per unit area) in the two atmospheres is about 1.1, obviously trusting the unwary to conclude that 1.1 is close enough to 1.176, but without realizing that supposed received power ratio is not the same as expected temperature ratio. I trust those with a scientific education understand how disingenuous his “explanation” really is. I consider him personally debunked as an “authority” by this observation alone. His understanding is simply not to be trusted as scientific–and that is typical of all the defenders of the climate consensus: Readily disproved, but dismissive of the truth as it suits them (in this instance, Mr. Hammer has conveniently forgotten the Stefan-Boltzmann law, that temperature varies as the fourth-root of the incident power, not linearly with it). Go away, Mr. Hammer (and Jo Nova, see how you too have been led astray, by letting this garbage in as authoritative science).

    31

  • #
    CHIP

    I don’t understand when Warmists say that the atmospheric CO2 greenhouse on Venus is responsible for its exceptionally high temperature of 500C. Even their own equations disprove the idea. Here, I’ll show you: I got a value of 230K for the assumed effective surface temperature of Venus from the following website: http://www.imcce.fr/vt2004/en/fiches/fi¬che_n13_eng.html NASA doesn’t give one on their Fact-Sheet. According to the S-B law 230K gives us 158.6W/sq.m. Considering that the atmospheric CO2 concentration on Venus is believed to be about 97.5% (or 975,000ppmv) the IPCC’s logarithmic equation for converting CO2-increments into radiative forcing increments gives us a radiative forcing value of 73.77W/sq.m for total the atmospheric CO2 on Venus. Therefore new surface irradiance is 232.37W/sq.m which gives us 253K. So the atmospheric CO2 on Venus can only increase the temperature by 23C. And it doesn’t matter if the effective temperature of 230K above is wrong. It doesn’t matter what baseline temperature you pick. The unfortunate fact is 73.77W/sq.m simply isn’t enough to increase the temperature on Venus by any significant amount. Hasn’t anyone told Hansen yet? I think we ought to. It might stop him worrying so much.

    10

    • #
      MaryFJohnston

      Hi Chip

      You are right.

      What’s causing the high temperature on Venus is a solar entity called THE SUN.

      The concept of CO2 induced runaway Global Warming is easily falsified with basic physics and bit of engineering know-how and it is uncontested that the limiting factor to increased global temperature is the Solar energy output.

      10

  • #
    bret

    so how long are the days on Venus 200+, the night time side is as hot as the sun facing side,

    so co2 is great at the movement of heat or is the planet itself generating this heat.

    10

  • #
    bret

    one venus day = 245 earth days.

    10

  • #
    bret

    i just love the pontificating by the scientists,

    bet it is heat from the planet, tried it with my empty pot on my aga with the lights out- ho ho,

    guess what my thermometer said suspended above the pot.

    10

  • #
    MaryFJohnston

    Hi michael hammer

    October 10, 2011 at 7:47 pm · Reply

    I think the ground IR is pretty much fully absorbed by atmospheric CO2 by about 10 metres Above Ground Level.

    The energy absorbed by all GH gases including CO2 is then transferred very rapidly by the usual processes to neighboring molecules. The heated air rises until PVT equilibrium is achieved.

    Convection, KE exchange etc call it what the Warmers like it is not all by Radiation they just picked radiation because it was handy at the time.

    Heat loss to that great Heat Sink in the sky (space – 273.16 deg C) has to be by Radiation, at least they got that right.

    10

    • #
      michael hammer

      Hi May;
      With the proviso that the absorption by CO2 is not at all wavelengths only those where CO2 absorbs only I agree with your first 2 statements. Energy meovement is not all by radiation, clearly convection plays a part especially in transporting energy between the equator and the poles however my calculation suggest radiation is the dominant mechanism (although I have to say other scetics whose opinions I respect disagree with me on this). All this is entirely consistent with the situation that the impact of a green house gas is to block radiation from the surface to space at the GHG wavelengths and replace it with radiation to space at those wavelegnths from the top of the GHG column.

      10

  • #
    Louis Hissink

    To understand the so called greenhouse effect, one has to understand the origin of the use of this term, i.e. why Sagan used it during the 1950’s in the first place. Greenhouses work by circumventing convection with a physical barrier, the glass construction. The atmosphere has no such barrier (except when the presence of clouds impede the passage of heat to space) but it also be useful to study the earth’s thermal profile from its surface to space – and which temperature is being arbitrarily used to support the S-B equation that is the core GH argument used here?

    10

  • #
    KevinK

    From the post;

    “Michael Hammer has some 20 patents in spectroscopy, and he explains why the Greenhouse Effect — where CO2 and other gases absorb and emit infra red — is very real, and backed by empirical evidence.”

    With all due respect to Mr. Hammer there is a LARGE difference between a “Greenhouse” gases ability to absorb / warm / remit / cool (in that exact sequence) Electromagnetic Radiation (i.e. Infrared Light) and said “Greenhouse” gases ability to produce a “Higher Equilibrium” Temperature at locations inside a complex heat transfer system.

    Yes indeed some gases absorb Infrared light and “redirect“ some of the energy in the light backwards (i.e. Backradiation) towards the surface that emitted the energy. Yes there is data that demonstrates this.

    The MUCH LARGER QUESTION is what exactly are the RESULTS of this redirection of some small portion of the energy flowing through the complex Sun / Earth / Atmosphere / Universe System ???

    To answer this question several more important details of how energy flows through a complex system must be examined;

    1) Is the total thermal capacity of the “Greenhouse” gases LARGER or SMALLER than the total thermal capacity of the upper levels of the Oceans or soil at the surface of the Earth ?

    2) How many orders of magnitude difference is there between said thermal capacities (if any) ?

    3) At what speed does the energy flow through the “Greenhouse” gases ?

    4) At what speed does the energy flow through the “Non-Greenhouse” gases ?

    5) Can the “Greenhouse Effect” slow the flow of energy through the system enough to cause some energy to be “leftover” at the end of each cycle of energy input to the system (i.e. sunrise/sunset) ?

    I will posit the following answers (expecting lots of incoming flaming arrows);

    1) NO, the thermal capacities of the “GHGs” are minuscule when compared to the thermal capacity of the Oceans alone. This makes the whole hypothesis that the “GHGs” are “forcing” the surface of the Earth into thermal equilibrium with said gases LAUGHABLE at best.

    2) The thermal capacity of the Oceans is three to four orders of magnitude LARGER than the thermal capacity of the “GHGs”.

    3) Energy flowing through the system via the “Greenhouse Effect” is travelling at close to the speed of light. There are some small delays as the energy is absorbed / remitted by the “GHGs” but even this slight delay is miniscule.

    4) Energy flows through the “Non-GHGs” at the speed of heat (aka Thermal Diffusivity) this is quite a bit slower than the speed of light.

    5) No, the ”Greenhouse Effect” is only capable of slowing / delaying the emission of infrared radiation from the surface of the Earth by somewhere between a few or perhaps as much as a hundred milliseconds. Please note that light can travel from the surface of the Earth to TOA in about 2 milliseconds, so even if one particular package of energy makes TEN returns trips as “backradiation” (after which the package of energy is less than 1% of its original size) the package has only been delayed by 20 milliseconds. For reference purposes one day on Earth contains about 86 million milliseconds.

    So, the question becomes;

    What is the final result of the presence of “GHGs” in the atmosphere of the Earth?

    I posit the following hypothesis;

    When comparing an atmosphere with “GHGs” against an atmosphere sans “GHGs” it appears that the presence of “GHGs” causes a small amount of energy to flow through the system at the speed of light. The speed of light is quite a bit faster than the speed of heat.

    THUS the presence of “GHGs” causes all of the gases in the atmosphere to heat up faster (more quickly) after an increase in the energy input to the system (i.e. sunrise or the dissipation of clouds). Alternatively, the presence of “GHGs” causes all of the gases in the atmosphere to cool down faster after a decrease in the energy input to the system (i.e. sunset of the accumulation of clouds).

    So in summary;

    1) The “Greenhouse Effect” has nothing to do with the “Average Temperature” of the surface of the Earth. This is determined primarily by the massive thermal capacity of the Oceans.

    2) The presence of ”GHGs” only changes the “response” / “lag” / “delay” time of the gases in the atmosphere.

    3) This effect is so small that we probably cannot afford to measure it.

    4) The historical temperature databases do not contain the necessary data (i.e. dT/dt) to confirm/refute this hypothesis.

    5) The calculations based on the “Stefan-Boltzmann” equation which attempt to predict the temperature of the surface of the Earth with/without “GHGs” are flawed. This is understandable since they do not properly consider the thermal capacities / depth / thermal diffusivities of the materials involved.

    So the “Greenhouse Effect” does not cause the alleged response (i.e. a Higher Equilibrium Temperature) at the surface of the Earth within the complex Sun /Earth / Atmosphere / Universe system.

    Cheers, Kevin.

    (FYI, I also have over 20 patents, but I hope that my arguments above will stand on their own without an “Argument from Authority”).

    10

    • #
      MaryFJohnston

      Hi KevinK

      Particularly liked 5 and 5 where you quantify residence time of IR energy in the system and knock misapplication of a thermodynamics equation.

      It never ceases to amaze me that environmentally qualified “scientists” will take an equation with complex parameters and strict boundary limits and adapt it to some element of “Climate Science” that need beefing up.

      My comment at 62 above was only a “first principles” based comment but you seem to have great familiarity with the material.

      10

      • #
        Mike W

        I was glad to see you allude to this earlier, it seems to get overlooked with all the emphasis on radiative transfer. Once those IR photons are initially absorbed, equipartition says that energy is very rapidly transformed into molecular kinetic energy, and the radiative modes of CO2 are effectively “frozen out” until TOA. I have read that H20 radiates microwave energy at tropospheric temperatures and pressures. The warming by “back-radiation” concept seems very much at odds with equipartion to me, but there’s a good possibility I haven’t grasped it completely.

        So yeah, warming by back-radiation? That’s bollocks, isn’t it? It’s warming because the absorbed energy is transformed into increased linear kinetic energy between collisions. I’m open to suggestions/explanations.

        10

    • #
      bananabender

      5) The calculations based on the “Stefan-Boltzmann” equation which attempt to predict the temperature of the surface of the Earth with/without “GHGs” are flawed. This is understandable since they do not properly consider the thermal capacities / depth / thermal diffusivities of the materials involved.

      The calculations assume that the Earth is flat disc that receives uniform radiation. This argument is totally farcical.

      Last time I checked the Earth was a rotating oblate spheroid with a mass of 5.9742×10^24 kg that is 70% covered in water.

      10

    • #
      michael hammer

      Kevin; firstly let me say that I agree the number of patents one may or may not hold is totally irrelevant. All your points seem to be directed towards the view that a green house gas causes warming because it somehow delays the passage of energy to space. I fully agree thats not at all how it works. Consider the case of a space blanket which is simply 2 thin mylar sheets bonded together with an extremely thin layer of aluminium evaporated onto one of the inner layers of the two sheets. Its physical and thus thermal mass is utterly miniscule yet it still has to power to warm someone significantly. And it can do so even if no part of the blanket actually touches the persons body. To answer the probable comment that this warming is due to blocking of convection let me point out that the same sheet without the evaporated aluminium layer is just as effective in blocking convection yet does not have anything like the same warming power.

      A green house gas works by absorbing the surface radiation before it can escape to space (only at the GHG wavelengths of course). This energy is converted to heat in the atmosphere and contributes to establishing a lapse rate. In turn, the top of the GHG column radiates to space at the GHG wavelengths. So it replaces surface emission with emission from the top of the GHG column. Since the top of the GHG column is normally colder than the surface the emission to space is reduced hence the GHG reduces heat loss to space. The surface responds by slightly increasing its temperature until the increased emission at other wavelengths makes up for the reduction due to the GHG at which point equilibrium is restored again.

      10

  • #
    bananabender

    Michael Hammer is an electrical engineer who has spent over 30 years conducting research for a major international spectroscopy company.

    An appeal to authority.

    [No it wasn’t. At no point did I declare that Hammer must be right because he was x or y I write about his bio so that people know he’s worth listening too and know where he is coming from. Tone down the aggression and stick to the science please. — JN]

    In the course of this work he generated around 20 patents which have been registered in multiple countries. Patents are rarer and more rigorous than peer reviewed papers, only available for economically valuable work, and costing thousands of dollars to process and maintain.

    Nonsense. Patents are easy to obtain and are often frivolous. An extremely high standard of work is required for publication in high impact scientific journals.

    Back to the arguments.

    Diagram 1 does not show proof of the Greenhouse Effect at all. It merely shows the temperature profile of a tiny section of the atmosphere at 20km altitude at one particular moment. If we superimposed tens of thousands of these spectral charts taken from all over the globe we would find a typical left-skewed distribution with no missing radiation at 600-800nm.

    The fact is that IR radiation emitted upwards from the surface is fully absorbed by the atmosphere within a few metres of the ground. So there is no possibility that the satellite is measuring IR radiation emitted from the surface. What the satellite is actually measuring is EMR emitted from hot gas molecules high in the atmosphere.

    The atmosphere is not heated by any magical Greenhouse Effect because gas molecules cannot store energy. This fact has been known since 1913 based on the work of Niels Bohr.

    Any radiation striking a gas molecules basically does one of two things:

    a) EMR excites an electron to a higher orbit. The electron instantly falls back to the original orbit releasing photons. No net heating of the molecule occurs. This is the basis of spectrophotometry.

    b) the radiation energises the internal molecular bonds causing stretching or twisting. This increases the rate of collision between molecules causing the gas to heat and expand. This energy is all transferred to other nitrogen and oxygen molecules with a few picoseconds. No atmospheric heating actually occurs because the gases simply rise, expand and cool (adiabatic lapse).

    How the atmosphere is really heated.

    This the standard model used by meteorologists since the 1920s (the vast majority of meteorologists are also AGW sceptics). EMR from sunlight heats the ground and oceans. Most of this energy is absorbed by the oceans. Water is evaporated and re-condenses as clouds releasing latent heat. This accounts for virtually all of the atmospheric heating. Smaller amounts of heating occur due to conduction and convection of heat from ground surfaces and tiny amount of heating occurs due to (non-Greenhouse) radiative warming.

    Convection transfers heated air from the surface to the upper atmosphere. This heat is then radiated into the vacuum of space.

    ie The Earth heats the atmosphere rather than the atmosphere heating Earth.

    Why is Venus so hot?

    Two words. Atmospheric pressure.

    The density of the Venusian atmosphere is ~90Bar at ground level. According to the ideal gas Law pV=nRT Venus will be bloody hot at the surface. The same Pv=nRT equation partly explains why Earth’s surface is warmer with an atmosphere.

    Bananabender did a fair bit of spectroscopy (near IR, NMR, GC/MS, AAS etc) in his youthful past as a biochemist/food scientist. However that is irrelevant because appeals to authority aren’t scientific.

    10

    • #
      michael hammer

      Bananabender; I have already commented on your appeals to authority comments. I do not see the need for me to apologise for my accomplishments in related spheres of endeavour and I do not think it is unreasonable for an author to be introduced with a brief statement of prior work. Indeed that is a quite common occurance and I note many warmists are introduced in exactly that way. I do however fully accept that to interpret such an introductory statement as an appeal to authority is not appropriate and I have not done so. I have not said I must be right because I am an expert. In fact if you look through these posts you will see I willingly admit where there is something I do not know. I am very happy to be judged on what I say on ths subject. By the way, multi national patents may or may not be easy to obtain in your opinion but they cost about $100,000 each. Someone has to think enough of the work to spend that sort of money.

      Further, you and others here seem to think I am a warmist or support the warmist philosophy. Nothing could be further from the truth. I am extremely sceptical of the theory of CAGW but I am not prepared to bend the truth as I see it to support a sceptical position (no I am not suggesting you are).

      So onto you points. Yous seem to be suggesting that if numerous Nimbus plots were taken and averaged the profel would flatten out and the dips would disappear. That is simply WRONG. I also have a plot taken from one of the mars probes (looking back towards Earth). It is an average over an entire hemisphere and shows exactly the same pattern.

      You say surface emission is fully absorbed by the atmosphere within a few metres. So if this energy is absorbed does it disappear? How does it manifest itself since you say such absorption cannot warm the atmosphere? By the way your comment is WRONG in important detail. Surface radiation in the atmospheric window makes it all the way out to space without being absorbed on the way. Only radiation at the GHG wavelengths is absorbed. From the intensity of emission at each wavelength it is possible and indeed quite easy to compute the temperature of the object which is doing the emission. That is what the dotted overlay lines in the plot are showing. At the CO2 line the emission tempeature is about 215K while at say 1100 or 1200 wave numbers the emission temperature is 300K. The only part of the atmosphere as fars as I know which is at 215K is the tropopasue and the only part which is at 300K is the surface. That totally supports the explanation I gave.

      Your explanation of what an electron does is sloppy. You talk about atomic absorption (which by the way usually requires discrete atoms not atoms bound in molecules which is the case for most atmospheric gases other than say argon)and say the electron instantly falls back to its original orbital. This is simply not true – read up about metastables and excited atom lifetimes. The lifetime is not zero! The rotational vibrational energy is not transferred to other moluecules in pico seconds. Transfer requires a collision and the average time between collisions is more than pico seconds. Mean free path at STP is around a few microns and the average velocity is around 300 m/sec so try a few nano seconds or more not pico seconds.

      You talk about missing radiation at 600-800 nm. Its not at 600-800 nm its at 600-800 wavenumbers whcih corresponds to about 15 microns not 0.6 microns.

      You state that the atmosphere cannot be heated by greenhouse effect because gas molecules cannot store energy. REALLY!!!! So if a molecule absorbs a photon and thereby vibrates more rapidly or rotates more rapidly that motion is not a form of energy storage? Sir I think you are gravely mistaken. What do you think happens if a molecule strikes a hot surface? If it cannot store energy the gas could not be warmed by such contact so what pray tell can warm a gas?

      You talk about the reason Venus is so hot. Please read my post on this thread – its number 9 ot 10 (I cant see the numbers on my computer). You happen to be right thats its primarily due to the thick atmosphere but ONLY becuase the green house effect on Venus is so strong.

      I could go on but I think I have said enough to put my case.

      10

      • #
        bananabender

        Michael,
        if you are going to [snip inflammatory]

        By the way, multi national patents may or may not be easy to obtain in your opinion but they cost about $100,000 each. Someone has to think enough of the work to spend that sort of money

        .

        I never claimed that patents are cheap. But is totally nonsensical to claim they are extremely hard to obtain eg the Amazon one-click patent. In most cases filing for a patent is simply a cost-benefit analysis. One group I worked with failed to patent a biotechnology process because we thought it wouldn’t be a commercial success. It has since generated hundreds of millions of dollars in royalties for another company. (My personal share of royalties would have amounted to several million dollars had we patented the process.)

        You say surface emission is fully absorbed by the atmosphere within a few metres. So if this energy is absorbed does it disappear? How does it manifest itself since you say such absorption cannot warm the atmosphere? By the way your comment is WRONG in important detail. Surface radiation in the atmospheric window makes it all the way out to space without being absorbed on the way.

        If you are going to criticise me at least have the courtesy to correctly repeat what I said.

        I said that IR radiation is converted to kinetic energy. This causes the atmosphere to expand. As the Sun passes over the Earth’s surface the atmosphere directly below expands outwards to create the Diurnal Bulge. It has been measured since 1966. Hard experimental evidence not 19th century theoretical claptrap like the GE.
        http://articles.adsabs.harvard.edu/full/1966SAOSR.207…..J/0000001.000.html

        Heat (“IR radiation”) is transferred to the TOA by convection. Don’t let the ugly fact that upwelling IR cannot travel more than 10m slay your beautiful theory

        Your explanation of what an electron does is sloppy. You talk about atomic absorption (which by the way usually requires discrete atoms not atoms bound in molecules which is the case for most atmospheric gases other than say argon)and say the electron instantly falls back to its original orbital. This is simply not true – read up about metastables and excited atom lifetimes. The lifetime is not zero! The rotational vibrational energy is not transferred to other moluecules in pico seconds. Transfer requires a collision and the average time between collisions is more than pico seconds. Mean free path at STP is around a few microns and the average velocity is around 300 m/sec so try a few nano seconds or more not pico seconds.

        Again you misrepresent me. I used a simple explanation for a layman. I never claimed that CO2 underwent atomic absorption.

        You are engaging in semantics. The lifetime is close to zero as to be meaningless. In any case the time to TOA (100km) is still measured in millseconds.

        One m3 of atmospheric gas (approx 50 moles) at STP contains ~3×10^24 molecules. That is ~10^7 molecules per cubic micron. Are you seriously claiming that a collision between between molecules travelling at 300m/s at that density will take microseconds?

        You state that the atmosphere cannot be heated by greenhouse effect because gas molecules cannot store energy. REALLY!!!! So if a molecule absorbs a photon and thereby vibrates more rapidly or rotates more rapidly that motion is not a form of energy storage? Sir I think you are gravely mistaken. What do you think happens if a molecule strikes a hot surface? If it cannot store energy the gas could not be warmed by such contact so what pray tell can warm a gas?

        The fact is that an individual gas molecule can’t hold onto that photon or avoid a collision. So it can’t store heat for any significant period of time.

        Heat is work. This remarkable fact was discovered by Mr Joule, a humble English brewer, who preferred experiments to theory.

        The only way to warm a gas is to force molecules to collide more frequently. This can only be done by compressing a gas. Attmepting to heat an uncompressed gas simply causes expansion and cooling. On the Earth and Venus the comptression is achieved by gravity.

        You talk about the reason Venus is so hot. Please read my post on this thread – its number 9 ot 10 (I cant see the numbers on my computer). You happen to be right thats its primarily due to the thick atmosphere but ONLY becuase the green house effect on Venus is so strong.

        Total unadulterated nonsense.

        The CO2 absorption bands are already fully (>99.99999%) saturated at the 380ppm C22 concentrations found on Earth. You can’t be taken seriously if you argue that CO2 can absorb several magnitudes more IR energy.

        I could go on but I think I have said enough to put my case.

        Ira Glickstein PhD (another electrical engineer) has already stated virtually identical arguments (expressed rather more eloquently) to yours on WUWT. He has been seriously mauled by readers on each occasion.

        http://wattsupwiththat.com/?s=glickstein

        10

        • #
          MaryFJohnston

          Hi Bananabender

          “”Heat (“IR radiation”) is transferred to the TOA by convection. Don’t let the ugly fact that upwelling IR cannot travel more than 10m slay your beautiful theory””

          This is the thing in a nut shell!!

          10

        • #
          michael hammer

          “The only way to warm a gas is to force molecules to collide more frequently. This can only be done by compressing a gas. Attempting to heat an uncompressed gas simply causes expansion and cooling.”

          How interesting, I always thought that passing a gas over a hot surface would warm it. Clearly I must be imagining that hair dryers work. The air coming out cannot possibly be hot according to your comment just expanded and cold. Its just my imagination.

          “The fact is that an individual gas molecule can’t hold onto that photon or avoid a collision. So it can’t store heat for any significant period of time.”

          I am also pleased to know that a hot gas cannot burn me according to your comment since it can’t store heat ie: energy and it takes energy to burn. Then again, hot air guns are used to melt plastic. Presumably my view that melting plastic takes energy is just another one of my delusions.

          Bananabender, I don’t think there is much point in us discussing this any longer. Clearly neither is going to convince the other.

          11

    • #
      MaryFJohnston

      BB

      Agree that temperature is proportional to pressure.

      P.V = nRT

      10

  • #

    Guys please lets give the “appeal to authority” statement a rest.
    Michael didn’t write the intro to this essay, Jo did.
    And she has not said “Michael is right because he has patents” nor did she insinuate anything similar.

    Guest posts are rare here at Novas, so Jo introduced Michael by letting us know who the man is and why he is writing a guest post.

    Your contributions and points raised are excellent and I’m awaiting Michaels replies.
    Lets keep to the details.

    with respect
    Baa

    —-

    Thank you Baa! –Jo

    10

    • #
      Truthseeker

      Baa – it is perhaps unfortunate that any introduction of the author by summarising their relevant achievements is by implication an appeal to authority, but it is still such an appeal. Stating either what the writer has done in this area is meant to add weight to what they have written. This is common practice and gives information that the reader may use (rightly or wrongly) in evaluating the content.

      I too am waiting for Michael’s replies, especially to Dr Huffman who has made two comments on this post (#26 and #57) and he is the author of the original analysis that started this particular discussion.

      10

    • #
      cohenite

      I’d really like Dr Huffman and Michael Hammer to look at this 3 parter on Venus which looks at the issues of pressure and greenhouse in great detail:

      http://scienceofdoom.com/2010/06/12/venusian-mysteries/

      10

  • #
    Bryan

    Michael Hammer posts here have a lot in common with posts by Ira Glickstein at WUWT.
    Both are Electrical Engineers.
    The posts are well presented and thought provoking.

    However it leads me to suspect that a good grounding in thermodynamics is not compulsory for all electrical engineering degrees.
    Michael devotes a lot of time to convince us that CO2 and H2O have significant IR absorption and emission.
    We all agree on this!
    But what has that to do with a 33K claimed “greenhouse effect”?

    The logical link has not been established.
    As cohenite says above this point has been dissected at length at scienceofdoom.
    Even SoD admits that radiation has no effect on the dry adiabatic lapse rate.

    10

    • #
      michael hammer

      Bryan; with regard to a logical link between the 33K wrming and CO2 energy absorption please see my post number 9 on this thread. It was origianlly going to be part of the article but turned out to be less work to leave it as a post under the article.

      10

      • #
        Bryan

        Michael, thanks for the reply.

        I have read your post 9 and by shifting assumptions you correctly arrive at a 13K rather than a 33K greenhouse effect (with these assumptions).

        From the thermodynamic point of view the dry adiabatic lapse rate = -g/Cp.

        g = Gravitational Field Strength and accounts for the effect of gravity in the atmosphere.
        For instance setting up a density gradient.

        Cp is the heat capacity at constant pressure and there is a lot of thermodynamics behind the simple symbol.
        Cp includes the heat transfer coefficients for conduction,convection AND radiation.

        Your analysis is really focusing primarily on the radiative process.
        This is useful but it is only part of an overall energy transfer process.
        Joseph Postma by including thermodynamics and a night/day paradime finds no room left for any greenhouse effect.
        Nothing you have written about the radiative properties of CO2 and H2O here is incompatible with such an analysis.
        Thank you for the effort you have made initiating this interesting debate and the patience shown in answering the many points raised.

        10

    • #
      MaryFJohnston

      Bryan

      I agree.

      I am immediately put off by calculations that try to find a theoretic quantity such as the “33K claimed “greenhouse effect”?”

      Such calculations are pointless because the target is obvious and short cuts will be used to replicate real life temps.

      10

  • #
    michael hammer

    I am finding that while many people make constructive comments or ask sensible searching questions which I greatly value, there are some who throw around uninhibited accusations of incompetence against me while making long winded assertions of how the world works without any backup at all. Assertions that conflict with much well established science. When I spend time trying to reply in a considered fashion I find that what I write is ignored and instead there are further assertions that I am ignoring their wisdom.

    Since I find that somewhat frustrating I would like to ask these people a question and yes I have raised this before.

    Space blankets exist and are readily available. They are commonly used in rescue missions to warm people who are at danger of hypothermia. They consist of a double layer of very thin mylar film with an evaporated layer of aluminium between the 2 layers which acts like a mirror. These blankets work extremely effectively yet the same double layer of mylar without the evaporated aluminium between the layers has very little warming effect. The space blanket is still very effective at warming even when it is far colder than the person it is warming, a typical situation in say a snow rescue mission. These are facts that can be easily verified – just go out, buy such a space blanket and try it. Please explain according to your world view how these blanket work? In case anyone thinks I should first answer this issue myself, I have done so in my previous post on the second law of thermodynamics at this site

    11

    • #
      Mike W

      The rotational vibrational energy is not transferred to other moluecules in pico seconds. Transfer requires a collision and the average time between collisions is more than pico seconds.

      Hey Mike, I find this really interesting. I’ve done a little reading of some physical chemistry textbooks, and posed the question on real climate. The answer I got (from Eli) was that the vibrationally excited Co2 molecules were extinguished very quickly through collisions, that they really didn’t have a chance to re-radiate at tropospheric temperatures and pressures. Pretty much a one-way cascade thanks to equipartition (different story at TOA). The same gist seemed to be supported in the PhysChem textbooks I looked at.

      10

      • #
        cohenite

        Well eli is an expert on excitation; according to him at 100 m Torr the average time between molecule collisions is a microsecond with somewhere between 100-1000 such collisions required to lose a quantum of vibrational energy aquired through absorption; even so this rate of deexcitation through collision is many orders of magnitude quicker than excitation due to thermal radiation; this is the explanation as to why saturation can never occur, according to eli.

        10

      • #
        MaryFJohnston

        That’s right Mike.

        CO2 takes in IR but rapidly disperses the energy via collisions.

        10

      • #
        michael hammer

        MikeW discussing this from the point of view of individual molecules is not very productive. Its about the same as trying to understand bulk flow of a gas by looking at the path of every molecule. Consider, the gas has a temperature and at the CO2 wavelength it has high emissivity. This measn the gas will radiate energy at that wavelength. NO I have no idea which moleculaes wll do the radiating but that does nto matter. Collectively the gas will radiate.

        10

        • #
          Mike W

          Thanks Michael, I hope you’ll continue to contribute but I’ll understand if it gets too time/energy consuming.

          I want to see if I have a proper grasp of how absorptivity = emissivity works in this context. Co2 absorbs strongly at 7 and 13um (sorry, I can’t intuitively use wavenumber yet). However within the troposhere that rotational vibrational energy is rapidly transformed into translational kinetic energy (a hotter atmosphere). We don’t see (much) re-emission of that energy at 7 and 13 um (equipartition), that’s why we have those “notches” taken out of the spectrum as seen by Nimbus. Rather that absorbed energy has been (kinetically) distributed over the whole of atmosphere, and the (hotter than it otherwise would be) whole of atmosphere “then” emits. The energy “missing” from the notches has been redistributed over other parts of the spectrum (once a new thermal equilibrium has been reached right?). In this sense absoptivity = emissivity.

          How does that sound?

          Happy for others to chime in too.

          10

          • #
            Mike W

            I re-read that, and realised I could have given the impression that I think the atmosphere radiates like a blackbody, err, at least I don’t think I think that. Obviously, gasses, even complex ones, can only emit over specific, discrete wavelengths. I think you can see what I’m getting at though:

            If absorptivity = emissivity, how do we reconcile the energy missing from the notches?

            I am fairly sure that just because the system absorbs at 13um it does not mean the system has to emit an equivalent amount of energy at 13um, is this correct?

            Sooo, ultimately, is the missing energy in the notches redistributed and radiated elsewhere in the spectrum, orrrrr, is it
            just “delayed” (although I can’t see how such a delay would manifest as the notch in Nimbus’ observations).

            10

          • #
            michael hammer

            Mike CO2 actually absorbs strongly at 2.6 microns, 4 microns and 14.7 microns. Certainly the energy absorbed by the CO2 is distributed among other species and in effect warms the local gas (by extension one could say it warms the atmosphere). Absorption depends on on the absorptivity however emission depends on both emissivity (=absorptivity) AND the temperature of the gas. It is not helpful to try and think of this in terms of what any individual molecule is doing. Consider the gas in bulk instead. So the CO2 right throughout the atmosphere is emitting at 14.7 microns (its too cold to emit significantly at 2.6 or 4 microns) in all directions and most of that emission is quickly re-absorbed by other CO2 molecules close by. Only at the top of the atmosphere will the upwards travelling emission be able to escape to space. The notch comes about because the gas at the top of the atmosphere is much colder than the surface so it is emitting less energy. The difference in energy between surface emission and top of atmosphere emission is redistributed within the earth atmosphere system by conduction convection and radiation. However since earth is a body surrounded by vacuum when we talk about energy balance of the planet as a whole we only need to consider radiation.

            If we were to use a detector of 14.7 micron radiation and monitor the level as a function of altitude we would find it continuously falling with altitude in line with the gas temperature according to Planks equation.

            To maintain equilbrium the planet as a whole has to emit slightly more energy at other wavelengths to make up for the reduced energy radiated at 14.7 microns and to do that it has to get very slightly hotter.

            10

          • #
            Julian Braggins

            Someone else posted this link earlier but I can’t find it again to acknowledge, but it brings up an important fact, well backed up with longstanding experimental papers, that a mixture of gases behaves differently from individual gases in their emission and absorption.
            These conclusions seem very important to this discussion.
            http://www.biocab.org/Mean_Free_Path.pdf

            By Nasif S. Nahle

            Conclusions
            This assessment demonstrates that the effect of an increased warming caused by an increase of absorptivity of IR by water vapor due to overlapping spectral bands with carbon dioxide does not happen in nature.
            On the overlapping absorption spectral bands of carbon dioxide and water vapor, the carbon dioxide propitiates a decrease of the total emissivity/absorptivity of the mixture in the atmosphere, not an increase, as AGW proponents argue 1, 2, 3.
            Applying the physics laws of atmospheric heat transfer, the Carbon Dioxide behaves as a coolant of the Earth’s surface and the Earth’s atmosphere by its effect of diminishing the total absorptivity and total emissivity of the mixture of atmospheric gases.
            Dr. Anderson and I found that the coolant effect of the carbon dioxide is stronger when Oxygen is included into the mixture, giving a value of ΔE = 0.3814, which is lower than the value of ΔE obtained by considering only the mixture of water vapor and carbon dioxide.

            10

        • #
          MaryFJohnston

          Hi Michael Hammer

          The question we have is related to your comment that:

          “Consider, the gas has a temperature and at the CO2 wavelength it has high emissivity. This measn the gas will radiate energy at that wavelength.””

          Question: Do you know, or have you just assumed, that CO2 will emit radiation before the energised molecule has a collision with another air molecule?

          If, as is likely, there is a collision, the CO2 ,molecule will not be able to radiate but the gas parcel can expand and rise and so bring the ENERGY closer to the TOA here it can be radiated (that being the only mechanism possible) .

          The reason that temperatures drop so quickly at night in desert regions is that there is no water vapour (gas) in the air.

          There is however still plenty of CO2 but it does not seem to act as a GHG at sufficient level to delay cooling.

          Water is the major GHG and is 95% or so of that GHG effect.

          CO2 is only 4 % or so of the total GHG effect but there is a bigger problem

          Human associated CO2 is only 3% or so of ALL CO2 effect.

          Constantly quoting say 1 deg C rise for a CO2 doubling is misleading for a number of reasons.

          1. I very much doubt that real physicist would agree that CO2 doubling would give anywhere near that.

          2. Human CO2 is paltry in the overall GHG effect and the Doubling comment ignore this fact and try to make it look as though humans cause the whole temp increase.

          My earlier comments are at 62 and 64.1

          10

          • #
            michael hammer

            Mary; many unrelated points in your post. Whether or not a particular energsed molecule emits before it collides ith another air molecule is not relevant. What is relevant is that on average a body of CO2 gas will emit 14.7 micron radiation at a level dependent on its temperature according to Palnks law. The CO2 right throughout the atmospheric column is continuous emiting and absorbing energy at 14.7 microns however it is only the CO2 at TOA which can radiate to space. Radiati0on from lower down is re-absorbed before it can escape to space.

            Temperature drop in the desert – I completely agree with your comment

            with regard to delaying cooling, people keep referring to retarding or slowing down the energy loss. I don’t see any relevance to this. All the CO2 does is create a lower emission temperature to space at 14.7 microns and that reduces the emission to space at that wavelength. Thermal mass and time constants have nothing to do with this.

            With regard to relative effects, water is inded the major GHG. My number is that water represents 80-100 watts/sqM while CO2 is about 27 watts/sqM. However water also has a second effect through clouds which substantially cool the Earth. Warmists get te sign of incremental cloud feedback wrong which leads them to predict 3-4C of warming per doubling of CO2 whereas a more reasonable estimate is probably about 0.3-0.4C per doubling.

            I am not sure if human CO2 is responsible for 3%. I think can make out a good case for saying that the rise in CO2 has nothing to do with man at all and I think others are now starting to wonder about that as well. To assume that the rise in CO2 is all due to man as the warmists do strikes me as blinkered and unjustified.

            The figure of 1C per doubling of CO2 is before any feedbacks and I have to disagree that most physicists would disagree. On the contrary I think most would agree – the calculation is simple and straight forward. HOWEVER as I said thats before feedbacks and I am convinced for several reasons that the overall feedback is very much negative which will substantially reduce that value.

            In your last point you say “make it look as though humans cause the whole temp increase”. Ahh what temperature rise is that exactly. I have data which in my view shows the global temperature rise plots are manipulated. I do not accept that the temperature today is any higher than it was in the 1930’s which implies there is no case to answer in the first place. Lets not forget, the actual hard experimental data does not support any problem at all. Take out the ever increasing “corrections” to the historical record and the monotonic temperature rise disappears. How the hell can historical data recorded 80 years ago require an ever increasing level of correction? The entire alarm is entirely based on model predictions whch have demonstrated no predicitve accuracy what so ever. I note they are now predicting an iceage for England. Does that mean man is simulatneously resonsible for warming the planet and cooling the planet. The whole theory of CAGW is a farce and one does not need to deny 100 years of careful science to show it.

            10

    • #
      Truthseeker

      Michael Hammer, I do not find your space blanket analogy very convincing. The human body continuously generates heat (if it is alive) and the space blanket merely slows the loss of heat because the nature of it does not allow the convective transfer of heat away from the body that would normally occur in cold conditions like the snow. Aluminium has the dual characteristics of conducting heat well when in direct contact with the heat source (aluminium cookware as an everyday example) and reflecting heat when it is not (fire protection suits use aluminium). The mylar simply allows the aluminium to act in a reflective way which is a known property of it. This prevents the convection transfer of heat from the warm body to the cold environment, so the warm body stays much closer to its natural (and therefore comfortable) temperature. Now if you are trying to use this as an argument for the greenhouse gas theory, you will have to join a few more of the dots for me.

      10

      • #
        michael hammer

        You are right to say the aluminium layer is reflective but that is not necessary to stop convection. Convection carries heat away in a moving fluid stream. To stop it you only need a physical barrier which prevents te fluid movement. The mylar does that without the aluminium coating yet mylar film without the reflective coating has little warming effect. Sure the body warms up because it continuously generates heat. In exactly the same way as the surface of Earth warms up because it receives energy from the sun in the visible portion of the spectrum. The blanket reduces energy loss by reflecting the radiated portion back to the body instead of it radiating away to the environment.

        10

        • #
          MaryFJohnston

          Hi Michael

          Many of us on this blogg have some considerable understanding of heat movement mechanisms and possibly are not too sure about Radiation being the only heat loss mechanism in play here.

          Can you give us a breakdown of the amount of heat “saved” by each of the following mechanisms in the blanket.

          1. Diffusion
          2. Conduction
          3. Radiation

          If you can’t do that I will start to suspect you learned your “thermo’ on a Warmer web site.

          The blanket reduces energy loss by reflecting the radiated portion back to the body instead of it radiating away to the environment.

          10

          • #
            michael hammer

            The mylar to all intents is a solid impenetrable barrier, Its far thicker and far more impenetrable than say glad wrap. If you think glad wrap is significantly permeable try blowing through it (no I am not suggesting suffocation – just hold some against you mouth for a moment and try blowing through it). I very much doubt if you can get any significant gas through it at all and remember the mylar is far thicker and far far more impermeable.

            The aluminium layer is extremely thin – of the order of nanometers it does nothing to impede conduction or for that matter diffusion (its far more permeable than the mylar). The only difference between an aluminised mylar layer and a non aluminised mylar layer is the impact on radiation. If you want hard numerical values clearly thats a lot of work to caclulate and I don’t think it is reasonable to expect me to devote massive amounts of time to something you will probably dismiss out of hand anyway.

            Its probably pointless but here are some other examples of the same phenomenon. Kids sometimes make solar furnaces (at least they did in my day) using a curved shaving mirror to focus the suns rays. Its easy to get the focussed spot hot enough to ignite paper yet the mirror itself is cold. Hence the cold mirror is reflecting and thus sendng energy to a far far hotter piece of paper. What is the difference between that and a cold reflective mylar blanket reflecting energy to a warm body?

            There are companies which provide reflective window coatings. Their advertising states the coatings keep the house cooler in summer and warmer in winter by reducing heat loss through the windows. How does the coating reduce heat loss, surely the glass does not allow diffusion and do you really think the very thin coating appreciably changes conduction through 3-4 mm of glass. The only thing it can affect is radiative loss. How does it do that? (or are all these suppliers liars?)

            An experiment to do, on a cold night, stand near a wall inside the house and note how hot or cold you feel. Now stand infront of an uncovered closed window. Do you feel warmer or colder? I think you will find you feel colder and if so why would that be? Does the glass allow more diffusion than the plaster? Since you are not touching the window any conducted loss has to go through a layer of air whch is far more insulating than the glass or the wall so there is no significant change in conductive loss.

            Mary, at some point I can only give you the same answer I just gave Bananabender. If your mindset is so rigid that you wont consider any examples, wont make an attempt to think through an alternative point of view to see if it explains reality well then there is no way we can move to a consensus and further debate is pointless. I have only one suggestion left. Look very closely at real world situations – not just those related to global warming and GHG but non controversial everyday situations all around you – and ask yourself how your perception of how things work explains what you are seeing. Try to be objective and don’t gloss lightly over the tricky bits that dont quite fit. If you find that your viewpoint explains everything to your satisfaction then no further debate between us has any point. If on the other hand you find there are things you can’t quite explain then maybe consider whether what I have been saying explains these things better. If that sound pompous I am sorry but its the best suggestion have. I should add that I do a great deal of that myself and I find it very enjoyable. I also find that the viewpoint of the slayers does not explain real world situations to me in any even half reasonable way which is ultimately why I reject it.

            10

    • #
      MaryFJohnston

      Hi Michael

      Warmed gases at the human skin level can penetrate the double layer of mylar weave by diffusion.

      The aluminium layer prevents diffusion by acting as a solid barrier and holds the energised gas close to the skin.

      As the human body (including the head gives off about 25watts it is no problem to increase the temperature of the trapped gas.

      The gas cannot diffuse through the aluminum layer but there can be some conduction.

      The aluminium would conduct heat and reduce diffusion of gas.

      The most important mechanism would appear to be the prevention of diffusion by the Al barrier.

      The Al is a physical barrier not a mirror.

      10

      • #
        Mike W

        Much obliged for the answers Mary/Cohenite.

        That would seem to suggest that “back-radiation” is a pretty misleading description of the warming effect of GHG’s?

        Or, should that concept be taken to mean that the whole of atmosphere (GHG’s and non GHG’s) radiates as a result of the new (warmer) kinetic temperature. Either way, it seems like a clumsy and not very useful idea (but one that seems to have a lot of traction in lay presentations).

        10

        • #
          MaryFJohnston

          Mike W

          “”“back-radiation” is a pretty misleading description of the warming effect of GHG’s?””

          You’ve said it so well and it only took 5 lines!

          The warmer concept that radiation is the only energy transfer mechanism is part of their “ignorance is Bliss” scientific system that has kept Global Warming going for so long.

          10

      • #
        michael hammer

        Mary its not a mylar weave, its a mylar film and if there si any diffusion of gases the rate is so low as to be insignificant. The aluminium layer is so thin that it would have little if any effect on diffusion, its just an evaporated layer. The mylar without the aluminium is just as effective at stopping convection but far less effectve at warming.

        10

    • #
      bananabender

      Space blankets exist and are readily available. They are commonly used in rescue missions to warm people who are at danger of hypothermia. They consist of a double layer of very thin mylar film with an evaporated layer of aluminium between the 2 layers which acts like a mirror. These blankets work extremely effectively yet the same double layer of mylar without the evaporated aluminium between the layers has very little warming effect. The space blanket is still very effective at warming even when it is far colder than the person it is warming, a typical situation in say a snow rescue mission. These are facts that can be easily verified – just go out, buy such a space blanket and try it. Please explain according to your world view how these blanket work? In case anyone thinks I should first answer this issue myself, I have done so in my previous post on the second law of thermodynamics at this site

      Ah…space blankets. The (second) last refuge of the Greenhouse Effect scoundrel. [Igloos being the last refuge.]

      Space “blankets” work by trapping an insulating a layer of warm moist air close to the body. This reduces convection and wind chill. They do not back-radiate energy to the body.

      The silver side of a properly used space blankets faces out not in.

      The silvering on a space blanket is designed to:

      a) reduce radiative losses from the outside surface of the blanket in still air.

      b) reflect sunlight away when used as a sunshade in hot climates.

      In windy conditions space blankets are no more effective than ordinary cloth.

      http://windowoutdoors.com/WindowOutdoors/Staying%20Warm%20-%20Interesting%20Observations%20on%20Heat%20Loss.htm

      I’m looking forward to your fascinating articles on:

      a) How igloos warm Inuits by back-radiation.

      b) How making a “teepee” of logs causes the fire to burn hotter due to back-radiation.

      c) Why the glass makes a light bulb filament glow brighter by back-radiation.

      d) How to cook a frozen chicken by wrapping it in foil and leaving it on the fridge. (back radiation again!)

      e) Why the dark side of the Moon is so cold (hint no Greenhouse Gases).

      10

      • #
        Winston

        Also of note with space blankets- the human body alters it’s internal metabolic rate and homeostatic mechanisms to promote maintenance of body temperature, something an inanimate object doesn’t do. Drape a space blanket over a cold, inanimate object and I don’t believe you would get a warm object at all. I think that’s a logical fallacy, sorry Mike. All it does is restrict convection of a layer of air heated by body temperature- no relationship to how an atmosphere works, just like a greenhouse bears no relation to an atmosphere either, IMO.

        10

      • #

        e) Why the dark side of the Moon is so cold (hint no Greenhouse Gases).

        Hint – faces away from the sun for 2 weeks in every 4.

        10

        • #
          bananabender

          I was being facetious.

          The obvious reason is that the dark side of the Moon is not exposed to sunlight.

          10

  • #
    Truthseeker

    Michael, I understand that you are in no way a proponent of CAGW or even AGW, but I recommend that you read “Slaying the Sky Dragon” that has a number of authors that go through the greenhouse gas theory in detail and from a number of angles including using the whole solar system and not just Venus. It has sections looking that the politics behind the CAGW movement, but it the bulk of the book relates to the science and I could follow most of it which should mean that you will have no trouble following the arguments.

    20

  • #

    Mr or Mrs Bananabender, I won’t have guest authors harassed with spurious overdone aggression about non-points. (eg, the non-existent “argument from authority”). Stick to the science, and stop the insults. If there is a good scientific explanation, say it. Drop the bravado, and unsubstantiated triumphalism, and be polite. Our understanding of science is not advanced by bluster.

    Anonymous people who need snipping are too much work.

    11

  • #
    mkelly

    http://wattsupwiththat.com/2010/05/06/hyperventilating-on-venus/

    The above is a link to Steve Goddard’s report on Venus. Within that is a link to Lobus Motl’s comments on Venus.

    Both, like myself, state that pressure is the dominate factor in Venus’ temperature. Even Mr. Hammer admits that pressure has an effect by stating the it broadens out the absorbtion range.

    What would earth’s temperature be if we had the same per cent CO2 but our pressure was 92 times higher? Why isn’t Mars temperaure higher with the roughly the same per cent CO2 as Venus. hint: pressure.

    10

  • #
    MaryFJohnston

    Hi Michael

    you write in reference to the spectral abs diag:

    “Note the big bite out of the spectrum at around 660 wavenumbers and the smaller bite at around 1000 wavenumbers. The former is at the CO2 absorption line and the latter at the Ozone (O3) absorption line. ”

    Could you please let us know where water sits on this diagram?

    That would be a very useful contribution.

    10

    • #

      I hope Michael won’t mind if I reply. Water vapour has an increasing absorption from around 1000 down to 400 (10 μm to 25 μm on the upper scale) – the effect is seen as an increasing gap between the curve and the 300°K Planck curve. The other band is from about 1250 (edge of the “atmospheric window”) to the RHS at 1600 (~6 μm) – it actually extends to around 4.5 μm, off the RH edge, again seen as a significant (in this case more than 50%) lowering of the curve away from the 300°K Planck curve.

      The first band overlays the main CO2 band centred around 680 – both water vapour and CO2 absorb here, and there’s a lot more wv than CO2 in the atmosphere (60-80 times more), though its absorption is lower than CO2 here. The second band overlays both methane (at 1300) and the main N2O (nitrous oxide) band which is close to that. Because these bands overlap, water vapour “masks” absorption by these GHGs, especially methane and N2O.

      There’s been a lot of disagreement about this masking effect and the extent of its dilution of the effect of the other GHGs. Just another aspect of the”settled science”.

      See here (Wikipedia) for a general diagram which should help.

      10

      • #
        Truthseeker

        MostlyHarmless, let me point to you to this article that shows that the combination of water vapour and CO2 actually decrease atmospheric temperature, not increasing it.

        10

        • #

          I’m tending to that view myself. In my response to MaryFJohnston I was clarifying the physics, not stating an opinion. Let me make it plain – if the science (in this case the physics) is convincing, and has good evidence to support it, then I must support it. The evidence for absorption and emission by GHGs is clear, and so I accept it. However, the consequences of that effect are not clear and there’s conflicting evidence. For example, I don’t accept that climate sensitivity to CO2 is known with any certainty – I believe it to be low, on the available evidence.

          So far I’ve expressed few of what I’d consider opinions on climate and related matters here – I’ve tried to restrict my comments to assessments of the evidence. I’m not referring to you directly, but I’d prefer it if respondents replied directly to what I write, and not what they think I meant, or what they think I believe.

          10

        • #
          Julian Braggins

          A great article Truthseeker, and I posted Nasif Nahle’s conclusions to it a little above this. I think it changes the whole argument about GHG’s and any greenhouse effect i.e. none, and possibly a small cooling.

          10

  • #
    Bryan

    michael hammer

    Your posts imply that others do not share your understanding of the radiative properties of CO2 and H2O.

    What they don’t share is an exaggerated version of their overall radiative effects.

    A hot surface radiates to a colder surface and the cold surface radiates to the warmer surface.
    For all wavelengths the hot surface radiates at a higher intensity.
    Further the hot surface will emit higher frequencies that the colder surface cannot match.
    Spontaneous Thermodynamic Heat transfer (which is the net energy transfer) is always from the higher to the lower temperature.

    Further do you accept that radiation does not play any part in the dry adiabatic lapse rate?

    10

    • #
      michael hammer

      Hi Bryan; with regard to your comments about hot and cold radiating surfaces I 100% agree (its exactly what I have been saying) With regard to your comments that I am implying that others do not share my understanding of radiative properties, clearly some do not. For example, those who claim that down welling radiation contravenes the second law of thermodynamics because a cold object cannot radiate to a warmer one. In fact I am not trying to imply anything, I am simply stating the truth as I see it in a debating situation where others are disagreeing and in some cases strongly disagreeing. For those who do share my viewpoint maybe it looks as though I am preaching to the converted. I do not apologise for commenting the way I am but I am sorry if you interpret it in a negative way.

      With regard to your last point, I fully accept that the dry adiabatic lapse rate is set by convective processes not radiative ones. But I have a question for you, its a hypothetical one posed by others in this thread. What happens if radiative processes impose a lapse rate below the dry adiabatic lapse rate, does convection stop. How could that come about? I think it could if the atmosphere was entirely free of any GHG (no not a realistic scenario purely speculative). It seems to me that only if radiative processes imply a lapse rate above the dry adiabatic lapse rate do convective processes start, and once started they establish the lapse rate and contributes to energy loss to space by transporting heat up into the atmosphere. That is of course the situation on Earth but is convection an inevitable condition in all cases?

      11

      • #
        Bryan

        Thanks for the reply michael

        You say

        ……”With regard to your last point, I fully accept that the dry adiabatic lapse rate is set by convective processes not radiative ones. But I have a question for you, its a hypothetical one posed by others in this thread. What happens if radiative processes impose a lapse rate below the dry adiabatic lapse rate, does convection stop.”…..

        The formula for the DALR is derived for the no convection condition.
        The DA lapse rate derived from thermodynamics is for still air.
        All that is required is heated base and cooling top and diffusion(collision transfer) can transfer heat along the gradient.

        The hydrostatic condition is also satisfied by constant vertical speed parcel of air.
        The meteorologists call this the neutral atmosphere and this condition is relatively stable.

        More usual however is the turbulent convective flow which will transfer heat at a faster rate.
        Meteorologists seem to place more emphasis on the latent heat of vaporisation of water for the departure shown from DALR for the environmental or moist lapse rate than a radiative explanation.

        Even during the DALR the CO2 must still be radiating but I think that its effects are included in the bulk heat transfer quantities such as Cp.

        I think that the Earth system has several storage systems.
        An analogy would be a tuned LC parallel circuit.
        As you know there is a much larger flow of energy between L and C than the make up energy from the supply(Sun).

        Take a parcel of air rising adiabatically in a gravitational field.
        The parcel will do work on the surrounding atmosphere so internal energy will be reduced and the temperature falls.
        At the TOA radiative losses cause the parcel to be denser than surroundings so the parcel now falls.
        The gravitational force effects now compress the parcel increasing internal energy hence raising the temperature of the parcel.
        Back at the Earth surface the parcel will have recovered most of its original temperature.
        This is no surprise because Gravity is a conservative force.
        This is just one example of an Earth energy storage system.
        Evaporation and condensation of water is another and so on.

        IPCC proponents seem to grant the “greenhouse gases” with the full credit for the 33K difference between the Earth surface temperature(288K) and the effective emission temperature of 255K

        10

        • #
          michael hammer

          Hi Bryan; I understand you point but you state the only requirement is for a heated base and a cooled top. The hypothetical question posed was an atmosphere without any green house gases and in that case it seems to me there is no cooled top. How does the top of the atmosphere lose energy. Cant be convection – already at the top. Cant be by conduction because the gas below is warmer (ie: lapse rate) and if there is no GHG it cant be by radiation which leaves….. nothing. That why I am wondering whether in such a bizarre situation there might be no convective circulation at all.

          10

          • #
            Bryan

            Thanks for the reply Michael.

            As you know all real objects radiate to some extent above zero K.
            But for a “what if” scenario with CO2 and H2O say having no IR response the consequences could be speculated on.

            The surface incident solar radiation would if anything increase.

            With zenith solar effects at equatorial region producing temperatures of around 80C then evaporation and condensation would still produce clouds and rain drops.
            So continuum radiation from liquid surfaces would be possible for seas clouds and rain as well as from solid surfaces.

            The exact outcome balance is impossible to determine.

            I think you are right however that the radiative properties of CO2 and H2O are very important at the TOA.

            Its a pity that the debate does not focus more on the consequences of IR active gases at this altitude rather than backradiation.
            The net radiative effect after all is all that matters.

            In my opinion the alarmist tendency have every reason to exaggerate the magnitude of backradiation.
            Perhaps that is the reason for the history of calibration problems for the pyrgeometer.

            There are various versions of the greenhouse effect.
            Some like Nullius in Verba explicitly think that there is a giant atmospheric heat pump effect in the sky forcing heat against the temperature gradient.
            However not enough detail is given to determine if he has a point.
            The only parts of the greenhouse theory I think have some value are the TOA effects and the differential emissivities of solar and earth surface radiation

            10

          • #
            Julian Braggins

            As I replied to another who asked the same question, cooling is just as effective or more so than warming, the cooling atmosphere in contact with the night-time surface would spread to the less dense warmer air at the terminator, and warmer air from above would replace it, no?

            10

  • #
    Richard Pearson

    Reading this blog it strikes me that my experiment to test the Greenhouse Effect hypothesis mimics CO2 concentrations in the atmosphere of Venus and the Earth. I am struggling to comprehend the mathematics underpinning the second law of thermodynamics but I do have a qualitative understanding – I studied it 40 years ago doing first year physics and chemistry in my medical course. If there is a physicist or reference that can help me comprehend the actual mathematics of the 2nd law I would appreciate it – my references say even mathematicians struggle to understand it. My understanding is that the greenhouse effect theory defies the 2nd law of thermodynamics no matter what anyone says. Here’s a simple experiment you can do to prove it yourself – it is also published at http://www.galileomovement.com.au/blog/?p=25 and has been critically evaluated by the whole Slayers team, Joseph Postma, and Nasif Nahle – they liked it. Tim Lambert thought the boxes too small and pointed to the experiment done by Mythbusters (nothing like this one).

    Kitchen Chemistry and the Greenhouse Effect Theory

    The greenhouse effect theory as it relates to Climate Change science can be stated as a process whereby back scattered radiative transfer of infrared heat from atmospheric gases and vapors that have thermal capacity causes additional surface temperature warming of the earth above that from the effects of insolation from the sun alone. The effect is said to be a ‘positive forcing’ factor for the temperature of the earth.
    Insolation from the sun heats the earth – over time an equilibrium state exists with a balance between incoming solar radiation and outgoing radiation. The earth radiates heat back into the atmosphere and space in the form of infrared radiation. Infra red radiation is invisible unless the object emitting it is very hot – red or white hot. It is a long wave form of radiation just outside the visible light spectrum.
    John Tyndall’s experiments in 1861 demonstrated carbon dioxide is a gas with thermal capacity –it can absorb radiant heat. In 1896 the original ideas about an atmospheric greenhouse effect and carbon dioxide (then called carbonic acid) came from Svente Arrhenius in Sweden. These ideas found little scientific support. Revival of the ideas came from the Swedish climatologist Dr Bert Bolin in the early 1970’s. Activists including Al Gore and the UN IPCC that was founded in 1988 to demonstrate human influence on global warming have reinvigorated interest in the Greenhouse Effect theory.
    An experiment devised to utilise the ‘greenhouse gas’ properties of carbon dioxide and demonstrate the Greenhouse Effect theory is described below.
    The initial experiment was done at Tewantin, Queensland, Australia on 28 August 2011 from 1.15 pm to 2.30 pm. Temperature was 25 degress Celsius in the boxes before they were sealed and exposed to insolation. The barometric pressure 1014 hpa. The humidity was 66 %.
    Two polystyrene foam 5 kg fish cooler boxes were painted inside with matt black paint. The black surface absorbs heat from sunlight and causes infrared radiative transfer of heat within the box. Heat energy is maintained in the box by the plastic wrap creating a ‘true’ greenhouse effect whereby convection is prevented from moving the hot air or CO2 away from its contained space. The heat energy is measured by monitoring the temperature. Infrared radiation can pass freely through the plastic wrap surface of the boxes.
    Type K thermocouples were inserted into the boxes with the sensor tip dangling in the mid space of the boxes.
    One box was sealed up with plastic wrap (Glad Wrap ®) and contained air.
    The second box was filled with carbon dioxide before being sealed with plastic wrap (Glad Wrap ®). Carbon dioxide was generated using kitchen ingredients baking soda and white vinegar to achieve the following chemical reaction with production of CO2.
    Na HCO3 + HC2 H3O2 → Na C2H3O2 + H2O + CO2
    Sodium bicarbonate + acetic acid → Sodium acetate + water + carbon dioxide.
    6 heaped teaspoons baking soda + 300mls vinegar → carbon dioxide to fill the box (12.3L)
    The carbon dioxide was produced in a bowl in the box in a closed room with little air movement. The bowl was then gently removed from the box. A burning taper (long match) was used to confirm the presence of CO2 by extinguishing the flame just below the rim of the box. CO2 is retained in the box since it is heavier than air. The box was then sealed with plastic wrap (Glad Wrap ®).
    Both boxes were then placed outside and tilted towards the sun – apart from one containing air and the other containing carbon dioxide the boxes could be considered identical in makeup and placement.
    The initial temperature before placement outside was 25 degrees Celsius. When placed outside the temperature in both boxes rose quickly and after a few minutes the temperatures were around 40 C. The box containing air heated more quickly for around 20 minutes – being about 4 C hotter than the box with the CO2.
    After an hour both boxes were observed to have the same temperature – the temperature fluctuated between 65 C and 70 C with small clouds coming over at times. The temperature was assumed to have reached its equilibrium state and the experiment was terminated.
    To confirm that the CO2 was indeed still in one of the boxes –the box was taken inside a closed room. Its plastic wrap was slit open and a lighted taper was introduced into the box by an independent observer on my invitation. The match was extinguished just below the rim of the box confirming the continued containment of CO2.
    Result – this preliminary experiment shows there is no such thing as back scattered infrared radiative transfer causing additional temperature rise above that from insolation by solar radiation. The Greenhouse Effect theory is not confirmed by this experiment and may be disproved by it.
    Further experiments are planned with attention to accurate recording of temperature changes. The boxes will be dehydrated using calcium chloride as a dessicant to produce dry air. The air can also be ‘sweetened’ by Tyndall’s method of lining a surface with glycerin to remove invisible particulates like pollens and bacteria from the air.
    Comment – the independent observer mentioned above is my young adult daughter. “But how can an experiment like this disprove a theory that hundreds of climate scientists around the world say is true – surely they know far more than you do”, she asked. “That my darling is science”, say I.

    Richard Pearson.

    10

    • #

      Result – this preliminary experiment shows there is no such thing as back scattered infrared radiative transfer causing additional temperature rise above that from insolation by solar radiation

      A bold assertion indeed, perhaps too bold.

      The box containing air heated more quickly for around 20 minutes – being about 4 C hotter than the box with the CO2.

      Did you consider why? In addition to the box interior, the CO2 was radiating out through the cover, hence the slower rate of heating. Ultimately, both boxes reached thermal equilibrium, when radiation out equalled radiation in. Not surprisingly both air and CO2 reached the same temperature. I’d argue that your closed boxes don’t model the atmosphere, where conduction/convection to higher, colder levels occurs. What you didn’t measure was the temperature of the interior surfaces of the boxes – that may have been quite revealing.

      10

  • #
    Bob_FJ

    Michael Hammer,
    Below your graph in the article you wrote:

    Note the big bite out of the spectrum at around 660 wavenumbers and the smaller bite at around 1000 wavenumbers. The former is at the CO2 absorption line and the latter at the Ozone (O3) absorption line. Those two bites represent energy that is not being radiated to space that would be if there was no atmosphere. In short it is the signature of a green house gas reducing Earth’s radiation to space at the green house wavelengths. I invite those who disagree to give an alternate explanation for what is causing these notches.

    But is it not true that there is considerable overlap in the absorption bands of H2O vapour and CO2, and that there is plenty of water vapour in the tropical Pacific, where the data was measured? And clouds? (and water vapour is generally agreed to be by far the greater absorber)
    Also, according to Trenberth in his energy balance diagram, about 60% of the energy leaving the surface is via thermals and evapotranspiration. (AKA collectively by some as convection). This results in warming of the air-column, and thus additional radiative effects, that ultimately escape to space. This is a majority of the upward infrared, but it has no labels attached as to where it came from does it? Is not attribution a tad tricky?

    Here is some spectral stuff….. don’t know how good it is:
    http://www.espere.net/Unitedkingdom/water/uk_watervapour.html
    http://en.wikipedia.org/wiki/File:Atmospheric_Transmission.png

    10

    • #

      See my reply to MaryFJohnston at #72.1 re. water vapour overlap ( I also linked to that Wiki diagram!)

      10

    • #
      Truthseeker

      Bob_FJ,

      Here is something that will give you another take on the combination of water vapour and CO2 and how it decreases atmospheric temperature, not increase it.

      Also, for a complete and robust debunking of Trenberth’s energy diagram (and the “greenhouse” concept in general), you can look at this.

      10

      • #

        I wouldn’t accept a complete debunking of K&T’s energy budget diagram, but can see a lot that’s wrong with it in detail. For example there’s no albedo effect included for absorption of “back radiation” at the surface, and I’m sure that their figure for surface radiation exiting through the”atmospheric window” is too low. These and other factors don’t disprove greenhouse theory, but they all tend to reduce the value of climate sensitivity to CO2. In K&T and elsewhere, I detect a tendency to err on the high side for factors affecting climate sensitivity, and I don’t like it – it’s not scientific (in the true sense of the word).

        10

  • #
    Mike W

    I re-read an earlier comment of mine and realised I could have given the impression that I think the atmosphere radiates smoothly, like a blackbody, errr, at least I don’t think I think that. Obviously, gasses, even complex ones, can only emit over specific, discrete wavelengths. I think you can see what I’m getting at though:

    If absorptivity = emissivity, how do we reconcile the energy missing from the notches?

    I am fairly sure that just because the system absorbs at 13um it does not mean the system has to emit an equivalent amount of energy at 13um, is this correct?

    Sooo, ultimately, is the missing energy in the notches redistributed and radiated elsewhere in the spectrum, orrrrr, is it
    just “delayed” (although I can’t see how such a delay would manifest as the notch in Nimbus’ observations).

    10

    • #
      Mike W

      Gah! You know, once new thermodynamic equilibrium is reached, then absorptivity should equal/balance emissivity.

      10

    • #
      MaryFJohnston

      Good pint Mike W

      “I am fairly sure that just because the system absorbs at 13um it does not mean the system has to emit an equivalent amount of energy at 13um, is this correct?”

      All energy is degraded over time:

      Atm UV and Visible spectrums in degraded in the turn around at surface and re emitted as weaker IR.

      Even IR can be degraded to weak IR.

      10

  • #

    Truthseeker #45.1, you are a gentleman, I will try to explain very complex subjects, on Venus, that needs 20 pages; in much less. Same problem; in my book on some subject is 25pages – on the website in 7 paragraphs… If you read paragraph one and seven – skip the rest – then blame me for not connecting the dots…? I simplify so much, everybody interested, to be able to understand. Manipulators as Hammer complicate for reasons

    VENUS IS WARMER FOR 2 REASONS, CO2 IS NOT THE OFFENDER Comparing wind cooling on earth with cooling in empty space is wrong. O+N in the wind have friction and resistance. Cooling by the empty space coldness is much more efficient. I.e. that coldness penetrates much more efficiently; penetrates without resistance or friction, different apple.
    If it wasn’t for that, earth would have boiled in days.

    Repairing in the shadow of ‘’Mir’’ they had to pump lots of heat – otherwise human becomes rock solid in minutes, because of that cold / efficient penetration. 2] NASA don’t tell the public that: on the moon, where is night, life is not sustainable – nearly 14 earth’s days of night on the moon– that coldness goes to below – 160⁰C. Otherwise will not get taxpayer’s support to go back. 3] that coldness penetrates on earth in the tropics, on top of some mountains is ice. 4] same coldness; on Mercury, on the dark side temperature gets to minus -200⁰C, you can liquefy nitrogen in a bucket. Even though is so close to the sun + 58% on Mercury’s surface is sunshine – but on the other side is colder than on distance as Saturn. (Mercury doesn’t spin around itself / same as our moon) Back to Venus:

    On Venus, her day is longer than her year = lazy. On earth, the velocity in fast spinning + forward is the most efficient cooling – trusting into that coldness (without resistance) If earth’s day become to 30h, instead of 24h – would have decreased cooling and boiled (or year to few days longer) Venus makes one spin in 8 earth’s months = that is the first reason of being hotter, not using enough of that coldness.

    Experiment: stick 2 shirts on individual sticks / strings. Deep them in boiling water. Then spin one around yourself very slowly, the other very fast. When is fast + forward – goes into new coldness. Slow spinning represents Venus. But you have to know about the efficiency in penetration of that unlimited coldness. That coldness takes all the tremendous heat from the earth, deposited by the sun every day + some thermal heat. Extra heat in earth’s atmosphere is not accumulative, thanks to efficiency of that coldness; that’s the first reason why Venus is hot.

    Reason No 2: in a dark room, put a football, and 2 candles 2m away. The candles to be spaced as much as the diameter of the ball = will deposit light on 50% of the ball. (then, because Venus is much closer to the sun) spread the candles further apart = they will deposit light on 55% of the ball. Therefore, will not be sunlight on Venus at any time ONLY on 45% = 10% extra surface on Venus is sunshine, than on earth + plus the proximity to the sun = stronger radiation = those factors are the second reason Venus is warmer. (geometry is reliable science – reason Mr. Hammer is not hitting the nail on the head, but beating around the bush / comlicating. Like starving man trying to s### = lots of hot air, but nothing solid. CO2, not guilty!

    I will disclose another half reason – which the Warmist can use it against us here on earth – but we will blame you, Truthseeker. Here is the self incrimination: look at the sunrise in the morning. From your eyes, up for 50km, the sunlight goes trough earth’s atmosphere – big part goes out again, not on Venus. Because CO2 intercepts all of that sunlight. I call it; penetrates trough earth’s crown and part of the sunlight goes out, not on Venus. Don’t take me wrong, extra CO2 will not increase the warmth on the earth – because of my formula: EH=AE=ECI If you are not familiar of my formula = you are not a Skeptic

    Now Truthseeker, use your grammar / communication skills – elaborate correctly on all those factors, in few paragraphs. Then, Jo should be a good sport – should use it as ‘’ right to reply story’’ Because Julia is introducing the tax – simultaneously Hammer is misleading the people with 140y old, HALF story. See how difficult it is to present lots of complex issues in few paragraphs for website.

    Hammer experimenting CO2 day heat absorption, to insinuate it for the phony GLOBAL warming; but not night coldness absorption… Is same as if Ross Garnaut was calculating the cost of damages the moon would have made; IF falls on Barrier Reef. Loss of tourism + damages to buildings +++++, and he as economist would come up with correct billions of dollars – you start questioning him about the number of billions – he will win. But, will the moon fall? There is less chance CO2 to produce GLOBAL warming, than the moon to fall on the reef = Hammer did his job, Ross didn’t his (Ross Garnaut is the yardstick for measuring dishonesty) Truthseeker, take my chalenge

    10

    • #
      Truthseeker

      “Truthseeker #45.1, you are a gentleman” – thank you.

      I will see if I can take up the challenge of explaining your post (#77) using clearer language, but there are some statements that I find mystifying simply because of the way that they are expressed.

      “Comparing wind cooling on earth with cooling in empty space is wrong. O+N in the wind have friction and resistance. Cooling by the empty space coldness is much more efficient. I.e. that coldness penetrates much more efficiently; penetrates without resistance or friction, different apple. If it wasn’t for that, earth would have boiled in days.”

      OK, I had difficulty initially with “O+N” (the correct jargon of Oxygen in the atmosphere is O2 and for Nitrogen N2) in the above statement, but what I think you are saying is the following;

      Comparing cooling on Earth with cooling in empty space is like comparing apples and oranges. Things are cooled in empty space much more efficiently because there is no friction or resistance whereas Oxygen and Nitrogen in the wind has both. It is the spinning of the Earth in its orbit (which generates wind) that stops the seas from boiling.

      My reply is that I did not understand that you were talking about the planetary bodies and not including the atmosphere. I took the analogy to mean you were referring to the planet (including the atmosphere) movements though space. Hence my reference to a vacuum. Now that I have a better understanding of what you are trying to communicate, I will agree with this statement. (As an aside, your “earth boiling” statement is an example of not connecting enough dots for the reader).

      “Repairing in the shadow of … same as our moon)” is a series of statements where the phrasing is a little unclear. Let me rephrase them for you.

      When doing repairs in the shadow of space station “Mir” they had to pump lots of heat otherwise the astronauts would have frozen solid because of the extremely efficient heat loss to space. NASA do not tell the public that night time on the moon the temperature is below -160C because they want taxpayer support for future missions. There is coldness the penetrates Earth in the tropics as there are mountains with permanent snow. On Mercury the dark side gets to -200C which is cold enough to liquify nitrogen in a bucket despite it being close to the sun. This is because is does not spin just like our moon.

      I don’t disagree with any of these statements although I would have said it is the high altitude and therefore lower air pressure that is causing the heat loss for the tropical mountain tops (same mechanic different wording).

      “Back to Venus: … enough of that coldness” OK this is where it gets difficult because of you use of “trusting into that coldness (without resistance)” which I take you to mean the vacuum of space, but your previous comment was telling me I could not compare the effects of a vacuum to that of an atmosphere on a planet. This is not the only problem with trying to decipher your message. I get from this paragraph that since Venus spins slower (day is longer) it will get hotter and that Earth’s relatively fast spinning orbit keeps it cooler. OK so far. The next statement about if Earth’s day was 30 hours long it would have boiled is a bit of an exaggeration but I do see that you are saying that the slower orbit means the Earth would be hotter. OK again. Venus making one spin every 8 months is the first reason for the warmth that it has. OK I get that, however “not using enough of that coldness” is simply mystifying. Cold is the lack of heat, so “using” it is a bit of a stretch for easy understanding.

      With the experiment using two shirts and boiling water, you are giving a practical example to emphasise your point and it is a good example. Lets move on.

      The next experiment with the football and 2 candles is another good example to use and it shows what you mean about the angle of the edge of sun and that it shines on a much larger portion of Venus that it does for the Earth. Combining this with the reduced distance that Venus is from the Sun also emphasises the point and I can give this a tick for understandability. The next analogy with regard to Mr Hammer is probably uncalled for. Lets move on.

      You can blame anyone you like, including me if that makes you feel better. The term “self incrimination” does not mean what you seem to think it means. In fact this paragraph is very hard to follow because in one breath you say that I can see out on Earth because the sunlight goes through atmosphere, but not on Venus because of the CO2 and then you say that CO2 does not warm because of a formula. In fact it is the opaque cloud cover that stops you doing the same sunrise viewing on Venus. The clouds on Venus may be made up of CO2 (like 96% of the Venusian atmosphere) but it is the fact that they are clouds that is blocking the sunlight not because they are CO2. I am a skeptic of CAGW and I have seen your formula but that correlation is not causation. The only problem I have with your formula is that you use “Atmosphere Expends” when I think you mean “Atmosphere Expands”. I also think it would be better to use an arrow (->) rather than an equals sign to show the causation from extra heat causing atmosphere expansion causing extra heat loss. The term “extra coldness intercepts” just leaves me cold.

      I suggest that you should contact Memoryvault and take him up on his generous offer to re-write your website and posts so that they are more understandable to more people. He is a professional technical writer and that definitely trumps my occasional training / user manual that I write as part of my job.

      10

    • #
      Truthseeker

      Stefan, if you want an example of how to communicate scientific ideas effectively then I suggest you go this site. I think you will agree with much of what he says (as do I) and it also complete debunks the “greenhouse” effect the Michael Hammer and others (on both sides of the CAGW debate) are trying to promote.

      10

  • #
    KevinK

    Mike Hammer wrote (post 64.3);

    “Consider the case of a space blanket which is simply 2 thin mylar sheets bonded together with an extremely thin layer of aluminum (sic) evaporated onto one of the inner layers of the two sheets. Its physical and thus thermal mass is utterly miniscule yet it still has the (sic) power to warm someone significantly.”

    Now I have indeed considered this case of the magical space blanket since in my business we wrap portions of Earth orbiting satellites with the same material to control the temperatures of the satellite.

    Firstly; the space blanket has NO ability to “warm someone significantly”, the power to warm the person comes from the metabolism of the person. The space blanket (by reflecting virtually all radiated energy across the entire blackbody spectrum of the emitting skin) slows the flow of heat away from the body thusly making the heat from the metabolism last longer.

    Secondly; since the space blanket reflects virtually all of the radiation it is a very poor analogy to”GHGs” which only absorb / remit a very small amount of the energy in a very narrow spectral band.

    Thirdly; regardless of the physical and thermal mass of the space blanket the thermal capacity of the ”GHGs” in the atmosphere are insufficient to “force” the surface of the Earth into thermal equilibrium with said gases.

    Fourthly; as another poster mentioned if you cover a rock with a space blanket it does not magically become a warmer rock. It just becomes a rock that is cooling more slowly. The thermal blanket does not slow the cooling enough that the rock slowly warms up.

    Lastly; in the analysis of thermal energy flows through a system it is insufficient to simply identify that some of it flows “backwards” (therefore things are warmer), it is also necessary to understand the speed at which it flows and the thermal capacity of both the source and destination of the energy before determining if a “higher equilibrium” temperature can occur.

    I am not disputing that at some level of “Greenhouse Effect” from gases in the atmosphere a “Higher Equilibrium” temperature could exist. I am simply pointing out that the level of “Greenhouse Effect” from the currently observed gasses in the atmosphere of the Earth are not capable of this feat. Once the gases can slow the flow of heat so that some is “leftover” after each 24 hour cycle of energy input then a “Higher Equilibrium” temperature could occur.

    Right now the “GHGs” are only capable of slowing the flow of heat by at most a few seconds (more likely tens of milliseconds or less), so they only have to increase the delay by 85,999 seconds and then the warming should kick in as predicted.

    (sarc on) Given the current rate of increase in “GHGs” this should happen sometime around Tuesday, June 7, 1007211. It should happen before noon, but my climate model has not yet been peer reviewed, so it could happen in the afternoon, or possibly the next day. (sarc off)

    Congratulations on your many patents.
    Cheers, Kevin.

    10

  • #
    Mike W

    To maintain equilbrium the planet as a whole has to emit slightly more energy at other wavelengths to make up for the reduced energy radiated at 14.7 microns and to do that it has to get very slightly hotter.

    Snap! Thank God for that, I’m actually very comfortable with that idea. Bit embarassed about those emission wavelengths I pulled out of my a$$, thanks for being a sport about it.

    Michael, presumably if we compared the Nimbus observations at altitude with ones taken at surface level we could measure not only the notches where GHG’s have absorbed the upwelling IR, but the extra energy shuffled into the other wavelengths? Are the measurements sensitive enough to demonstrate that aspect? Just curious, seems like the ovious question to ask.

    And, thanks again.

    10

    • #
      Mike W

      So the CO2 right throughout the atmosphere is emitting at 14.7 microns

      I almost glossed over that, it’s pretty significant. As I’m sure you’ve noticed, I had been given the impression that Co2 did not do much re-emitting at any frequency until TOA. Blame Eli! Lol, I’m sure that wasn’t his intention. I am happy to take your advice that Co2 emits at 14.7um thoughout the atmosphere, and I’m sure that’s been experimentally verified, which explains why there is such focus on radiative transfer.

      I’ve always understood the overall effect on climate to be real but finite, diminishing, relatively insignificant and fairly benign; more effect than cause, and definitely not a climate master control lever. I think your own view might be quite similar? Hope that doesn’t get lost in all this nuts and bolts discussion, enjoyable as it is.

      10

  • #
    Richard C (NZ)

    OK, but why not advance the investigation of GHG heating effect on the earth?

    Only a few e.g. banabender, MostlyHarmless and Cameron H, allude to the actual heating effect of DLR on surface and subsurface material (off topic I admit). That topic is being thrashed out with brave (and learning) AGW proponent Nick here:-

    http://www.climateconversation.wordshine.co.nz/2011/09/quote-of-the-week-3/

    Siddons, Postma, Nahle and Johnson (Slayers) are well down that track (the one “consensus” climate science doesn’t go down) e.g.

    The Model Atmosphere

    http://principia-scientific.org/publications/The_Model_Atmosphere.pdf

    Observations on “Backradiation” during Nighttime and Daytime

    http://principia-scientific.org/publications/New_Concise_Experiment_on_Backradiation.pdf

    There’s a strong case at the CCG link I think and the many supporting links there that DLR from clouds and GHGs has neither heating nor evaporative effect (the planks of AGW).

    Now the Slayers are saying that what has been measured is not DLR but radiation emitted by warm air masses.

    Postma and Nahle comprehensively dismember Trenberth, Fasullo and Kiehl’s (TF&K09) “Earth’s Global Energy budget”. In particular: the estimated solar flux at the earth’s surface; the non-existent surface heating effect of DLR; and, found in respect to IR thermometers and radiometers that what is really measured when the devices are pointed towards a clear sky is radiation emitted by globules of air at high altitudes.

    This is where the debate should be IMO.

    10

    • #
      bananabender

      Thanks you.

      Quantum effects (eg “greenhouse gases”) really only occur at atomic scales. At planetary scales we are dealing with thermodynamics. The heating of the atmosphere should be studied in the context of physical chemistry and engineering not radiative physics.

      10

    • #
      michael hammer

      Ahh Richard; you say “what is really measured when the devices are pointed towards a clear sky is radiation emitted by globules of air at high altitudes.”

      EXACTLY energy emitted by the air back towards earth’s surface ie: down welling radiation. Of course it doesn’t have to be from high altitudes, low altitudes works just as well. So its not down welling radiation instead its down welling radiation.

      10

      • #
        MaryFJohnston

        And could you give us an estimate of the length of travel of each “parcel” of Downwelling radiation.

        Would it be Kilometres or perhaps metres or perhaps centimetres.

        Or smaller??

        There is a very definite temperature gradient from Earth surface (T = 23 deg C ) to the great heat sink in the Sky = Space (T = – 273.16 deg C)

        Heat follows the temperature gradient!!

        There is something very unnerving about the term “Down Welling Radiation”.

        Like Global Warming it is real but its magnitude is effectively NIL.

        10

        • #
          bananabender

          The down welling radiation should also make the water at the bottom of the oceans superheated (not just around thermal vents), the land surface hotter and the upper atmosphere much colder.

          10

        • #
          Richard C (NZ)

          Mary #80.2.1

          It’s the length of travel of a photon AFTER it crosses the surface boundary that tells us if DLR (or any radiation) has any heating effect on the surface and subsurface material.

          The medical physics term for this is “track length” and it’s the track over which the photon can transfer energy to the material. The track length of DLR in ocean is no more than about 100µm (on a good day) and only effective at about 10µm. Meanwhile, UVs track length is about 25m+ and effective at about 1m, see “The measurement and penetration of ultraviolet radiation into tropical marine water”. Infrared can only penetrate water no deeper than ~500 µm so it’s obviously SW solar that does the heating despite RealClimate’s stretch.

          10

          • #
            Richard C (NZ)

            Probably should make the distinction between IR-A and IR-B in the solar range ( about 2µm – there’s an overlap) in the DLR range.

            By convention, the divide is 4µm

            10

          • #
            Richard C (NZ)

            Arrgh!, the blog saw the “less than” and “greater than” sign as html and dropped a bunch of comment. Last comment should read:-

            Probably should make the distinction between IR-A and IR-B in the solar range (less than about 4µm) and IR-C in the DLR range (greater than about 2µm – there’s an overlap).

            By convention, the divide is 4µm

            10

          • #
            MaryFJohnston

            Thanks for the reply Richard C NZ

            Interesting summary.

            My comment above is heavily based on the improbability of the “Downwelling ” schema put up by some “scientists’ and the more likely truth that the downwelling, while real, will only penetrate a very short distance towards the Earth.

            In effect , the Downwelling thing is a non event and a warmer distraction from reality.

            I’m not familiar with the breakdown of IR into A B and DLR but from frequencies quoted it looks like going from A to C is moving from less enegetic forms of IR to most energetic, have I got that?

            10

          • #
            Richard C (NZ)

            Mary #80.2.1.2.3 you say:-

            “…from frequencies quoted it looks like going from A to C is moving from less enegetic forms of IR to most energetic, have I got that?”

            No, other way around. If you look at the table second down right hand side of the Electromagnetic spectrum, you can see that IR energy-per-photon in the solar spectrum say 1µm, is 1.24 eV but energy in the DLR range say 10µm is 124 meV.

            For the breakdown of infrared see Commonly used sub-division scheme

            10

          • #
            MaryFJohnston

            Thanks richard C NZ

            Counter-intuitive. It’s been a long time since I did physics and thermod.

            I assumed higher frequency stuff was the higher energy packet — wrong.

            Thanks

            10

        • #
          michael hammer

          Hi Mary; I can give you a general idea. 90% of radiation is absorbed in a 1 absorbance unit think layer of material. For CO2 at 14.7 microns the total atmospheric column corresponds to about 2000 absorbance units so 90% of the 14.7 micron radiation is absorbed in about 1/2000 of the atmopsheric column. The distance depends on the pressure. 1/2000 of the total atmospheric column at sea level is pretty small. AT say 8km altitude the air is less dense so the distance will be greater but still pretty small. Of course at say 11 microns in the atmospheric window there is no GHG absorption so at this wavelength there is no down welling radiation and the radiation emitted by Earth’s surface is also not absorbed and travels out to space.

          By the way in water the distance is microns which is why down welling radiation does not penetrate deeply enough to warm anything much below the surface

          10

          • #
            Richard C (NZ)

            michael hammer #80.2.1.3 you say:-

            Of course at say 11 microns in the atmospheric window there is no GHG absorption so at this wavelength there is no down welling radiation

            You better have a chat to Prof Petty because his Nauru plot does show DLR at 11µm

            10

      • #
        bananabender

        The “down welling radiation” being measured is mostly latent heat released as atmospheric water vapour condenses.

        You only need to visit a desert to see the non-existence of the Greenhouse Effect. The Sun goes down and the temperature drops from 50C to below freezing in a few hours. Why? because there is almost no water vapour in the atmosphere to warm the air via latent heat.

        10

      • #
        Richard C (NZ)

        michael hammer #80.2 you say:-

        EXACTLY energy emitted by the AIR back towards earth’s surface ie: down welling radiation. Of course it doesn’t have to be from high altitudes, low altitudes works just as well.

        [my emphasis]

        TF&K09 Fig 1 has DLR emitting from clouds and GHGs (at whatever altitude). This is the WHOLE POINT of AGW. You have in effect sided with the Slayers by saying “energy emitted by the air” and have destroyed the notion of CO2 being a player – thanks.

        You say:-

        So its not down welling radiation instead its down welling radiation.

        Huh?

        10

      • #
        MaryFJohnston

        Supplied “”For CO2 at 14.7 microns the total atmospheric column corresponds to about 2000 absorbance units.

        So based on say 10,000 metres effective atmosphere ( prob should use 20km ) get a 5m * 90% absorption of IR by CO2 (assuming water doesn’t get to it first).

        This is about what I understood from other sources ie 10m.

        10

    • #

      If I were you I’d take the Slayers assertions and arguments with a large pinch of salt (translation: scepticism). I’ve dissected some of Siddons’ posts myself. His analysis is sloppy and error-prone, and often fundamentally flawed. Johnson has a tendency to come to unsupported conclusions, such as this one you linked to over at CCG

      As a defense Prof Petty would probably argue that the only thing that matters in the end is the net flow, and that the net flow anyway is the same in both forms of the SB law, and thus that it does not really matter if DLR is reality or fiction. From scientific point of view this argument has an obvious weakness. From political point of view it may not matter if CO2 alarm is reality or fiction, but from scientific point of view it cannot be without importance.

      “From scientific point of view this argument has an obvious weakness” – but he doesn’t say what that “obvious weakness”
      actually is. He claims on his blog that scientists aren’t actually measuring radiation at all, but temperature – absolute nonsense. In one post where he develops this theme, he says that “infrared cameras” (actually radiometers) “measure frequency” – an extremely difficult thing to do for light (including IR of course). He then claims this “measurement” is input to Wien’s Displacement Law to obtain temperature, and this is then used via Stefan-Boltzmann to calculate radiation flux,

      IR detectors or sensors used in IR cameras are sensitive to the frequency of the incoming radiation, from which the emission temperature is calculated (using Wien’s displacement law) and from the emission temperature the emitted radiation is calculated (by SB).

      The problem is that Wien’s law relates three variables – I challenge anyone to obtain a result from such a law with one input value.

      Radiometers do just what it “says on the box” – measure radiation directly. They use photo-voltaic sensors which produce a measurable voltage. He seems to accept that satellites measure outgoing radiation, yet refuses to accept that similar instruments on the ground can measure DLR.

      10

      • #
        bananabender

        The problem is that Wien’s law relates three variables – I challenge anyone to obtain a result from such a law with one input value.

        A $20 ear thermometer does it effortlessly. It detects the emitted wavelengths within the ear canal with a photocell. The voltage signal is converted to a temperature and displayed on a little LCD screen.

        10

      • #
        mkelly

        From this general law, it follows that there is an inverse relationship between the wavelength of the peak of the emission of a black body and its temperature when expressed as a function of wavelength, and this less powerful consequence is often also called Wien’s displacement law in many textbooks.

        where λmax is the peak wavelength, T is the absolute temperature of the black body, and b is a constant of proportionality called Wien’s displacement constant, equal to 2.8977685(51)×10−3 m·K (2002 CODATA recommended value).

        For wavelengths near the visible spectrum, it is often more convenient to use the nanometer in place of the meter as the unit of measure. In this case, b = 2,897,768.5(51) nm·K. This from Wikipedia.

        As you can see there are not three variables. So if we know a T we can get a lambda or vice versa. We only need to know one thing.

        10

    • #

      I’ve just finished reading Nahle (Observations on “Backradiation” during Nighttime and Daytime) which you linked to. His discourse is rather rambling, and I can see some unsupported assertions, but he makes a monumental error early on. He ridicules the idea of “dividing the annual average of incident solar power flux on each square meter of the outer sphere [atmosphere] by four” (my bold). He says it’s 1365 watts/sq.m incident (he quotes 1360 and 1365 at random), and then multiplies that by 0.51 to account for night-time.

      So far so good, but he doesn’t take account of the fact that the 1365 (or 1360) figure is only relevant for the area of the “disc” of the Earth as seen from the sun. Incoming radiation is spread over an area twice that of the”disc”, the sun-facing hemisphere.

      It’s the reason for the factor of four – divide by two for the area of the hemisphere, and again by two for day/night. Put another way, the incident radiation is spread over the entire sphere in a 24-hour period, and that sphere has an area of four times that of the incident “disc”. Incident radiation varies from 1365 with the sun overhead, to near zero with the sun on the horizon. By accounting only for day/night, all his subsequent calculations are out by a factor of two.

      10

      • #
        bananabender

        So far so good, but he doesn’t take account of the fact that the 1365 (or 1360) figure is only relevant for the area of the “disc” of the Earth as seen from the sun. Incoming radiation is spread over an area twice that of the”disc”, the sun-facing hemisphere.

        The Earth is not a disk. It is a rotating solid sphere.

        The non-illuminated hemisphere of the Earth is constantly radiating heat to space.

        If we ignore the 22.5 degree axial tilt most of the incoming radiation is actually concentrated in a relatively narrow band between the Equator and about 45 degrees latitude. The centre section (the “Equator”) gets 1365W/m2 radiation and the poles get almost zero radiation. That is why the tropics are far hotter than the poles.

        According to the Stefan-Boltzmann Law the tropical regions must radiate vastly more energy back into space than the poles because of the huge difference in absolute temperature.

        A quick and dirty calculation:

        Sahara at 55C (328K) T^4 = 11574317056
        Antarctic -60C (213K) T^4 = 2058346161

        So in this example the Sahara is radiating 5.6x as much energy per unit area as the Antarctic.

        The flat disk insolation model of the Earth is total garbage. Therefore the claimed 33k extra temperature generated by the alleged Greenhouse Effect is also total garbage.

        Nahle’s work is imperfect but it is orders of magnitude better than anything produced by mainstream “climate scientists”.

        10

        • #
          bananabender

          Addendum:

          If the IPCC version of the Greenhouse Effect was real the entire Earth’s surface would have a relatively uniform temperature because the atmosphere is well mixed. In fact the recorded ground level air temperatures varies by about 150C (61C in the Sahara to -88C in Antarctica).

          10

        • #

          You didn’t answer the point I made.

          10

        • #

          Posted the last comment to early. You said “The Earth is not a disk. It is a rotating solid sphere.” – that is exactly my point. Nahle uses a figure of 1365 watts/sq.m for the entire hemisphere facing the sun. It should be obvious that the 1365 only applies to the area with the sun directly overhead. Elsewhere the sunlight strikes at an oblique angle, and the radiation covers an increasing area towards the “limb”, i.e. the extreme edge of the hemisphere.

          If you cast a shadow on a wall with a ball, the shadow is a circle – it’s the area of that circle that’s intercepting the light, but the light is spread over the hemisphere facing the light, and the ratio between the two is 2 to 1 – Nahle’s error. For Nahle to be correct, sunlight would have to be striking the surface of the hemisphere vertically all over, all the time.

          For a given sq. metre which has the sun overhead at midday, the insolation is zero until after dawn, rises to a peak at midday, the drops to zero again after sunset. Nahle has it at a constant insolation throughout, 1365 for 12 out of 24 hours.

          10

          • #
            Richard C (NZ)

            MostlyHarmless #80.4.1.3

            Remember it’s instantaneous at zenith but you do have a very good point.

            He seems to be trying to demonstrate that what he ends up with (668.85 W/m2) starting with the TOA zenith (1365 W/m2) is consistent with “measurements of thermal radiation impinging on Earth’s surface (insolation), which fluctuates around the average of insolation of 1000 W/m2” (see pg 4 pdf) without discounting the value anymore to account for curvature.

            But the 1000 W/m2 is also at zenith (a surface perpendicular to the Sun’s rays at sea level on a clear day). So both have to be discounted progressively outwards to the rim of the hemisphere.

            Postma shows a circle with greater than 90% zenith flux in Figure 6, The Model Atosphere. He describes the size of this circle:-

            The solar energy flux density reaching the Earth is F 1370 W / m2 , which is a temperature equivalent of 394K or +121 C. This energy flux density is reduced by the reflective albedo losses from the Earth, and so becomes reduced to 360.5K or +87.5 C at the input. The portion of the Earth which is closest to the Sun is disk-like, and this is indicated by the solid ellipse drawn at the solar zenith. Allowing for a deviation from perpendicularity of no more than 10%, the zenith circle is drawn to-scale in terms of the linear cross section of the sphere; it amounts to almost 50% of the diameter, and can be calculated to occupy a surface area greater than one-third larger than the entire continent of North America. This means that almost 50% of the cross-section of the Earth is continuously being insolated with radiative heating of +87.5 C! Circling this area is for illustrative purposes only, and obviously, would not form any sort of absolute constraint or geographical area of the surface because the solar insolation varies as the smooth cosine function.

            The average input over the entire sun-lit hemisphere has an equivalent of +30 C input temperature. But we cannot and do not treat this as a physically real average over the entire hemisphere, because then we would force ourselves into a position of forgetting about the physics at the zenith, and be thus required to invent a fiction to explain the temperature found there, as the standard greenhouse model illogic would have us do it. The shading of the day-lit hemisphere graphically reminds us that the solar flux is reduced by the cosine of the solar zenith angle (i.e. is maximum at the zenith, and zero around the periphery).

            He should also discount the +87.5 C because that’s only at zenith but he does tell us what Nahle should do to discount the maximum surface 668.85 W/m2 flux at zenith out to 0 at the rim.

            So 535.1 W/m2 is a flawed value but then so is the 163 W/m2 that he is disputing. I don’t think the flux can be realistically represented by one value, it has to be percentage bands starting with the 90% zenith circle then outwards towards the rim.

            Nahle has some work to do on the theoretical but it’s the experiment and results that are important I think.

            10

          • #
            Richard C (NZ)

            Have contacted Prof Nahle via Biology Cabinet for clarification re #80, 4.1.3 and 4.1.3.1

            Don’t know when he will pick up the email though.

            10

          • #

            Dear Richard,

            Effectively, I used averages of solar constant and insolation to calculate the amount of power absorbed by the surface applying the fundamental mathematical procedure used in astrophysics. I didn’t touch the problem of efficiency because it was not the purpose of the introduction to the experiment, but it is also essential to the theoretical issue.

            According to the concept, solar constant is an average of total solar irradiance received on each square meter at TOA. As the solar constant includes the whole spectrum of radiation emitted by the Sun, I calculated the amount of thermal radiation alone. Then again, as such amount of thermal radiation enters the atmosphere, I calculated the mitigation by the atmosphere, which includes radiation reflected and absorbed by the atmosphere before it impinge on the surface. The total is almost the same as dividing the solar constant by four because it is 0.51 of thermal radiation and 0.50 of mitigation by the atmosphere: (1365 W/m^2 * 0.51)* 0.5 = 348.075 W/m^2. These and the following numbers could change a bit, according to the value taken as solar constant and coefficients.

            From 348.1 W/m^2, we have to discount the amount of solar thermal radiation reflected by the surface, but after it has impinged on the surface, not before. After reflection by the surface, the amount of thermal energy is 324 W/m^2. After that, I calculated the amount of thermal radiation absorbed by the surface, which is also another average, i.e. ~240 W/m^2.

            The flaw on other calculations consists on dividing the solar constant by four to include mitigation by the atmosphere and to isolate thermal radiation, i.e. solar irradiance that can be transformed into thermal internal energy.

            However, after doing such division and calculations, they discount them once again, so it is not a division by four, but a division by eight; hence their unphysical results and their need of mistaking thermal energy transport as the greenhouse effect, which is not physical in the way they explain it. Actually, the thermal energy transport by the air molecules is the so called greenhouse effect, but the interest is placed on other things except on science. As McKelly suggests, reality is quite different to theory.

            Just to give an example, the number of possible collisions of carbon dioxide with other molecules of the same species or of other species is 3.53 x 10^5 x^-1. The latter means that the molecule of carbon dioxide can move for about 0.15 meters before it collides into another molecule, and that the time it takes between collisions is 0.00283 milliseconds. However, the time the molecule of carbon dioxide takes to emit radiation is 120 picoseconds. Therefore, it is more possible that carbon dioxide cools other molecules by collision than warming them by radiation. The latter is well explained by redshift after radiation, which draws another impossibility regarding the absorption of the energy emitted by carbon dioxide molecules by other molecules of the same species, or by Nitrogen, Oxygen and Argon.

            Regarding the globules of air, it was demonstrated by experimentation during a windy night. Backradiation became chaotic as the wind speed increased or decreased, which is impossible to happen because thermal radiation has not gravitational mass, but inertial mass. The most feasible conclusion is that what we are measuring is not backradiation, but stationary energy of globules of air; otherwise, the increase of change of temperature with time would not be possible because the emittance of air remains constant at 0.201 under such conditions.

            All the best,

            Nasif

            10

          • #

            @MostlyHarmless…

            Posted the last comment to early. You said “The Earth is not a disk. It is a rotating solid sphere.” – that is exactly my point. Nahle uses a figure of 1365 watts/sq.m for the entire hemisphere facing the sun. It should be obvious that the 1365 only applies to the area with the sun directly overhead.

            1365 W/m^2 is an annual average of the total solar irradiance impinging on any square meter of the outer sphere. We cannot take any angle into consideration before it impinges on the Earth’s surface; therefore, the solar constant is the same at a distance of 1 AU from the Sun.

            Angle of incidence is taken into account to calculate local insolation at any hour during daylight, obviously, but not to reduce the power of the Sun on the outer sphere. Imagine an egg; the eggshell is the outer sphere and the yolk is the Sun. The only things that could mitigate the solar constant are distance, large objects like the Moon, and relatively dense clouds of interplanetary dust. Here the formula to calculate solar constant:

            Bolometric radiation from the whole surface area of solar sphere:

            Pʘ net = (0.9875) (6.079 x 10^18 m^2) (0.56697 ((erg/s)/(m^2 K^4)) (5804.135 K)^4 = 3.8626 × 10^33 erg/s

            Solar Power emitted from each square meter (flux of power) of surface area of solar sphere:

            fʘ = 6.354 × 10^14 (erg/s) / (m^2)

            Yearly average (considering aphelion and perihelion) of incident Sun’s flux power on each square meter of surface area of the outer sphere:

            fOS = fʘ * (Rʘ / d)^2 = 6.354 × 10^14 (erg/s)/m^2 * ((6.955 x 10^8 m) / (1.496 x 10^11 m^2 = 1.37334 x 10^10 (erg/s) / (m^2)

            1.37334 x 10^10 (erg/s) / (m^2)) is the average of solar irradiance received per each square meter of the surface area of the outer sphere, whether the Earth is there or not.

            (1.37334 x (10^10) * (erg/s))/(m^2) = 1373.34 W/(m^2)

            After satellite confirmation, the average is 1365 W/m^2.

            We cannot “mitigate” this amount or power before it enters the atmosphere and strikes the surface of the Earth.

            “Elsewhere the sunlight strikes at an oblique angle, and the radiation covers an increasing area towards the “limb”, i.e. the extreme edge of the hemisphere.”

            You are referring to insolation, not to solar constant.

            “If you cast a shadow on a wall with a ball, the shadow is a circle – it’s the area of that circle that’s intercepting the light, but the light is spread over the hemisphere facing the light, and the ratio between the two is 2 to 1 – Nahle’s error. For Nahle to be correct, sunlight would have to be striking the surface of the hemisphere vertically all over, all the time.”

            There is not any error in my calculations. You are confunding solar constant with insolation. They are not the same thing.

            For a given sq. metre which has the sun overhead at midday, the insolation is zero until after dawn, rises to a peak at midday, the drops to zero again after sunset. Nahle has it at a constant insolation throughout, 1365 for 12 out of 24 hours.”

            For a given square metre which has the Sun overhead at midday, the insolation is around 1000 W/m^2; you wrote it wrong. Please, correct the phrase.

            Again, you are confunding solar constant (1365 W/m^2), which is the same day and night because the Sun does not turn off in any moment, with insolation, which is the incident solar irradiance on a given surface area of the hemisphere facing the Sun. Please, correct your argument.

            All the best,

            Nasif

            10

      • #
        mkelly

        He ridicules the “divide by 4” idea because it is not physical reality. It can be shown (and is on SOD site) that in Canada near 45 N direct sunlight is 700 W/m^2 at noon. At the equator the surface recieves near 1100 W/m^2.

        10

      • #
        Richard C (NZ)

        MostlyHarmless #80.4 you say:-

        ……incident solar power flux on each square meter of the outer sphere [atmosphere] by four” (my bold). He says it’s 1365 watts/sq.m incident

        No he doesn’t

        He defines Solar Constant on pg 7 pdf:-

        Solar Constant is the amount of energy emitted from the Sun, at all wavelengths of the electromagnetic spectrum, impinging on the surface area of a virtual ellipsoidal sphere, known as the outer sphere, whose semi-major axis is twice the distance from Sun to Earth (average of 1.49597871 x 108 km, or 1 Astronomical Unit, i.e. 1 AU), and its major perimeter is traced by the orbit of the Earth moving around the Sun. Solar Constant to Earth system is 1365 W/m2, which is the annual average of measurements by satellite.

        Next he defines insolation:-

        Insolation is the amount of solar power impinging on a given surface area of a planet. It is ~1000 W/m2 on Earth. Over real situations and locations, the energy absorbed and reflected by the atmosphere, and the amount of energy reflected by the surface are discounted from the total solar irradiance impinging on top of the atmosphere. It gives a theoretical value of 668.85 W/m2. However, this value fluctuates due to the incident solar angle; therefore, a real measurement could be higher or lower than 668.85 W/m^2. From 668.85 W/m^2, only 535.1 W/m^2 is thermal radiation absorbed by the surface. 56% is stored by surface and subsurface materials, i.e. 309.43 W/m2, which causes a surface temperature of 24 °C.

        So the effective solar heating agent on surface and subsurface material is 309.43 W/m2. But TF&K09 say that DLR from clouds and GHGs is a global average 333W/m2. The question is: what discount factor must be applied to 333W/m2 (or any of the observed spectra) to arrive at the effective heating flux of DLR on surface and subsurface material. I say it’s about 0.99 (and I’m not being facetious).

        Irrespective of the veracity and quality of Nahle’s discourse (I agree. it’s not easy going and there are questions), the answer to his question:-

        Does thermal radiation emitted by a cooler atmosphere warm up a warmer surface by backradiation?

        was no.

        In addition, surely you must agree that The Realistic Terrestrial System Model in “The Model Atmosphere” (Postma) is a better graphical representation of the radiative thermodynamic ensemble than the Standard Atmospheric Greenhouse Model?

        10

        • #

          Dear Richard… I’m just now in hurry and leaving out. I’m working on a paper on the issue you propose. However, I will come back in a couple hours to explain some generalities of this issue. I’m sure you’ll find them interesting.

          Best,

          Nasif

          10

        • #

          And there’s the problem

          Solar Constant is the amount of energy emitted from the Sun, at all wavelengths of the electromagnetic spectrum, impinging on the surface area of a virtual ellipsoidal sphere,

          The solar constant is the amount of energy impinging on a square metre at right angles to the rays (the Sun/Earth axis), and not the surface of a sphere (ellipsoidal or not).

          He goes on to say

          However, this value fluctuates due to the incident solar angle; therefore, a real measurement could be higher or lower than 668.85 W/m^2

          So why does he use 668.85 W/m^2 when that figure is clearly not the average for the whole surface?

          10

          • #

            @MostlyHarmless…

            You’ll receive 1365 W/m^2 on every square meter of the outer sphere. That is the meaning of that m^2 in units W/m^2. 1365 W/m^2 is an annual average of total solar irradiance.

            668.5 W/m^2 is thermal radiation alone. Thermal radiation is power that is transformed into heat. The remainder is not thermal radiation, but other kinds of radiation that are not transformed into heat.

            Solar irradiance is one thing; thermal radiation is another thing. Total solar irradiance is 1365 W/m^2; thermal radiation, from total solar irradiance, is 668.5 W/m^2.

            All the best,

            Nasif

            10

          • #

            You’ll receive 1365 W/m^2 on every square meter of the outer sphere.

            No you won’t – only the part of the sphere which is at right-angles to the incoming rays will receive that amount. Other parts are at increasing angles towards the outer “edge”, and receive progressively less. Check out definitions of “solar constant”on the ‘net. Right-angles to the rays is part of the definition.

            10

          • #

            @MostlyHarmless…

            “No you won’t – only the part of the sphere which is at right-angles to the incoming rays will receive that amount. Other parts are at increasing angles towards the outer “edge”, and receive progressively less. Check out definitions of “solar constant”on the ‘net. Right-angles to the rays is part of the definition.”

            It seems you don’t understand what the solar constant is and what the outer sphere is.

            Let’s try again…

            Bolometric radiation from the whole surface area of solar sphere:

            Pʘ net = (0.9875) (6.079 x 10^18 m^2) (0.56697 ((erg/s)/(m^2 K^4)) (5804.135 K)^4 = 3.8626 × 10^33 erg/s

            Solar Power emitted from each square meter (flux of power) of surface area of solar sphere:

            fʘ = 6.354 × 10^14 (erg/s) / (m^2)

            Yearly average (considering aphelion and perihelion) of incident Sun’s flux power on each square meter of surface area of the outer sphere:

            fOS = fʘ * (Rʘ / d)^2 = 6.354 × 10^14 (erg/s)/m^2 * ((6.955 x 10^8 m) / (1.496 x 10^11 m^2 = 1.37334 x 10^10 (erg/s) / (m^2)

            1.37334 x 10^10 (erg/s) / (m^2)) is the average of solar irradiance received per each square meter of the surface area of the outer sphere, whether the Earth is there or not.

            (1.37334 x (10^10) * (erg/s))/(m^2) = 1373.34 W/(m^2)

            After satellite confirmation, the average is 1365 W/m^2.

            Bolometric means the whole electromagnetic spectrum emitted from the whole surface area of the Sun. Divide bolometric emissions from the Sun by the solar surface area and you’ll get the amount of bolometric solar emission from each square meter of the Sun. Remember the Sun is not turned off on any region of its surface area, so it is radiating in all directions the same amount of power.

            Such radiation emitted in all directions because the Sun is an sphere, which is turned on on every square meter of its surface area, travels accross 1 AU and it is mitigated by distance to the amount known as solar constant. Solar constant is received with the same intensity on the surface area of the outer sphere, which is an ellipsoidal sphere, of course:

            http://earthobservatory.nasa.gov/Features/SORCE/sorce_05.php

            http://earthobservatory.nasa.gov/Features/SORCE/sorce_02.php

            http://solardat.uoregon.edu/SolarRadiationBasics.html

            http://edmall.gsfc.nasa.gov/inv99Project.Site/Pages/science-briefs/ed-stickler/ed-irradiance.html

            http://ocw.tudelft.nl/fileadmin/ocw/courses/SolarCells/res00026/CH2_Solar_radiation.pdf

            Lang, Kenneth. 2006. Astrophysical Formulae. Springer-Verlag Berlin Heidelberg. Vol. 1. Sections 1.11 and 1.12.

            Maoz, Dan. Astrophysics. 2007. Princeton University Press. Princeton, New Jersey.

            Sutton, David B., Harmon, N. Paul. Ecology: Selected Concepts. 2000. John Wiley & Sons, Inc. New York.

            All the best,

            Nasif

            20

          • #
            Richard C (NZ)

            MostlyHarmless #80.4.3.2.2

            You and Nasif are at cross-purposes because you are confusing solar constant with insolation (see the definitions – 2 different outer spheres).

            But when it comes to the efficiency of insolation on the earth’s surface hemisphere you are on the right track and Nasif is addressing that in the future, see up-thread #80.4.1.3.3

            I didn’t touch the problem of efficiency because it was not the purpose of the introduction to the experiment, but it is also essential to the theoretical issue.

            And see #80.4.3.1

            I’m working on a paper on the issue you propose. However, I will come back in a couple hours to explain some generalities of this issue. I’m sure you’ll find them interesting

            The “issue you propose” and the technique for discounting the zenith maximum for surface insolation efficiency over the hemisphere is laid out in #80.4.1.3.1

            ……..but he [Postma] does tell us what Nahle should do to discount the maximum surface 668.85 W/m2 flux at zenith out to 0 at the rim [pg 36 pdf].

            […] I don’t think the flux can be realistically represented by one value, it has to be percentage bands starting with the 90% zenith circle then outwards towards the rim.

            See The Model Atmosphere for a graphic (Figure 6, pg 35 pdf) showing “the 90% zenith circle”.

            So your concerns are being addressed but you will have to wait for Nasif’s paper (unless he gives us the overview here and now) and in the meantime make do with zenith values (consistent with observed zenith values) and Postma’s Figure 6.

            10

          • #

            What happens on the surface of the sun is irrelevant – what matters is what happens over a square metre perpendicular to the Sun/Earth axis. Only a small area of a sphere at 1 AU from the sun is perpendicular to that axis – it’s self-evident, and shouldn’t need a proof.

            Solar Insolation – Goddard Space Flight Center Education Programs (with diagrams)

            The solar constant is the average amount of energy striking one square meter (perpendicular to the suns’ rays) each second at the top of the earths’ atmosphere. The satellite measured solar constant is 1366 W/m^2

            Which also says something very obvous

            The angle at which solar radiation strikes a surface dramatically effects the amount of energy received by the surface.

            Solar Radiation and the Earth System (NASA)

            Solar Constant – The solar constant is the amount of energy received at the top of the Earth’s atmosphere on a surface oriented perpendicular to the Sun’s rays (at the mean distance of the Earth from the Sun). The generally accepted solar constant of 1368 W/m2 is a satellite measured yearly average.

            solar constant — Britannica Online Encyclopedia

            Solar constant, the total radiation energy received from the Sun per unit of time per unit of area on a theoretical surface perpendicular to the Sun’s rays

            10

          • #
            Richard C (NZ)

            MostlyHarmless #80.4.3.2.5 you say:-

            What happens on the surface of the sun is irrelevant

            Nobody is saying it does (except you).

            You’re STILL confusing solar constant with insolation – they are at the surface of 2 different outer sphere’s: Solar constant elliptical with the sun at centre, insolation a circular sphere with the earth at the centre. Where they touch is where the elliptical average 1368 W/m2 takes effect.

            Nasif is perfectly aware (as I have shown) that he has not determined the efficiency of insolation (that you are disputing) because “it was not the purpose of the introduction to the experiment” but he is “working on a paper on the issue”.

            Meanwhile here’s a challenge. There is nothing to stop you working out the efficiency of insolation over the hemisphere yourself. The technique is (from Postma’s The Model Atmosphere):-

            the solar flux is reduced by the cosine of the solar zenith angle (i.e. is maximum at the zenith, and zero around the periphery)

            Do it yourself or wait for someone like Nasif to do it for you – it’s up to you.

            Just don’t wait for climate science to do it or you will wait forever.

            10

          • #
            Richard C (NZ)

            Turns out (from the NASA GSFC link) that climate science is on to it after all and provides the procedure (same as Postma’s) to work out insolation efficiency over the hemisphere.

            Procedure:
            )
            In this investigation students will calculate, graph & compare the solar insolation at different latitudes, dates and times of day. While comparing the calculated insolation values, the student will identify cyclic patterns in seasonal and daily insolation at different latitudes. Show all work on your worksheet

            Step 1: Calculate the maximum solar insolation received at your school, that is the insolation at noon on the summer solstice on a clear day.

            * First calculate the zenith angle.
            Z = [equation doesn’t copy]

            * Use your zenith angle to calculate insolation.

            I = S cos Z

            Step 2: Use the procedure in step 1 to calculate the insolation values for the table on your worksheet. On a sheet of notebook paper show each calculation and box your answer.
            Add the values to your table.

            Step 3: Use a different color to graph the insolation values for each of the different latitudes.

            Question is: why didn’t Kiehl and Trenberth (and latterly Trenberth, Fasullo and Kiehl) use the NASA GSFC procedure?

            10

          • #

            Nobody is saying it does (except you).

            Nasif introduced it #80.4.3.2.3

            Nasif is perfectly aware (as I have shown) that he has not determined the efficiency of insolation (that you are disputing) because “it was not the purpose of the introduction to the experiment” but he is “working on a paper on the issue”.

            Then his paper is merely an exercise ( a long-winded one at that) in pointing out various errors (as he sees them) in existing radiation budgets, etc., and bears no relation to the real situation at all. Thank you.

            BTW, I agree that there are many errors of varying magnitude in K&T’s energy budget. I’m not attempting to defend that at all. If the opportunity arises here, I’ll highlight those I’ve identified.

            10

          • #

            @MostlyHarmless…

            You’re going round and around the same thing. Wherever you stand on the orbit of the Earth around the Sun you will receive, in average, 1365 W/m^2 of solar irradiance.

            On surface of Earth, things change because you must take into account mitigation by the atmosphere, angle of incidence of solar irradiance and day hour. If you stand on a square meter of the surface where you receive Zenith solar power, It would be ~1000 W/m^2.

            Two different things that you confound very easily.

            Please, MostlyHarmless, behave professionally and do not put in my mouth things that I have not said. You said at 80.4.3.2.5:

            What happens on the surface of the sun is irrelevant – what matters is what happens over a square metre perpendicular to the Sun/Earth axis. Only a small area of a sphere at 1 AU from the sun is perpendicular to that axis – it’s self-evident, and shouldn’t need a proof.”

            That is your argument that Richard said that only you say, not me.

            I say that what happens on the surface of the Sun is crucial for understanding the thermal radiation that the Earth is receiving every second, every day. If you don’t consider the amount of thermal radiation emitted from the surface area of the Sun, which is our main primary source of energy, you would never understand climate.

            Also, you are turning off the Sun on its surface except on one square meter. The Sun is a sphere and it is radiating the same amount of thermal radiation at any angle, from any square meter of its surface area, and any object at a distance of 1 AU from the Sun will receive ~1365 W/m^2 of solar power.

            The circumference of the Earth’s orbit is 9.42 x 10^11 meters. Drawing a set of surface areas of one square meter each on such circumference, each one of those meters of the orbit receives in average ~1365 W of solar power.

            10

          • #

            @MostlyHarmless…

            You are speculating on my work as you say on your post 80.4.3.2.8:

            Then his paper is merely an exercise ( a long-winded one at that) in pointing out various errors (as he sees them) in existing radiation budgets, etc., and bears no relation to the real situation at all. Thank you.

            BTW, I agree that there are many errors of varying magnitude in K&T’s energy budget. I’m not attempting to defend that at all. If the opportunity arises here, I’ll highlight those I’ve identified.

            Have you read my paper, which I have not started writting yet?

            I must tell you that my paper will not be “merely an exercise”. It will be supported by experimentation.

            10

          • #

            Addendum on my post 80.4.3.2.9, last paragraph:

            The circumference of the Earth’s orbit is 9.42 x 10^11 meters. Drawing a set of surface areas of one square meter each on such circumference, each one of those meters of the orbit receives in average ~1365 W of solar power.

            The circumference of the Earth’s orbit is 9.42 x 10^11 meters. Drawing a set of surface areas of one square meter each on such circumference, each one of those meters of the orbit receives in average ~1365 W of solar power at the same time.

            The add is in bolds.

            Best,

            Nasif

            10

  • #
    MaryFJohnston

    michael hammer

    Thank you for the reply @ 68.1.3.2.1

    It seems we agree on many basics.

    It is only the extension of the basics that concerns me.

    One large example is the comment about the absorption diagram which refers to the big bite by CO2 without any mention of Water ?? Was there any attempt to show the value for the human CO2 contribution on the diagram?

    Another issue is the attempt to calculate watts/ m2 and work out a heat balance. This is a warmer thing and representative of their half baked “science”.

    They will get every important (to them) heat source/sink except the important ones.

    The reason is that the system is so complex you can’t actually do it correctly.

    The final watts/m2 figures quoted need to be qualified; at night? daytime, at equator, at pole etc — there is no “one figure fits all” answer to this and yet we at still bombarded with exact figures.

    Falling into the Warmenista trap of quoting stuff like “The figure of 1C per doubling of CO2 “” is also a concern.

    Is that for the first doubling or the second or third? The effectiveness of CO2 as a devil heating agent is reduced as more is added and it is reduced logarithmically.

    This is a big problem and while qualified in most areas of the AGW theory many years ago I would not consider putting myself up as an expert – I’m here to learn.

    10

    • #
      michael hammer

      Starting at the end you are right the response to adding CO2 is logarithmic which is why we talk about incremental energy per doubling. Is it for the first doubling or the seond etc – its for any doubling but remember what doubling means. If you are 1 pm then doubling means addin another 1ppm. Since we are at 280 ppm doubling means adding a further 280 ppm. If we got to 560ppm the next doubling would mean adding a further 560 ppm.

      One can in fact get a reasonable estimate of the impact of CO2. If you look at the “bite” in the spectrum in the original post and estimate the amount of energy it represents it is about 27 watts/sqM (I compute that from the formulae not by estimating from the graph of course). Now since CO2 is about 2000 abs thats about 1000 times the level to saturate line centre (which when the logarithmic relationship starts and also when the and green hosue impact starts to become n any way significant) 1000 times is about 10 doublings so the incremental energy per doubling is about 2.7 watts/sqM which I compute to contribute about 0.8C without feedbacks. Others claim values betwen 0.8 and 1.2. However thats before feedback. I claim there is very good evidence to suggest feedback is STRONGLY negative in which case the change with feedbacks is far less.

      Gloabl warming may be sort of right in theory but if the impact is neglegible (as I beleive it is) who cares and why pass laws to address irrelevant issues.

      10

      • #
        MaryFJohnston

        Again Michael Hammer thanks for the reply and again we have much in common.

        Where we differ is that I work from first principles and you seem to be refining calculations (eg watts/m2 and forcings) which are fraught with traps for the unwary.

        The thing not mentioned in your reply was my comment about the absorption diagram which refers to the big bite by CO2.

        This comment, from my point of view, is misleading because that bite would possibly include water?

        How much water – actually a lot — how much human CO2 — actually not much at all.

        There would be value in making the attempt to show the value for the human CO2 contribution on the diagram.

        A great deal can be shown by quantifying the effect of H2O, All CO2 and human origin CO2.

        I have done this in past post and the point is made immediately that the effect of human origin CO2 as a GHG is negligible. That is ALL we are obliged to show; the basic science.

        Human CO2 is an irrelevance, why adorn it with ambiguity?

        The main point I make though, is that the mass, heat and momentum balance of the atmosphere is EXTREMELY complex and any one who claims to have it even roughly worked out is not being honest.

        I think this is what we have every right to push at the Warmer movement.

        10

        • #
          michael hammer

          I also look at first principles and the calculations I do are not refining numbers, they are to get ball park estimates to see how significant an effect is likely to be. For example when i calculate the impact of CO2 per doubling I get a number between 0.8C and 1.2C, thats quite a range but it does not include 3-6C. But you seem to be talking more about your gripes with the warmists so let me respond with my gripes about the warmists

          1 They claim 0.6C of warming since 1970 and at an accelerating rate. However their “corrections” or better manipulations, of the data contribute almost all of the 0.6C and the accelerating rise matches their accelerating corrections. I have plots from Hansen of global temperature released in 1998 and exactly the same plot from about 2003 and they are widely different by about 0.5C. How pray tell does the historical record from 60 years ago change between 1998 and 2003? Why do ALL the corrections act to exacerbate the apparent rise in temperature? Things which are known distortions such as urban heat island (which creates a trend the other way) is totally discounted and considered to minor to correct for. Why did the national academy of science show 0.7C of northern hemisphere cooling betwen 1940 and 1970 yet now the same plot from Hansen shows no cooling at all? These factors are of the same size as the claimed trend. How can you possibly claim a reliable trend when the difference between data sets equals the trend claimed? I do accept there has been some warming between 1970 and 1998 but in my view simply back to the temperatures in the 1930’s. So what is the problem exactly?

          2 The claim is CO2 levels are rising and man is responsible. Yes they claim man only contributes 3% but that 3% builds up year over year. That would imply no stabilizing feedback at all which in my view is utter BS. Satellite images show that plants are putting on more growth in the higher CO2 atmosphere, thats stabilizing feedback! But that does noo suit the argument so its conveniently ignored. More significantly, there are well over a century worth of wet CO2 determinations documented. These were carried out by extremely reputable and careful scientists. They show CO2 levels in the 1800’s well above what we have today. There was a period towards the end of the 19th century when levels were at about 280ppm but then in the erly 1900s it went up again and in the 1930’s was at about 400 ppm. Warmists claim the 280 ppm data collected at the end of the 19th century is correct but all the other measurements were flawed and poorly done so they should be discarded. Don’t know if you know the yiddish word Huzpah it means more or less monumental gall and seems to fit very well here. Pick the data that suits your argument and discount all other data as flawed? Does that strike you as maybe bias? If the wet chemical determinations are correct then clearly something else is going on and it needs careful research.

          3 The claim of positive feedback in the climate system and not just positive feedback but MASSIVE positive feedback. If that were true the climate would have been grossly unstable and would have been wandering around all over the place. Yet they also claim it has been stable at 287K for millenia. Both cannot be true in my mind. That sort of stability requires very tight negative feedback.

          4 Their claim of positive feedback leads to one testable hypothesis – notably that there should be a hotspot in the upper troposphere in the tropics. Yet quite literally hundreds of balloon flights have shown there is no hotspot. IN any rational scientific endeavour that would debunk the thoery right there but instead the warmists claim Oh no you cant trust thermometers for this sort of measurement we know its there because of wind shear. Thats like saying you cant determine the weight of something by using a scale its better to use a clock – ie: absurd!

          5 All the hype over runaway temperatures when one comes right down to it are based on models which have not yet got one prediction right. The models leave out things such as solar magnetic fields which means they don’t figure in the simulations ie: zero impact. What else do they leave out. Who determines the relative importance of each factor. Basically the scientists ascribe a high sensitivity to CO2 and then say Look the models show a high sensitivity to CO2.

          6 The warmists say the temperature rise must be due to man because we dont know of any other reason. That implies they know every possible reason and none other than man fit. I suspect its far far more likely they have very imperfect knowledge of the things that influence climate. Its all very well to use the Sherlock Homes quote “when you have eliminated the impoassible whatever remains however improbable must be the truth” but it does rather assume you know all the factors and which are impossible.

          I could go on but surely that is enough. If the entire basis of their claim was false it would have been debunked years ago. Scientists are not that stupid. The problem is far more subtle. The very basic premise is correct. CO2 is a green house gas and green house gases do reduce heat loss to space and thus increase the Earth’s temperature. However the question is by how much and what are the mitigating factors. The warmists manipulate what experimental data there is to suit their purpose and manufacture more data which is purely conjecture but is presented as fact. I find that reprehensible and dangerous.

          10

          • #
            MaryFJohnston

            Hi Michael

            Thanks for the summary — most is familiar stuff but your two comments are still, I feel, giving too much credit to the warmer side.

            1. ” Scientists are not that stupid.”

            No I agree but you can’t say that about Climate Scientists who area special case.

            It never ceases to amaze me that environmentally qualified “scientists” will take an equation with complex parameters and strict boundary limits and adapt it to some element of “Climate Science” that needs beefing up.

            2. “”The problem is far more subtle.””
            Not sure I can “see’ this.
            The entire atmospheric dynamic involves Mass Heat and Momentum transfer.

            I don’t believe that most scientists can even start to appreciate just how complex this large system really is.

            The failure to acknowledge it (atmospheric science) as a work in progress seems to be a fault of both Climatologists and other mainstream scientists.

            10

  • #
    mobihci

    this is what hammer is looking for –

    http://geosci.uchicago.edu/~rtp1/papers/PhysTodayRT2011.pdf

    but it has problems passed on from earlier versions. from memory, lubos motl over at the reference frame talked about the earlier version of that paper by pierrehumbert a few years ago. there were some fundamental flaws then, and it looks like they have not been revised, but thats just me, and i am no pro.

    this is also an important text on this-

    http://oceanworld.tamu.edu/resources/ocng_textbook/chapter05/chapter05_02.htm

    which shows the role water vapour plays.

    10

  • #
    Bob_FJ

    Michael Hammer,
    Please refer to a number of posts @:

    Mostly Harmless @ 72.1, My own @ 75, Mike W @ 79, and various posts by MaryFJohnston starting at 72.

    Will you be responding to the unanswered questions therein?

    10

    • #
      michael hammer

      Bob; my computer does not show the post numbers and without these I cannot find the posts you are referring to. I have asked Jo whether there is something I can do the recover those numbers. Maybe if I had the post times it would help, I can see those. Sorry

      10

    • #

      Indeed – I do have a life outside the internet, believe it or not.

      10

  • #
    Lars P

    Michael,
    I am not sure, is this diagram above of the “earth radiation” goes from radiating “rocks” or is it above oceans?
    As the earth is 75% covered with oceans and only the rest is rock I would say that any radiation theory that calculates the average temperature of the earth not being based on oceans is plotting the wrong planet.
    Can you help me on this, is this radiation observed by nimbus over oceans or over land? Are there any differences?

    10

    • #
      Lars P

      The further question would be, does water (oceans) radiate the same as solid ground? Do you have some sources pointing this out – how different is it? What spectrum do the oceans radiate? As what we see at Ceres is what is coming out from the whole system, I would like also to understand is what is going out from the oceans in form of radiation.
      Once we have the “oceans radiation” we can compare with the Ceres result. This would be better then comparing with black body, or does black body aproximate well oceans radiation?

      10

    • #

      As the legend below the diagram says, “tropical Pacific Ocean”, so it’s relatively warm with high atmospheric water vapour content. Plots obtained over different surfaces, land (with vegetation), desert, polar ice, all show very different profiles. CO2 is much the same in all, but water vapour and ozone vary – for example these are low over the polar regions.
      You might be interested in Surface Emissivity Maps for Use in Satellite Retrievals of Longwave Radiation from NASA.

      10

      • #
        Lars P.

        Thanks for the answer. Looking at the Venus versus Earth graph what struck me was the very strong correlation that Huffman finds:
        “The derivation of the radiating temperature above is for absolute temperature, in degrees Kelvin (K), so the 1.176 factor relates the Kelvin temperatures, not the Celsius temperatures.”
        It is visible also in Jo’s graph if one takes all under 11.5 km height. It is there. It is difficult to explain it as coincidental resulting from other reasons, when it is exactly the ratio resulted from distance from the sun.
        So Huffman has a point.
        On the radiation part I am trying to understand where are the flaws of the greenhouse theory. What the wavelenghts/watts radiance graphs shows is a total outflow from the earth. I guess it is commonly understood as some escaping waves and some blocked in the atmosphere, whereas it is in fact a result of radiation coming from the total system, including radiation from the atmosphere. The curve there differs from the blackbody curve. It would ideally fit the 270°+ blackbody curve (maybe close to oceans’ temperature of +4°C) where some pieces at some wavelenghts (14 to 17) are reduced whereas others (8 to 13) are increased.

        10

        • #

          Lars, it is a striking coincidence indeed that the number matches – 1.176 – but notice that if that’s true, that planetary temp is solely decided by distance from the sun, then albedo by inference would have no, as in “zero” effect.

          Try to imagine a black planet and a white planet at the same distance from the sun, and convince yourself that they would be the same temperature regardless.

          Doesn’t work does it?

          10

          • #
            Lars P.

            Jo, thank you for answering, this is a good question indeed. We all know that black would get faster warmer in direct sunlight then white.
            Huffman tries to explain it the following way:”both atmospheres must be warmed, overall, essentially in the same way, by direct IR solar irradiation from above, not by surface emissions from below” & “the real effect of infrared absorption and emission in the atmosphere is to enable a more efficient heat flow in accordance with the governing lapse rate, not to trap or slow down heat transfer to space” & “Thus we know that the visible portion of the Sun’s radiation is not what heats the two atmospheres (because Venus doesn’t take in 1.9 times as much visible light as the Earth, it takes in substantially less). Both atmospheres do, however, absorb infrared, and the comparison I have made shows they both must absorb the same portion of the incident infrared from the Sun, thus preserving the 1.9 power ratio calculated from their distances from the Sun.”(picked from discussions in comments to his post).
            Further question I would need to search for would be:
            How big is the sun IR radiation in comparison with Earths’?
            Is radiation in different wavelengths effectively equally transformed into heat? Would different sources of light, lets say ultraviolet 100W as well as visible light 100W or infrared 100W heat the same way their target? This sounds like a home experiment one could try. Then would be the albedo question – if it is the same for all spectrum or not.

            10

  • #
    co2isnotevil

    It’s been my hypothesis that the reason the Venusian surface is so hot is 2-fold. First, the thermal mass of Venus is between the surface and space, while on Earth, the surface is between the thermal mass and space. Second is that Venus stores energy in it’s thermal mass as energized CO2 molecules, while on Earth, the thermal mass is ground state water. The Earth stores energy as a temperature difference across the thermocline, where the thermocline insulates the deep ocean cold, thermally connected to polar ice by the thermohaline, from warm surface waters heated by the Sun. In fact, the dT/dx of the thermocline and the corresponding power flux passing through it can be calculated from the insulating properties of water.

    The Venusian atmosphere should also develop a thermocline like structure which will also act to insulate a hot region (surface) from a cold region (space). In this case, the source of power arriving at the surface is the energized CO2 of the atmosphere which is as decoupled from the Sun as the bottom of Earth’s oceans. Note too that clouds on Earth act to to thermodynamically decouple the surface from the Sun.

    On a related topic, the plot of the spectrum emitted by the Earth shows something very important. Even though the atmosphere is 100% opaque at 15u, over 1/3 of the energy is not being held by the planet and escapes anyway (see half-of-the-energy-is-flung-out-to-space-along-with-the-model-projections). The 3.7 W/m^2 claimed as incremental forcing is calculated as the incremental absorption from doubling CO2, not the increase in power not leaving the planet, which will be less.

    10

  • #
    Truthseeker

    Michael Hammer,

    If you want something specific using actual real-world data and with some nice equations to sink your itellectual teeth into, let me point you to this article that also shows the error of your ways …

    Long live science!

    10

    • #
      co2isnotevil

      Truthseeker,

      There’s an easier way to determine the spectral characteristics of the atmosphere, use MODTRAN or the equivalent and calculate it from HITRAN line data. This is something that computers are actually good at.

      You can also measure it. The planet emits 240 W/m^2 @ 255K and the surface emits 390 W/m^2 at it’s average temperature of 288K. The emissivity is defined as the ratio between the power leaving the planet and the power leaving the surface, or 240/390 = 0.615. The value of 0.4 is about 1-e, which is the net absorption of about 38%.

      Of course, you must consider that the planet is 2/3 covered by clouds and the global emissivity of .615 gets 2/3 of it’s contribution from clouds and 1/3 from the clear sky. One way to do this is using HITRAN based simulations which put clear sky absorption at about 64%, which since half ultimately finds it’s way out, puts the clear sky net emissivity at about (1-.64)+.64/2, or about 0.68 (32% net absorption) making the cloud emissivity about 0.583 (42% net absorption), such that 2/3 * 0.583 + 1/3 * 0.68 = 0.615 (38% net absorption). Direct measurements of IR measured from weather satellites confirms these relative emissivities.

      10

      • #
        Truthseeker

        co2isnotevil – I have to say I am not sure if you are agreeing with me or posing a counter argument. You need to be careful as the original article that started this discussion was only comparing Venus and Earth at the same air pressure levels (1000 mb to 200 mb). Talking about what is happening at the surface of Venus may well be adding oranges to a discussion about apples.

        10

  • #

    Truthseeker # 46.2.1 I hope you will see this comment, with delay, hectic days. Promoting my book. Nevertheless I will get to the point:

    When wind, O+N takes the heat away, can be stopped by shirt / empty shopping bag. On the other hand, the coldness outside the stratosphere works on different principle. Earth is much more efficiently utilizing that unlimited coldness – by spinning fast and orbiting faster than Venus. It was almost as shorthand text, but the rest you did understand.

    Therefore, by emphasizing that grammar is the most important, you are suggesting that you can, but the rest of the English speaking people are stupid, cannot understand without better presentation. WRONG! I have completely simplified complex issues – everybody over 12y old can understand what is on my website; if interested in the truth. It all depends on the 80% of the people on the street; that are not involved, but are most important for the politicians. If they are well informed, or misinformed.

    You say that you have being few times on my website (same applies to Mostly Harmless). Not one pointed something wrong, with a comment there. WordPress model has a chart how many people clicked, it was over 600, but only 2 comments, none yours, or from harmless. Instead, on a sleazy way, both of you are suggesting in Joanne’s blog, for me to get grammatical assistance from memoryvault – exclusively to put people of my blog. Would be the same, if I suggested to you; to get scientific advise from memoryvault; but I will not reduce myself to that level – we are all after same result. My motto always is: nobody knows everything – everybody knows lots of things.

    Just happens that, I am the only one with all the solid proofs on climate/ phony GLOBAL warming. Appropriate would have being: if you wanted additional evidences, to make there comment. B] to articulate on the things you believe I am correct, on anybody’s blog, even on yours. I have the most solid proofs / gold bullions there, your problem is; they are not presented on a silver dish…? In other words; both of you are suffering from inferiority / superiority complex. Nothing wrong with that, present my formulas and everything associated, on Oxford English, for similar sufferers.

    To please you three, I personally will never use words as: albedo, observation, noise, debunking, energy budget, anthropogenic, crapogenic. Those words and similar are exclusively used by the Warmist – people on the street to think that they are talking science. You are singing Warmist lyrics = dignifying the Warmist and giving them oxygen. On this page mostly harmless craps about the ‘’evil methane’’ = dignifying the Warmist /supporting Warmist lies. You wouldn’t suggest to him, to inform himself of some truth, on my website. Why? To shorten even more: Warmist are 10 steps ahead of you, with a bridle; I am 100 steps ahead of the Warmist. Memoryvault, god bless you my son. If you did read the whole text, where you put your comment, could have informed them now, with real proofs; how wrong they are.

    Bottom line: Truthseeker, what Gore says, is commented on 50 languages, in 100 countries. They don’t ask him for permission / or for money – to spread his theories. He has lies, I have the most solid proofs; use your English and present it skillfully; support my proofs. You being so proud that you succeed to learn your mothers language better than me… English is not my first or second language – I am trying to spread the truth on any language I can (Gore’s lies have reached everywhere – but Japanese or Ukrainian Warmist don’t blame him in public for not mastering all their languages.

    Your real problem is not, for me not presenting the real proofs / facts in silver box… but realizing that I don’t warship the IPCC’s lyrics, as you do. I don’t debate how big GLOBAL warming gases CO2+H2O are, because I have proven ‘’beyond any reasonable doubt’’ that they are not.

    Your perfect grammar is useless, unless you are using it to help. People on the street are not interested in the Warmist words I quoted above (sophisticated crap is for people without solid proofs and responsibility. Lots of innocent people are already suffering, because of that Warmist crap) My proofs can stand any scrutiny – no need for sophisticated crap. THE NAKED TRUTH

    Truthseeker, if you find your white cane and your guide dog – go to my website – read the lot – then comment if my proofs are solid or not – please, don’t grasp at your better English grammar; stop kissing your mirror, we are talking science, not novels.

    10

  • #
    Truthseeker

    Stefan,

    Good to here from you again. Lets get a few things straight. At no point did I say or imply that “grammar is the most important”. What I have been trying to show you is that your ideas, although powerful, are not well communicated and therefore they lose the impact that they should have. I have certainly made no such accusation that “English speaking people are stupid” and to do so is just insulting and beneath you. I defy anyone at any age to understand what is on your website fully. I have certainly tried but have got lost in the bad formatting (superficial I know, but helps a person to read what is written), logic jumps from one point to the next that are not connected well and using phrases like “extra coldness intercepts” which are just mystifying. As for commenting on you website, I find that I can’t because I do not have a WordPress, Facebook or Twitter login and have no intention of getting one. You need to allow a guest login just like WUWT does and you may actually get some comments. Saying that my suggestions to you about getting some help with your communication of your ideas is “sleazy” is just wrong and confirms for everyone that you do not understand the simple advice that I am trying to give you. I am also not suggesting you get scientific help from anyone. I have only tried to give you examples where ideas are communicated well and how powerful that can be. I have to say that that comment #87 is evidence that you are not comprehending the meaning of my words and are finding conflict where there is none.

    You can use whatever words you like and you do not have to please us. The words you have listed here are fine. The only word that I have a problem with is “coldness” and certainly in the way you use it. Cold is a relative term. An ice block of water is cold to a person but warm to a block of frozen nitrogen. You need to express your concepts in terms of heat or energy flows and not in terms of “coldness intercepts”. It is not a term English speakers use. I did a Bing search of “coldness intercepts” with the quotes to get the exact phrase searched and got only 7 hits (most of which are on your website). (I find Google is getting political with its search engine which is why I use Bing).

    You are correct about Gore and how his message of lies is relayed around the world by a publicity machine that none of us real people have access to. However it is a very inconvenient truth that even lies when communicated well have power that they cannot get from scientific merit alone. You have scientific merit on your side. Learn the lesson that communication counts and you need to do it better.

    I will say now the following very clearly so that there is no misunderstanding. I do NOT accept anything that the IPCC has published. I think the IPCC is an entirely political organisation with no scientific credibility whatsoever. You have more scientific credibility in you little finger than the whole of the IPCC ever had. I am trying to use what communication skills I have to help you. I have gone to your web site and tried to read it, but it is seems like an unstructured brain dump of good ideas badly expressed and not comprehensible to the ordinary person. Getting 600 clicks actually proves my point. There is so much interest in this subject that if you had actually been able to convey your excellent ideas well, your hits rates would have been exponentially higher.

    As for kissing my mirror, I do not even use a mirror to shave (electric razors are good that way). Again you are trying to pick a fight with someone who is not against anything you have to say on a scientific basis. By all means talk science, but please lets talk it in a way the makes sense to everyone.

    10

  • #

    truthseecer #88, first regarding the ”extra coldness intercepted” : it says on couple of places on my blog: when you warm up your bedroom – the air expands, 5m3 out the door. where that air goes, in the yard? NO. There was already air in the yard. Those 5m3 are gone up – to increase the volume of the troposphere by 5m3. In relation to the temperature in your bedroom and the upper end of the troposphere is much colder.

    If those 5m3 didn’t go up, that unlimited coldness would have zoomed trough unused. But because is gone, in 3,5 seconds intercepts extra coldness, instantly shrinks and falls down. Because of fast spinning of the planet + winds – that extra coldness falls somewhere far from your room. Otherwise, would have defeated the purpose of warming the room.Nevertheless, warming your room, say by LP gas zero increase in temperature.

    lets for you truthseeker, we explode 50 megaton / 50 bombs simultaneously – the air expands so fast – that speed of the air expanding demolishes concert buildings, not the bag of plutonium. Up where the air expands, in relation to the ground temperature is big, big difference. Therefore, the volume of the cooling system increases. Same as; when the temperature of your car engine increases – if the radiator can enlarge accordingly – you will never have hot engine. Imagine your radiator as a piano accordion.

    When you sleep, cold; you are in foetus position – when you warm up too much – you spread and increase surface area by 3-4%. Troposphere can increase in volume by 100% in seconds. Where the troposphere expands up when warmed, is much colder than your bedroom. I am sure that you will find a lot in that ”condensed version” in PDF form. I have repeated so many thing, so many times for the last 6years… of course in the book is in more details, but people don’t read any more. reason friend suggested: to put some on internet – they feel the monitor is as a TV box…?

    for 9 months, I had a website managed by green administrator = reason I didn’t learn anything. He was sabotaging constantly and was blaming the net. when I realized that is him – was over. Then I asked another… they don’t make them any greener – but I know him for many years. When he was constructing the new website – he was ridiculing me that 30000scientist must be drunk, or I become naive. Unfortunately, after we deposited the text; he must have read the lot – realized that I have proofs.

    After 3 days he was supposed to teach me how to manage it – his behaviour changed completely. made the widget so small letters – cannot read it; another few bad things – I showed him the door. Now I am trying to do something, without trusting them… you are criticizing /demanding perfect job… I am learning on the phone; instructions from oversea… Anyway, I am writing another book, will insert few pages; how hard is to get the truth public. Will be a satirical book. Have a nice weekend. If you read all what is there, will be glad you did, including the 100p in PDF. Especially if you are Australian, you will understand why I get so emotional. CO2 is substituted for hammer and seacle… I came from the east of the iron curtain, can feel better than the rest what is happening here… Nobody even think about the damages they do. Skeptics are obsessed in justifying the carbon, no idea about the damages. please read it, you will see

    10

  • #
    Truthseeker

    Stefan, this is much better. I did read the bedroom analogy, but the problem I had was that I was that I kept hitting the roof of the bedroom. You are saying that when you warm up the bedroom the air goes up by expansion, but there are a number of barriers to this including the bedroom ceiling, roof insulation and roof tiles. I know that they are not air tight, but the room does warm and so the expanded or heated air cannot be going far. So, this analogy is not very good but that does not mean you are wrong in what you are trying to say. The 50 megaton bomb analogy is much better and shows the power of temperature and expanding air.

    I am sorry to hear about your problems with your web site and I can appreciate that it is not your core skill set. Still, using WordPress or LiveJournal or similar generic web blog environments should take away a lot of the pain. Do something simple. Select your whole text and make it all the same font in both typeface and size. This will help enormously. Your downloadable PDF has about 4 fonts on the one page, for no reason. Also, just add a few blank lines occasionally. The unbroken mass of text does not help.

    I think that the discussion about the composition of our atmosphere is irrelevant and the physics of energy transfer by various means, including the expansion of gasses is much more import than anything else.

    10

  • #

    Michael Hammer # 7 and many other of your comments… By sceptical people referring to H2O + CO2 as Greenhouses gases are the precursor of all evil. Now listen very carefully: the earth is not 14-15⁰C warmer than the moon because of the water vapor and CO2 gas. They are not GLOBAL warming gases. Reason the earth is warmer than the moon is: in the earth’s atmosphere are many kilometers tick layer of oxygen and nitrogen. They are TRANSPARENT to the sunlight – the sunlight gets trough it to the ground. But that same O+N prevents the unlimited coldness to get to the ground. Plus, as perfect insulators, O+N are slowing cooling. That unlimited coldness on the moon touches to the ground. So, air lets the sunlight in; but as perfect insulator; keeps the unlimited coldness 40km away up. I am the only one with real proofs, but because of people like Hammer confusing – we will have carbon tax. because of people like him – lots of people are already suffering / billions ripped-off. Therefore, if you read my book, or my website; you will see that O+N are the greenhouses gases, not CO2 + H2O. ENOUGH MISLEADING!!!

    Plus, everybody ignores that the earth produces her own heat, not the moon. Water in the sea, dam is a shock absorber, so is CO2 when is up in the air. They make warmer upper atmosphere, cooler days on the ground. All the crap you have on this page is to confuse. You even know the amount of heat the sun deposits per square meter?! Does it deposit the same amount on the flat valley as on the side of the hill? Which side of the hill, the northern, or western? On half of that square meter has green grass, the other half is dry, or eucalyptus with silvery leafs…

    Nobody knows what was the temperature in the atmosphere even for the last year. Because nobody monitored on every cubic meter of atmosphere for every 5 minutes, for the whole year. I keep repeating: the atmosphere is not as human body – if under the armpit is 1⁰C warmer = the whole body is warmer by that much. Atmosphere doesn’t work that way. If you don’t know that basic, but pretend that you are informing = you are doing more harm to the truth than Al Gore. Shame, shame, shame!!! People, don’t let him confuse you, correct info: http://globalwarmingdenier.wordpress.com

    00

  • #

    Truthseeker # 90 you say that you have being hitting the roof /sealing. Obviously you didn’t hit hard enough, to wake up. You understand very complex issues, so I didn’t think that I need to explain to you in minute details, but I will now: when you warm up your room – the air expands by 10% – 50m3 becomes 55m3. Those 5m3 go out via door, or window, chimney – I didn’t because you know about the openings on your room better, sorry.

    It is important to know that; you by warming your room with fossil fuel – you increased the volume of the troposphere by 5m3. UP THERE, means: troposphere is gone up into the stratosphere. Some confused was getting my book; to explain / ad a bit extra for all of you:

    People, don’t put the stratosphere together in the cooling process as the troposphere, don’t! Stratosphere is very cold, but those gases don’t bring coldness down! Those gases aerosol, ozone, helium; just spin there, useless. Attention is on oxygen + nitrogen; the troposphere. When your room / your town (city island heat) expands by 10% – that extra volume, especially on sealevel; where density is greatest – GOES UP. Or lifts other air wedging – to go up. the difference between your town’s temp and the extra 10% where is gone is big, big

    City of 500km3 = 50km3 goes up, above the troposphere / digs up into the stratosphere – there is colder than 300m below in the troposphere = intercepts more coldness – shrinks extra = becomes heavier per volume – drops down as shotgun pellets – but doesn’t drop down on your town to equalize – therefore, your town is warmer – but that air has cooled somewhere far away EQUAL AMOUNT IS COLDER = ZERO WARMING. Can you dig my formula now Truthseeker? P.s H2O + CO2 up in the clouds don’t make cooler atmosphere. Same as shade-cloth doesn’t make cooler atmosphere. Makes it more pleasant on the ground. Upper atmosphere they make warmer – but up there cooling is much more efficient. CO2 +H2O are shade-cloth gases = good guys. people spend money for shade-cloth, not because is bad – you tell them. water and CO2 make better climate, they should stomp molesting those molecules!!!

    For the last 6 years, I have explained those things thousands of times. Still a Skeptic proudly yesterday was explaining to me that 98 was the hottest, is getting cooler now…? Who told you? IPCC and Plimer… How do they know? 98 wasn’t warmer or colder than last year. PLANET WASN’T WARMING, NOW IS NOT COOLING! Monitoring on 6000 places / nor evenly distribution / not for every 5 minutes / not for every meter altitude from the ground to the stratosphere… they know?? Mr. Hammer knows even for Venus with precision. Because of people like him and Frank, the Warmist are persisting; not because they can have one single legitimate proof. They even started talking of sunspots / bigger sun – skeptics instead of instantly dismissing the crap – they are making bigger backdoor exits for the Warmist – in case. People are suffering / billions squandered, leading Warmist should be in jail already; unfortunately my proofs are skipped /disregarded…!!! back to my website, for real proofs boys http://globalwarmingdenier.wordpress.com

    00

    • #
      Truthseeker

      Stefan, after many attempts, you seem to be finally making progress in explaining yourself. I now get what you are saying (I think). An area at sea level heats up for whatever reason (human activity, natural occurrence) and the heated air expands and the heat transfers up to the top of the troposphere because of the Oxygen and Nitrogen in the atmosphere. This process stops when it reaches the stratosphere because that is where the Oxygen and Nitrogen ends. The Stratosphere gases do not transfer this energy due to their nature. However since it is much colder in the upper troposphere, the heated air contracts and losses heat and falls again to the sea level. By that time the rotation of the planet means the cold air arrives at another location. The overall effect on the planet is zero, but you do have localised temperature variations.

      Like I have said before, your scientific argument that all heating are equally matched by more cold somewhere else has merit. You have to keep repeating yourself because it takes that long before people can understand what it is you are trying to say.

      00

  • #
    kevin Moore

    Mercury,Venus and Mars have highly unstable obliquity. Earths tilt from the perpendicular is stable and maintained by the Moon thus giving the four seasons.

    Some miracles would have to occur before Venus ever became fit for human habitation.

    00

  • #

    All these discussions with W/m2 bandied about willy nilly – yet I think most do not understand the problems of the W/m2 unit and black and grey bodies.
    The difference is subtle, but yet massive.
    Reality is a grey body……..Anyone want to deny that???

    http://www.globalwarmingskeptics.info/forums/thread-1071.html
    What is a Watt???

    In short,
    W/m2 = a power figure for black bodies.
    W/m2 = an energy flow figure for grey bodies.

    ie, it has two very different meanings that are constantly confused / misapplied without people realising what they are doing.
    Thinking about the actual grey body reality, as a series of black bodies, simply can not work – full stop.

    GH “theory” is based upon this misunderstanding most have missed to date.

    00

    • #
      MaryFJohnston

      Good outline derek

      I have never been happy with the use of W/m2 heat balances because they will get every source or sink EXCEPT the important one.

      Basic physics is all that is needed to disprove the Devil CO2 schema.

      People with Environmental Climate Science qualifications misuse and abuse complex thermodynamics for Pop science.

      00

  • #
    margolin

    harry huffman won’t allow any challenges to his ideas to be posted to his blog, and when you do he bans you — not the attitude of someone truly interested in knowledge. he claims that even though venus has a large albedo (a 0.9 bond albedo) the radiation is still absorbed, which clearly violates conservation of energy. and he admits he can’t explain the observed spectrum of the earth, and says its some ‘funneling’ which he can’t explain, but dismisses the gh theory that does explain all observations. he won’t even allow this youtube video to be posted, that clearly shows co2 absorbs ir radiation: http://www.youtube.com/watch?v=kGaV3PiobYk . he can’t be taken seriously.

    00

    • #
      Truthseeker

      Margolin, Dr Huffman was right not to allow the spreading of bad science such as what is being shown in the video that you have linked to. What is really happening is that cold CO2 (compressed gas is always cold to the touch – try it yourself with any compressed gas) is being pumped in the way and the heat of the candle is being spent equalising with the newly arriving colder gas. The same thing would have happen with compressed nitrogen or any other non-flammable gas. Once the equilibrium of temperature had been reached, the candle will appear as bright as before. Some things are powerful because they are simple, like Dr Huffman’s Venus and Earth comparison, because they eliminate extraneous variables. Other things are too simple because they do not take into consideration variables that are important.

      00

      • #

        The video’s rubbish but that’s not the reason. “I can show how carbon dioxide affects climate with this experiment…” Insulting, contemptible dishonesty! The CO2 is indeed absorbing the infrared radiation, but no one disputes that it does! No one but a crank disputes that CO2 absorbs infrared. I can show you that trains are a bad means of public transport by showing you experimentally what one does to a penny placed on the tracks. But no one who disputes the badness of trains would dream of doing so by disputing that they can squash pennies.

        00

  • #
    margolin

    truthseeker: huffman is not a “dr.” — he only has a master’s degree, not a phd.

    00

    • #
      Truthseeker

      Margolin – quite right. I am not sure where that came from. I have used the incorrect honorific.

      00

  • #
    margolin

    truthseeker: if huffman’s idea is so simple and correct, why doesn’t it explain venus’s cooler stratosphere, or earth’s current cooling stratosphere (beyond that from ozone loss); or earth’s observed spectrum of outgoing radiation, which is also recently decreasing with time at the absorption wavenumbers of co2 and ch4? huffman attributes the latter to some undefined ‘funneling’ effect, which he won’t explain. his blog also makes clear that he believe radiation is both absorbed and reflected at the same time, which violates conservation of energy.

    his theory is so simple it’s simple-minded. he won’t answer these objections and instead bans anyone who asks questions he can’t answer.

    00

    • #
      Truthseeker

      Margolin – Mr Huffman specifically compares like to like with respect to air pressure. That is why the comparison is from 1000 mb to 200 mb only. He does not introduce extraneous variables that are not directly comparable. Also I think your point about both absorbing and reflecting is a misquote or at least an incorrect understanding of what he is trying to say.

      00

  • #
    margolin

    truthseeker: tsk tsk. in the video the co2 gas released into the chamber comes to room temperature _very, very quickly_ — in much, much less time than elapses over the course of the video. after all, it is a gas.

    your explanation is wrong.

    00

    • #
      Truthseeker

      Margolin, actually the CO2 is being continuously released into the chamber meaning that there is a constant flow of colder gas. Also, to prove this characteristic of CO2, the person in the video needs to do the same with compressed nitrogen and compare the results. That would be more convincing.

      00

  • #
    margolin

    MaryFJohnston
    Basic physics is all that is needed to disprove the Devil CO2 schema.

    notably, you haven’t provided any, and neither has huffman provided one that is consistent with observations of venus’ statospheric cooling, venus’ and earth’s cooler stratosphere, each planet’s measured spectrum of outgoing radiation, and the earth’s decreasing outgoing radiation at specific wavelengths.

    on the other hand, the greenhouse effect does explain all of these observations. that makes it a far superior theory.

    00

  • #
    MaryFJohnston

    haven’t looked at the Huffman material but I have previous done an outline on CO2 in the warming schema.

    Not going to discuss the science here because it’s so basic.

    If you had done physics at Uni you would know it for yourself.

    00

  • #
    margolin

    MaryFJohnston wrote:
    haven’t looked at the Huffman material but I have previous done an outline on CO2 in the warming schema. Not going to discuss the science here because it’s so basic.

    ha. now i know you are a fake.

    If you had done physics at Uni you would know it for yourself.

    i suspect i’ve done far more physics than you are uni, and i know that simple physics and simpistic answers do not explain the observed facts.

    00

    • #
      MaryFJohnston

      Human CO2 will not, cannot cause rampant Global Incineration.

      Get a life — and an education!

      A PhD in Climate Change Ethics is of no use.

      Do science.

      Be independent – think for yourself.

      00

  • #

    Joanne #84.2.1.1 correcting the host cannot be the most prudent, but you are a good sport: behind your remark of ‘’black and white planet’’ are hiding the answers for the precursor of the Warmist’ evil; but not by the answer you would expect on your comment. ‘’Black’’ is important WHERE is it positioned. If is in the upper atmosphere, where reflection of the sunlight goes directly out – contributes to cooling. B] in the upper atmosphere cooling is much more efficient. I.e. CO2 on the earth during the day goes in upper atmosphere = positive.

    2] if ‘’darkness’’ is on the ground, as soil on earth; which means: there is no dirty cloud up, the ground is darker – absorbs the heat – above the ground is a many kilometers thick cover of the best insulating gases (oxygen and nitrogen) they contribute to extra warming / the heat is trapped below. That’s why oxygen and nitrogen are the greenhouse gases, not CO2. They are transparent – same as the roof of a normal greenhouse – they let the sunlight trough. Then as the glass roof on a greenhouse, O+N are slowing cooling. Imagine if the unlimited coldness was touching the ground on the earth; as it does on the moon. Therefore: earth is not warmer than the moon, because of the CO2; but because O+N let the sunlight, but keep on distance the unlimited coldness. If people can understand those factors – Warmist are disarmed. CO2 also intercepts more coldness at night than O+N – those two factors cancel each other.

    Unfortunately, debating CO2 as a greenhouse gas is giving oxygen to the Warmist = the Skeptics are doing Warmist’ dirty job. Venus is warmer than the earth, for two reasons: 1: because of proximity to the sun, Venus is covered by sunlight on about 55% of the surface, therefore 45% is under darkness at any time, not the earth. How to explain: when you look at the moon or at the sun from the earth, they look same size. B] if Venus had same size moon; then you looked at both objects – the sun would have appeared much bigger. Simple geometry, aluminates more at the curvature and the polls of Venus, but earth doesn’t get that benefit. It’s accumulative, because is constant. Plus at the crown on earth – sunlight goes trough and most of sunlight passes out in space again. On Venus, is not only the size of the planet that intercepts the sunlight, but where the sun appears on the horizon there, from the ground to the end of the stratosphere ALL the sunlight is intercepted.

    Reason 2: because the earth travels a bit faster than Venus into the unlimited coldness + spins much, much faster than Venus = cooling is much more efficient. Venus is a lazy planet – to compare it to a dif on a car / earth to the tread of the car tire. In a very cold night, the tread of the tire would have lost much more heat when traveling, than the dif-housing. Earth is trusting with much bigger velocity into extra coldness, than lazy Venus. NOTHING TO DO WITH CO2!!! But the Skeptics discussing CO2 as a greenhouse gas, are shooting themselves in the foot = that prolongs the Warmist lives (laughter prolongs life) Leading Warmist must be rolling on the carpet from laughter; when the Skeptics get stuck into CO2 as greenhouse / warming gas. Don’t be surprised if Al Gore sends a box of medals, for you to distribute to the sceptical Smarties that call CO2 a greenhouse gas.

    In my book, I suggested to the Skeptics: to spread some sooth on the roof of a normal greenhouse as experiment – the temperature inside would have instantly DECREASED, not increased. Unfortunately, on websites, my English is not perfect; grammar for them is only important, not science…
    Without taking in consideration the factors I pointed + CO2 is a good heat conductor, O+N are not. Yes, that’s why carbon is perfect alloy with steel / similar metallic behaviour, O+N are not – calling CO2 a greenhouse gas is self defeating, Skeptics, CO2 is a ‘’shade-cloth affect gas’’ not Greenhouse: http://globalwarmingdenier.wordpress.com Julia is not introducing ”flat rate carbon tax” because Warmist have one legitimate proof, but because Skeptics are assisting them…

    00

  • #
    margolin

    Truthseeker wrote:
    compressed gas is always cold to the touch – try it yourself with any compressed gas

    um…compressing a gas makes it warmer, not colder.

    00

  • #
    MaryFJohnston

    Right

    It’s only colder when it’s decompressed (ie during decomp)

    It will eventually reach equilib with surrounds at constant P.

    00

  • #
    margolin

    MaryFJohnston wrote:
    It’s only colder when it’s decompressed (ie during decomp)

    Yes. I was correcting Truthseeker’s claim (#95.1)

    It will eventually reach equilib with surrounds at constant P.

    And how long does that take? You can use volume values estimated from the video I showed….

    00

    • #
      KinkyKeith

      During compression it heats up.

      In a cylinder at rest the heated gas will exchange with atmosphere and reach equilibrium ie cool down.

      During decompression eg spraying insecticide from a compressed can; the can will feel cold.

      00

  • #
    margolin

    Truthseeker wrote:
    What is really happening is that cold CO2 (compressed gas is always cold to the touch – try it yourself with any compressed gas) is being pumped in the way and the heat of the candle is being spent equalising with the newly arriving colder gas. The same thing would have happen with compressed nitrogen or any other non-flammable gas.

    The video is a demonstration for television — it’s not intended to be a scientific proof. But that has been done too, first in the 19th century. You can read all about it in John Tyndall’s book, “Contributions to Molecular Physics in the Domain of Radiant Heat”: http://ia700301.us.archive.org/7/items/contributionsto01tyndgoog/contributionsto01tyndgoog.djvu

    Arguing that GHGs don’t scatter IR radiation is silly — next you’ll be arguing that we can’t be sure 1+1=2.

    00

  • #
    margolin

    Truthseeker wrote:
    Also I think your point about both absorbing and reflecting is a misquote or at least an incorrect understanding of what he is trying to say.

    It is not. Here is what he said on his blog:

    “What is truly remarkable is, a good portion of that power is reflected back into space by Venus’s thick cloud cover (which makes the planet particularly bright to Earth observers), yet the Venus atmosphere is still heated by 1.9 times the power that heats the Earth atmosphere, as the temperature data shows.”
    http://theendofthemystery.blogspot.com/2010/11/venus-no-greenhouse-effect.html?showComment=1317033724760#c7722041404145419846

    A gas (atmosphere) can’t be heated by energy that is reflected away — that violates conservation of energy. And it’s the only way Huffman can maintain his claim.

    A proper treatment is as Michael Hammer does in the post at the top: the ratio of energy absorbed is 1.1, not Huffman’s 1.91 .

    00

  • #
    margolin

    stefanthedenier wrote:
    Reason the earth is warmer than the moon is: in the earth’s atmosphere are many kilometers tick layer of oxygen and nitrogen. They are TRANSPARENT to the sunlight – the sunlight gets trough it to the ground. But that same O+N prevents the unlimited coldness to get to the ground. Plus, as perfect insulators, O+N are slowing cooling. That unlimited coldness on the moon touches to the ground.

    This is gobblygook. There is no such thing as “coldness” — there is only heat. Some places have more of it, and some places have less. Atoms do not ‘keep out coldness’ — they scatter photons (and other particles, and other atoms), and that is all. Different atoms scatter photons in different amounts, depending on the atom and on the photon’s wavelength.

    Sheesh. Your ‘science’ sounds like it’s from the Middle Ages.

    00

  • #

    Margolin # 108 is ridiculing me that I am from middle ages, for believing that is such a thing as ‘’coldness’’… Well margolin, I am writing for other people that believe in existence of coldness. I am simplifying things, for people on the street; not to be coned by people like you that CO2, CH4 are GLOBAL warming gases. Lots of people are getting robed, many more will be robed; because of people like you spreading lies. P.s for practical communication, we refer above zero centigrade as warmth / below zero as coldness – zero as the freezing point of water; and we will continue to do so.

    There are few visitors on this blog that have nothing better, but to pick on superficial. I don’t know from which century you are, but we are not on the same frequency. You are similar like them, to waste your boring life on superficial. I am here to prove ‘’beyond any reasonable doubt’’ that people like you are misleading, because they have being brainwashed themselves. In another comment #97 you are lying that CH4 is a GLOBAL warming gas. Those lies are very destructive. Because of my genuine concern I am writing; not to compete with people that demand perfect grammar, or don’t believe that coldness exist.

    1] Let people decide if there is such a thing as coldness on the moon. 2] for the tremendous damages regarding lying that methane (CH4) is a GLOBAL warming gas; would be too much here to prove to you; but go to my website: http://globalwarmingdenier.wordpress.com there is a page on methane – when you learn the damages the Organized Crime in the Warmist Cult are doing – you will see why is so urgent to present the truth to the public; instead of picking on superficial. Read every sentence of it, before start picking on superficial again.

    P.s: photons are the things that illuminate when bounce from nontransparent object, like the lights from your torch. Then again, if you don’t believe in coldness, most probably you wouldn’t believe in darkness also. Because darkness is only less light / less photons. I was thinking that all the nutters are with the Warmist. Clicked on 5-6 Warmist blogs; but they don’t get at each other’s throat for very superficial. See how wrong I can be. I have all the necessary proofs, if the people on the street know about my proofs; Warmist propaganda / carbon tax / Kyoto Protocol would be dead in few months… instead, I am debating if coldness exist… (if we find somebody who doesn’t believe that heat exist also… we might start getting big government funds / grants) isn’t it? if coldness doesn’t exist, those electric heaters should be all confiscated – will save bundles on electricity bill. I wander how Margoling come up with that first; good on ya margolin, keep on the good work.

    00

    • #
      Crakar24

      Stefan let me give you a hand with your English and some up your post another way..

      Margolin- You are an idiot

      00

  • #
    Crakar24

    Oh Gawd

    Some and sum……………

    00

  • #
    Truthseeker

    Margolin, since I am not a physicist and you say that you are (or at least have had significant training in physics) let me point you to this well written analysis of the “greenhouse gas theory” which deals not only with Earth but Venus as well.

    Here is one quote from that study that really should be at the forefront of any scientific discussion of this topic …

    It should be pointed out immediately that the “Greenhouse Effect” is indeed a theory – it is not a benign empirical fact, such as the existence of the Sun, for example. As a theory it has a scientific development which is open to inspection and review. It is extremely curious, from a scientific standpoint, that the word “theory” is almost never associated with the term “Greenhouse Effect” in public and academic circles.

    Kind of hard to argue with that …

    00

  • #

    Post 111 – Re the use of the word “theory” with relation to the “Greenhouse Effect”.
    In respect of the “greenhouse effect” is it not actually a hypothesis, rather than a “theory”?
    Also, given what the “Greenhouse Effect” predicts via AGW and computer modeling, that
    it’s predictions, such as they are, have already failed many, many times,
    so the “Greenhouse Effect” is a failed hypothesis, not a theory.

    NB – Am I correct in thinking a theory is a (experimentally) proven hypothesis of the hypothesis’s predictions?
    Actual repeatable, verifiable experiments that is, not thought experiments that prove nothing.

    00

    • #
      Truthseeker

      Derek,

      I agree with the sentiment, but while CAGW uses the “Greenhouse Gas Theory” at it’s core, the failure of the computer models is really a failure of the CAGW dogma. The “Greenhouse Gas Theory” is based on raidiative properties of various gases, specifically CO2 in this case. While the diagram at the top of this post may indicate the validity of this theory, it does not prove it which is the point of the paper that I had linked to. This may be a little pendantic, but terms and meaning are important in science.

      00

      • #
        KinkyKeith

        “”While the diagram at the top of this post may indicate the validity of this theory, it does not prove it””

        In relation to the above comment it is interesting that Mr hammer doesn’t show where water vap is on this spread.

        Is it actually the “big Bite”?

        00

        • #
          Truthseeker

          KinkyKeith, even more fundamental question to that is why are we expecting a smooth curve of output across the wavelengths? Why would a complex energy output (from the Sun) hitting a complex array of fluids (from a planet) reflect a nice even distribution of energy output levels by wavelength? Is the Sun producing a nice even distribution of output by wavelength? What about all the possible combinations of interactions of the incoming energy and the various and complex fluids (and solids)?

          Even so, your question about water vapour is a good one in the context of the analysis that Michael Hammer is putting forward.

          00

  • #
    KinkyKeith

    Stefan

    Are you having us on? Your English is sometimes perfect and then lapses to your “normal”.

    btw Coldness is not a scientific term and I have previously suggested you were using it to help others understand.

    But.

    maybe it’s time we left Coldness behind and got a little more scientific.

    00

  • #
    margolin

    Stefan: So you say “coldness” is a substance with T < 0 C. And earlier you said oxygen keeps the "distance the unlimited coldness." What does this mean??? That no gas with an average temperature of less than 0 C gets through the atmosphere?? Much of the atmosphere is already below 0 C….

    Your claims are still half-baked gobblegook — and it's not just your English.

    00

  • #

    KinkyKeith #113 I do what I can, for people on the street to understand. I believe that must be simplified for them / not complicated. 2] I believe that: what 80% of the people on the street know, that’s what the politicians do. It is obvious that the excessive scientific jargon puts off the people; same as simple /solid explanations / proofs put you off.

    Reason the Warmist are wining with lies – even though Skeptics are on the side of the truth… unfortunately, the active Skeptics suffer the superiority / inferiority sickness = losing ground. Otherwise, they would have used my real proofs and win against the Warmist, quickly. Using Warmist therms doesn’t make things scientific. Warmist have ”scientific lies” I have the real scientific proofs, used on simple language. I don’t know what you stand for, but if my proofs put you off, because they are expressed in English language, that I learned on the street…

    let me tell you something: even on languages that I can express on snobbish way, I don’t. I have proven with my proofs, no need for me to sound ”scientific” Some of those snobbish Skeptics sound to me as: starving man trying to s%$# – lots of scientific hot air, but nothing solid. I wander why are they wasting their time; singing constantly the Warmist’ lyrics; when they can go fishing, or do something constructive

    00

  • #

    margolin # 114 you are spoiled for arguing. Warmist love to see your attitude in the Skeptic’s camp. I said: oxygen + nitrogen insulate the ground on the earth – on the moon the unlimited coldness touches the ground. That coldness is off minus – 90 degrees centigrade. It actually touches the ground on parts of Antarctic for few days in the year – but the rest of the surface and close to the surface is much warmer than what happens on the side off the moon, where is night. Thanks to the brilliant insulation capability of those two gases.

    Zero Centigrade is not same as -90C. margolin / memoryvault / geeaye. You are using 3 identities; schizophrenics have two, but you have improved even in that department – good on you. You must have being driving up the wall people around you – so, you have invented 3 off yourself. Be proud of you, not many can do that. The Reds are pillaging the democratic west

    I would prefer not to debate if zero C is warmer than minus -90C, because of O+N. If you start arguing senselessly with another one of you, the rest of us to try to get the message, people not to be robed, because off the phony GLOBAL warming; without you twisting what I say. You must have your own problems, but I can’t help you, I am not into medicine…

    00

  • #
    margolin

    I have only one identity, Stefan. Sorry to disappoint you.

    Everything you write sounds like that of a crank — everyone is wrong but you, only you see the light, everyone else is lying, it’s all a conspiracy, etc etc etc. You have weird ways of explaining things, nothing makes much sense, and no one can really be sure what you’re talking about. So you can never be proven wrong. Very clever, in a strange way.

    In what journals has your work been published?

    00

  • #

    margolin #117 1] when they start publishing my work in journals – will be the end of the propaganda. 2] if you, or anybody wants to try to prove me wrong, I challenge to go to my website: http://globalwarmingdenier.wordpress.com it’s not a secret place. The thing is: when they do go; find that my proofs are convincing, but not the way they believed = they start complaining about my English grammar or anything superficial; just not to comment on my proofs. If you have interest in real proofs, go there. If you prefer to avoid what I am proving, just looking to argue… it’s your time you are wasting.

    00

  • #
    richard

    greenhouse gases-

    on earth, during the day, the atmospheric gases cool the earth, at night they slow down cooling.

    Moon, no atmospheric gases, during the day – much hotter than the earth, at night – much colder.

    00

  • #
    richard

    i always thought the heat came from internal on Venus, it is a new planet , the whole surface erupts every so often ( one big volcano) , in the same way the temp of the earth was much hotter billions of years ago.

    The night side is as hot as the day side indicating this. (220 days long)

    Not sure much of the Suns energy penetrates the thick atmosphere on Venus.

    00

  • #
    richard

    of course we see the staggering greenhouse effect in the desert at night where it reaches , now let me see, ah yes zero degrees.

    00

  • #
    co2isnotevil

    Richard,

    the Venusian surface has more in common with the bottom of Earth’s oceans, then with the surface of the Earth. The bottom of the oceans are also at a constant temperature (about 0C). Contributing to the constant temperature, is that ocean water has sufficient insulating properties and is deep enough that a thermocline arises to insulate the bottom of the ocean from the top. In simple terms, the bottom is 0C because below that water freezes and ice floats.

    On Earth, the ocean is defined by a pressure/density/temperature profile as gravity acts on water constrained by its physical cosntants. Similarly, a Venusian ocean arises from the PVT profile of CO2 as dictated by gravity and of course, CO2 is a much better insulator then water. On Venus, it’s a dense CO2 atmosphere which stores most of the planet energy, just as Earth stores energy in ocean water. The Venusian atmosphere and our oceans share all of the required characteristics for establishing an ocean storing energy and a constant temperature ocean bottom.

    George

    00

  • #
    richard

    if you put heat under a pot of water no matter how large, the heat rises and leaves, it is not stored, it’s the same with venus and co2 , the night side is 220 days long and the same temp as the dayside, the heat is coming from internal.

    i doubt the suns energy goes far on Venus due to the thick atmosphere, of course gases under pressure heat up.

    ‘When carbon dioxide gets into the thermosphere, it acts as a coolant, shedding heat via infrared radiation. It is widely-known that CO2 levels have been increasing in Earth’s atmosphere. Extra CO2 in the thermosphere could have magnified the cooling action of solar minimum.
    “But the numbers don’t quite add up,” says Emmert. “Even when we take CO2 into account using our best understanding of how it operates as a coolant, we cannot fully explain the thermosphere’s collapse.”

    the effect of co2 lower causing warming in the lower atmosphere is just political.

    We know on earth that during the day the earth atmos gases are cooling the earth only at night do they slow down cooling.

    00

  • #
    richard

    George ,

    the magic you have bestowed on Co2 on Venus causing the extreme heat through the sun’s energy would be utilized on earth in thermal insulation – it is not!!!

    00

  • #
    co2isnotevil

    No magic at all, and a few 10’s of km’s of dense CO2 is a very good insulator whereas if you heat one end, the other won’t notice. It’s the same reason why as the top of the ocean heats and cools, the bottom temperature is unaffected. In effect, the ocean bottom is isolated from the surface and the Sun. It’s all a matter of scale. At a large enough scale even diamond can isolate hot from cold. Just as things get strange at the small scale, they can get equally counter intuitive at the large scale. BTW, CO2 has a low thermal conductivity making it a relatively good insulator in the first place! http://en.wikipedia.org/wiki/List_of_thermal_conductivities

    Secondarily, it’s a matter of where energy is stored. On Venus, most of the energy is stored between its surface and space in it’s dense CO2 atmosphere, which has an ability to store energy comparable to our oceans. On Earth, most of the energy is stored below the surface in the oceans.

    At some point in the middle of the Venusian atmosphere is a surface defining the point where most solar power has been captured. On Earth, this surface is the combination of planet surface and cloud tops, which can be approximated as the planets surface. The point is that the surface of Venus is not equivalent to the surface of Earth, but that the surface of Earth is closer to a point somewhere up in the atmosphere which would experience seasonal and diurnal variability and that the surface of Venus is closer in character to the bottom of our oceans which don’t.

    George

    00

    • #
      richard

      dear George , left you some other comments but this link may interest you,

      http://www.astrobio.net/exclusive/311/venusian-cloud-colonies

      the surface of venus, covered by lava flows not sure if this made up, but we know the mantle has not hardened and know the surface of the planet erupts.

      00

    • #
      richard

      dear George,

      “On Venus, most of the energy is stored between its surface and space in it’s dense CO2 atmosphere, which has an ability to store energy comparable to our oceans.”

      slight difference by hundreds of degrees and if co2 could do this we would have found the holy grail of energy storage, that huge increase in temp, you have found the holy grail of energy increase.

      00

      • #
        co2isnotevil

        There’s no free energy here. The equilibrium temperature of the Venusian surface is completely decoupled from solar power, hence no night/day and no seasons. The greenhouse effects acts only upon the portion of the planet whose temperature is dependent on solar power. On Earth, this is the cloud fraction weighted sum of clouds and surface upon which solar power is incident. Venus has 100% cloud cover and this is only the clouds. Once cloud temperatures are established, the PVT profile of the Venusian atmosphere dictates the required surface temperature. It’s not a runaway greenhouse effect, but a runaway cloud coverage effect.

        There is power leaving the Venusian surface and entering the atmosphere, but this has nothing to do with the greenhouse effect, and while it adds to the power emitted by the surface, it subtracts from the amount required for the atmosphere to be in equilibrium based on cloud temperatures and it’s PVT profile, as dictated by gravity. The atmospheres effective insulating properties arise owing to the limited depth that solar power penetrates.

        On Earth, surface power mostly originated as incident solar power and we ignore the W/m^2 or so which is emitted from the planet as radioactive decay in the core. On Venus, some emitted surface power also originates from radioactive decay and while Venus is slightly larger than Earth and cools differently, it’s unlikely that it’s internal heat would be a whole lot more than Earths.

        00

        • #
          crakar24

          To put that in laymans terms there is a reason why Venus is the the brightest object in the night sky (hint not much sunlight makes it down to the surface).

          00

  • #
    richard

    George,

    you answered your self,

    “No magic at all, and a few 10′s of km’s of dense CO2 is a very good insulator whereas if you heat one end, the other won’t notice.”

    the heat on venus is at the planet surface as well , you can not have it both ways, Remember the night side at 220 nights is the same temp as the side facing the sun, if co2 could keep the temp at this level for this long it would be used on earth as a thermal insulator- it isn’t.

    by the way george, go to the bottom of the ocean and find one of the erupting volcanoes, there are many hundreds of thousands , sit on one and see if you fry, they are spewing heat out at hundreds of degrees if not thousands.

    00

  • #
    richard

    George,

    “No magic at all, and a few 10′s of km’s of dense CO2 is a very good insulator whereas if you heat one end, the other won’t notice.”

    The sun hardly penetrates the atmosphere and the little that reaches the surface, if it does, supposedly causing the high temps on Venus would be utilized on earth as a heat exchanger or in thermal insulation- it is not ,

    I suggest you try to create the high volume of Co2 in a pressurized clear container leave in the sun – maybe the desert and see if you can reach the high temps of Venus.

    Now in 1821 in Australia the thermometers were exploding at 152 degrees, so be careful.

    00

    • #
      co2isnotevil

      The Sun doesn’t penetrate at all, not because of the CO2, but because of the dense clouds. Also, it’s the pressure gradient that’s important for establishing the surface temperature.

      A container of CO2 at any pressure heats only because the container is holding in the heat. If the container was transparent to IR radiation, the same amount of power entering would be leaving. If you have a glass container, then you do have a case of a real greenhouse effect caused by the glass container, not the CO2 within. CO2 is not opaque to IR radiation, only to a limited number of wavelengths. Glass is opaque across all IR wavelengths.

      00

  • #
    richard

    two things.

    The daytime is over two hundred earth days and yet the nightside stays as warm as the dayside- why?

    The mantle of Venus has not hardened yet , what temp is coming up from the planets interior?

    00

  • #
    richard

    http://www.telegraph.co.uk/science/space/9024912/Nasa-climate-change-test-chamber-to-simulate-Venus-toxic-conditions.html

    If co2 really could increase temps as seen on Venus I think some enterprising soul would have developed something along the lines of what NASA – see above- is building and be selling them for heating.

    00